Вы находитесь на странице: 1из 121

Physics, Chemistry, & Maths Lomonosov

Russia
Physics. Word problems. 8 cells. (Option for writing in MIEM).
In all the proposed objectives, it is desirable to bring the answer except some, possibly convincing arguments in favor of this answer.
1. a). Subway train arrives at the station "Kirov". What is the half open door before?
b). At the moment there is a train stop push. In what direction?
2. Evaluate the tire pressure of a car and a bicycle. Allowed to make a mistake 10 times.
3. Small Chriqui thrown from the plane. He falls to the horizontal plate and an elastic rebound. What is the acceleration of the ball for the first
time after the bounce (for example, when he rose from the plate by 1 cm)?
4. Which of the ball more thermal capacity: that is on the table or that hangs by a thread? (Balls exactly the same).
1979.
Mathematics
1. How many ways are the number 1979 can be written as the difference of two squares of natural numbers?
2. The competition involved 10 skaters. The competition is judged three judges in the following way: each judge in his own distributes
between skaters places (from first to tenth), after which the winner is the skater with the least amount of seats. What is the greatest value may
take this amount from the winner (winner only)?
3. At the deck of 16 cards, numbered from top to bottom. Allowed to take some of the top deck, and then removed and the remaining portion
of the deck, without turning, "punch" into each other. Could it be that after several such operations the cards are numbered from the bottom
up? If yes, for what is the least number of operations it can happen?
Theoretical physics
1. Why is extinguished match if it blow?
2. Is it possible to cram 40 people into a phone booth? (Weight pers. 70 lbs.).
3. In the two goals srelyayut same bullets in different places (see. Fig.). Bullets merges into the washer. Which of them will spread to the wall
(without friction)? m << M

4. "Doppler effect". From the subway exit pop 10 men in seconds. and at a speed of 60 km / h run for trolleybus, delete a speed of 30 km /
h. How many men in seconds. will get on a bus?
Experimental physics

1. Why is the equilibrium is stable?

2. Dependence of the rolling direction of .

3. Why circuit AA breaks away from the table, if the funnel to pour the water?

4. Why if you look through the tube of clear liquid on the inscription of coffee - it does not turn over, and TEA - flips?

3rd Tournament them. Lomonosov


1980.
Mathematics
6 cells.
1. Can a rectangular table 5 * 10 so arrange the numbers so that the sum of any row of numbers would be equal to 30, and the sum of the
numbers of any column is equal to 10?
2. The sum of several numbers is equal to 1. Can the sum of the squares to be less than 0.1?
3. What are the 5 most small integers having an odd number of divisors (in the number of divisors including 1 and the number itself). Explain
that there is a natural.
7-8 cl.
1. a) What are the first 10 positive integers with an odd number of divisors (in the number of divisors including 1 and the number itself).
b) try to formulate and prove the rule that allows the following to find such numbers.
2. a) Is it true that m any triangle can be cut from which you can make a rectangle?
b) The same rectangle -> square.
c) (may be done at home) Is it true polygon -> rectangle?
Mathematics oral 7-8 cells. (Game)
1. By placing a circle 15 minuses. Each of the two playing with his progress can cross out one or two neighboring minus. The player who last
zacherknt minus.
2. 15 stones. One divides into two piles, one (and then alternate with passages) one pile takes himself another divides again into two .. The
player who can not make progress.

3. 15 nuts. Each of the two playing with his progress can not take yourself more than half of all nuts. The player who can not make a move.
Theoretical Physics
1. To determine the density of Hg (mercury) with an accuracy of 1%. For a while all the necessary data many times a day were broadcast on
radio and TV.
2. Will there be enough energy produced by the power plant to evaporate the water passing through the turbines it?
3. Peter 2 times wider Wani, but a 2-fold lower. One of them weighs more?
4. When testing a missile mounted in the tail of the aircraft to protect against attacks from behind, was discovered a surprising fact: when
starting a shell he turns around and catches up with the plane. How to explain it.
Experimental Physics
1. Why does not take off?

2. The tape from the reel. Than H is, the v winder anymore.

3. Why is rotated in the fall and how to make to rotate in the opposite direction?

Chemistry
1. To determine the surface area of the activated carbon. C = 4 * 10 3 J / (kg * O C) L = 2.3 * 10 6 J / kg C = 1 kcal / (kg * O C) L = 500 kcal /
kg
2. What fundamental differences should be expected in the composition of lunar minerals compared to the earth.
3. How to light the burner without matches at the hardware store?

4th Tournament them. Lomonosov


1981.
Mathematics
Grade 6
1. From the results of A divided by 2; A is divided by 2; A is divided by 12; A is divided into 24 three true and one false. What is it?
2. To write the numbers 0; 1; 0; 0. In one step is allowed to add one to any two of them. Can I get that all the numbers become equal?
3. Several boxes together weigh 10 tons. Each of not more than one ton. How trhtonok certainly enough to take away this burden?

Grade 7
1. Motorist rode the 4-wheel drive with a spare tire 5000 miles, changing the tires so that they wore out the same way. How many kilometers
passed each tire?
2. Uvadratnaya area 100 m * 100 m square is lined with slabs of 1 m * 1 m four colors. No two plates of one color are not common side or
top. How many plates can be one of the colors?
3. Several boxes together weigh 10 tons. Each of not more than one ton. How trhtonok certainly enough to take away this burden?
8-9 classes
1. Can there be a trapezoid 4-gon with sides L = 10, 11, 12, 13,
2. Dana repeating decimals with period N . Can this period when multiplying the fraction by 2 or its division by 2
a) increase;
b) decrease?
3. 50 at the round table. They give 101 ball, among them 2 k +1 black. First 3 gets the ball, the other by 2. Then the motion of the balls
according to the rules: if a person has a stroke after 3 balls of the same color - the game is over, if different - reserve the 2 identical, and gives
one a different color neighbor on the right. What is the maximum number of steps can be in the game if
a) k = 1;
b) 1 < k & LT; 49?
Correspondence math competition (6 classes)
1. Each izstoron A , b , C of the triangle over the corresponding sides of a 1 , b 1 , C 1 of the other triangle. Is it true that the area of the first
triangle is greater than the area of the second?
2. There are 10 pairs of black and brown 10 pairs of gloves. Take a random one glove. How many gloves must take to certainly get a pair of
gloves of the same color?
3. The commander of the soldiers decided to build a tower to 4. In this case, the soldier Ivan was superfluous. Then the commander ordered
the building to 5. Again, soldiers Ivanov was superfluous. When in the column on the left more than 6 Ivanov, commander promised him
outfit out of turn, and then in the column 7 Ivanov found a place and no excess remains. How many soldiers could be the commander?
4. On the road connecting the two village, there is no flat areas. Bus always travels uphill at a speed of 15 km / h downhill and 30 km / h. Find
the distance between auls if the way there and back without stopping takes 4 hours.
Math games
1. The square 10 * 10 on graph paper. Two turns straight cut on the grid. One cuts, one of the resulting parts destroys, another gives to
another. He does the same thing. Plays one who got 1 * 1. Who wins the game with the right?
2. There are 10 blue and 10 red balls. Each turn, you can pick up a few balls of one color or more balls of the same color (previously
retrieved) put back. Make two moves in succession. The winner is the one who took all the marbles? Who wins the game if properly?
(Complication - more colors, 3 or 10).
3. 15-gon. Each turn is cut diagonally to get the parts with fewer sides destroyed, and the other - is given to the opponent. Plays who draws a
triangle. Who wins - the first or the second?
(Option - 17-gon).
4. Given the 15 nuts. Played by two, take turns, walked takes itself as a nut (> 0), but less than half. The player who can not make the next
move (this happens when the remaining two nuts). Who wins the game with the right - beginner or the second?
5. The hour hand stands at 12. Two igrayuschihpo queue move her. You can move the arrow to 2 or 3 hours in advance. The winner is the one
who put it on 11. Who wins the game with the right - the first or the second?
6. Written number 1000000. The first player writes natralnyh products of two numbers greater than 1, equal to 1,000,000 Second selects any
factor, and underneath it says equal to the product of two factors. And so on. D. In turn. The player who was unable to make the next
move. Who wins the game with the right - the first or the second?
(Option: the winner is the one who could not make the next move).
Home
1. 25 matches. Take turns, each Statement 1, 2 or 3 matches (light version 1 or 2). The winner is the one who at the end of the game will be an
even number of matches. Who wins: beginner or the second?
2. A - natural. The two take turns playing smash all of the available range (the first time during the whole set - the number of A ) to natural or
long-terms 1 natural factors. In one move, one number is divided by 2 terms or 2 factor. The player who can not make the next move. Who

wins the game if properly?


a) A = 10
b) A = 11
Theoretical Physics
6-7 class
1. Boy Loew boating and water photographed beach. When he published the photos, I could not understand where the image bank, and where
his reflection in the water. Specify the number of possible ways to do this.

2. Flatland - a flat country. It has only two dimensions. How can I attach the wheel to the car in Flatland? (Make-axis, as in the threedimensional world, it is impossible - she has nowhere to hang around there).
3. In carrying out accurate weighing scales placed in a hood, from under which evacuated. Why do it?
4. Mikhail Lomonosov issedoval movement of air in mines complex profile. In what direction will show through the winter and summer in
the mine, the illustrations?

8-9 class
1. Boy Serge watched TV and decided to photograph it interesting image. When he published the photos, I saw them on the light and dark
bands. How Come? What can we say about the device gate?
[task involves, but has been withdrawn.]
2. Towards each other fly two identical dumbbells. They elastically collide. How will fly dumbbells after the collision is over? Consider the
case of v 1 = v 2 and v 1 = / = v 2 .

3. The load falls on a vertical spring. At what height it will have a top speed?

4. In the foam float hidden two pieces of lead. What sluagh float will:
1) sink:
2) protrude from the water;
3) to tip over the surface of the water? foam << 1 m 1 = m 2 .

Experimental Physics
1. light a match and hold it horizontally. Burning, match bends upward. How Come?
2. strung on a thread button, which is passed back and forth. Then arrange the chopper, spreading his hands wearing the thread loops. Rate
speed buttons.
3. Take the flexible tube and omit one end of the vessel with water. Little water in the suction pipe, and omit the other end below the water
level in the first vessel, but on the other vessel. From the second end pours water, while the tube can be lifted its middle part is quite
high. Then lift the other end. For some time the water pours more, and then stops. And if now again omit the other end, it is not will
pour. Explain.

6th Tournament them. Lomonosov


1983.
Mathematics
5-7 cl.
1. All natural numbers are divided into good and bad. It is known that if A - good, the A +6 also good, and if B - is poor, and B +15 bad. Can
among the first 2,000 numbers be exactly 1,000 good?
2. What is the maximum number of pawns can be placed on a chessboard, if you can not put the e4, and if no two pawns can not stand on the
sidelines, symmetric with respect to e4?
3. How many pairs contained in the expansion of 1984! ?
8-9 cl.
1. In the first row are written in ascending order of numbers divisible by 9: 9, 18, 27, 36, ... in the second row - the sum of the first digits of
the series: 9, 9, 9, 9, ... In any place in the second row for the first time to meet the number 81? What is the number in the first row
corresponds to? That will meet before 4 times the number 27 or 1 times the number 36?
2. Change machines are changing:
20 -> 15, 2, 2, 1,
15 -> 10, 2, 2, 1,
10 -> 3, 3, 2, 2
1 After the exchange of rubles. 25 kopecks. Copper Copper determine what was silver.

( Note : in 1983 in the Soviet Union were in the course of the coin 1 rub. 50 kopecks., 20 kopecks., 15 kopecks., and 10 kopecks. white,
conventionally called the silver and coins 5 kopecks., 3 cop., 2 kopeks. 1 cop. yellow, conventionally called copper).
3. (8 cells.) In 1983 the plane point and a circle of radius 1. Prove that there is a point on the circumference, the sum of distances between all
of these points is not less than 1,983 in 1983.
4. (9 cl.) Two circles intersect line. Prove that the angle ABC = angle DEM .

Theoretical Physics
5-7 cl.
1. When will emerge from the bath faster one and the same volume of water - when it is a man or when it is not?
2. On the popular lecture said that the height of the Eiffel Tower 302 meters 15 centimeters. "Well, no," thought one of those present. Why he
thought so?
3. Archimedes had to lift a heavy load, and had two arms: He thought - "for us, weightlifters, it's just right." Which lever he chose?

8-9 cl.
1. Task 1 for 5-7 cells.
2. Task 2 for 5-7 cells.
3. How to determine the weight of the boat, having a coil of rope? It takes place during the boat ride.
4. When you turn on the speakers in the sequential translational network open one disables all in parallel - one short izamykanie disables
all. How to include?
Experimental Physics (5-9 cells).
1. (5-7 kl.) How to measure the diameter of the thread usual line?
2. How to weigh a book that weighs about 2 times larger than the limit dynamometer?
3. On the inclined plane rolled 3 bubble from the glue - empty, half-full with water and. Why half-stops before?
(8-9 kl.) Which part of the energy is spent on friction bubble?
4. Those bubbles are. With a weak push the base falls empty, with an average - complete. Why such a sequence?
5. (8-9 kl.) If narisovanna star * look through the slit, cut with a razor in the paper, the visible beam 1-2. If you rotate the gap, the rays begin
to spin. How Come?
Chemistry
1. The ideal syringe needle is lowered into the water and pull the piston. How does the rate of filling the syringe on how much pushed the
plunger and how quickly he pushed? Explain your answer.
2. Can be a simple substance molecule polar? If not, why not, if so, give examples.
3. As a fairly well-equipped laboratory light the alcohol lamp, if you forgot your home matches? Come up with more examples.

4. In the last century there was much debate about the structure of the benzene molecule, which has a composition of C 6 H 6 . Try to draw all
of the structural formula, which can describe the composition of such a molecule. Note that the C atoms can be connected double and triple
bonds.
5. Why concentrated hydrochloric acid fumes in air, and concentrated ammonia solution - no?
6. It is known that solutions of iodine in water and alcohol have a brown color, and iodine vapor and carbon tetrachloride solution purple. What could you explain this fact?
7. When bubbling xenon, methane under a pressure of 1 atm. through water at 2 O C precipitate colorless crystals of Xe * 5.75H 2 O and
CH 4 * 5.75H 2 O, which easily decompose with the release of the gas to under heating. What do you think, what is the structure of these
compounds?
8. Can one get two acid salts of the two acids - salt, two bases - salt, the salts of the two - base. If the process is possible - Write the reaction if not, try to justify their opinions.
9. How does the reaction rate on the concentration and temperature / ceteris paribus /? How could you explain this relationship?
10. Some waste production is oil containing water. It turns out that this oil burns even better dry. How can you explain this and how to do it?
11. What, in your opinion, different substances corresponds to the following formulas:
FHH
| | |
Cl-CH Cl-CC-Br
| | |
Br FF
Please note that due to carbon directed corners of a tetrahedron. Draw the molecule thereof.
12. What are the inorganic substances in living organisms and why are they needed?
13. 1.3 liters made of 130 g of a clear crystalline substance, which is in the process of dissolution zhidgosti level has not changed, but its
temperature is lowered. After complete dissolution of the substance liquid is evaporated, with remaining 0.13 g of solid residue. Determine
what kind of fluid and a substance dissolved in it.
Competition in Mathematics
Tasks
1.

On the sides of the hexagon was recorded six numbers, and each vertex - a number equal to the sum of
the two numbers on the allied side. Then, all the numbers on the sides and top of a single number
erased.Is it possible to restore the number on the top?

2.

Vertices A, B, C of a triangle are connected to points A 1 , B 1 , C


Can the midpoints of AA 1 , BB 1 , CC 1 lie on the same line?

3.

Three chess players A, B and C played a match tournament (everyone with everyone played the same
number of games). Could it be that by the number of points won first place A, C - last, but by the number
of victories, on the contrary, A finished last, C - the first (for the victory is awarded one point for a draw a half-point)?

, which lie on opposite sides (not top).

Problem solving in mathematics competition


1.

Yes I Am. Assume that a number of 1 , a 2 , a 3 , a 4 , a 5 , a 6 are recorded on the ribs 6 in order of the
polygon counterclockwise. Then, six-sided polygon vertices will be recorded (counterclockwise) of the
following:
A 1 + a 2 , a 2 + a 3 , a 3 + a 4 , a 4 + a 5 , a 5 + a 6 , a 6 + a 1 . We can assume without loss of generality
that the only erased in the top number - is a 1 + a 2 . Then it can be recovered by the formula:
(A 2 + a 3 ) - (a 3 + a 4 ) + (a 4 + a 5 ) - (a 5 + a 6 ) + (a 6 + a 1 ).
After removing the parentheses it turns out that all the numbers except a 1 and a 2 , enter the right pane,
double, and with different characters.

2.

The middle of said segments lie on the respective average lines of the triangle (as an average parallel to
the base line). They also lie inside the triangle (point A 1 , B 1 , C 1 can not celebrate the vertices). Since
the line can not cross the three sides of the triangle, the midpoints of the segments and can not lie on
the same line.

3.

Yes I Am. Place in the match-tournament determined by the difference between the number of wins and
the number of lesions. Here is a table of the tournament in 8 circles (as in match-tournament contenders
in the 50s and early 60s) that meets the specifications (in the row corresponding to the player and the
specified number of victories over the other players):
A
A
B

glasses

victory

8.5

7.5

Competition for mathematical games


Terms Games

1.

Honeycomb (1x1 squares) are arranged in a 9x9 square. In the central box instead of honey - tar. In one
move is allowed to make vertical or horizontal incision (squares on the sides) and eat one of the cut
pieces, in which there is no tar. Play two of the moves made in turn, loses the one who has to eat
tar. Who's winning? (You can consider the case where the tar is not in the center.)

2.

On a strip of size 1x8 in the 4th, 6th and 8th cells are chips. Each move any of them can be pushed to
the left by any number of cells, so as not to jump over another chip and not get to the occupied
space.Played by two, take turns. The player who can not make a move. Who's winning?

3.

The arrow on the clock shows twelve hours. In one move is allowed to move in a clockwise direction to
two or three hours. The winner is the one who put the arrow on the eleven. Who will win?

4.

The cake is in the shape of a regular pentagon. In one move is allowed to make an incision along the side
or diagonally. The winner is the one whose turn is after the cut a piece containing cherry, located in the
center of the cake. Who will win? Consider a plywood table on which is painted a regular pentagon in the
center of which stands a vase, and the loser is the one whose turn after vase falls.

5.

In a series of written seven minuses. In one move, you can fix on the plus one or minus two
adjacent. The player who can not make a move. Who will win? Consider the eight minuses in a row; daisy
with eight or nine petals.

The story of what math games


In this section, we did not give solutions of games offered at the competition. Otherwise, you will not be interesting to play them. But it is
useful to discuss what a mathematical game .
With conventional games like chess, checkers, or of them have in common is that there are two players chasing opposing goals. A difference
is that chess would be a fatal blow, if it turned out that the white, for example, with the right game always win. A mathematical games just
interesting to invent the correct ways to play. Mathematics, however, are able to prove that the right way to play there in the ring games. But
to find the way to chess and Go is not yet possible, even using super-computers (for checkers situation is somewhat different). But those
games that we'll play a lot easier, and so they manage to find out who wins the game at the right.
And how do understand what the right play? After a sequence of moves in the game alternates. Suppose for example, I wrote down the right,
winning sequence of moves for me. After all, the enemy is not obliged to follow it!
However, if it is possible to formulate a rule, following which one player always gets a prize at all possible moves of the second, it is clear
that the outcome of the game is defined (it should be, of course, know this rule). Mathematics in such cases we say that the first player has a
winning strategy . It is the search of a winning strategy and one of the players is a mathematical game.
Of course, if the rules of the game you like, you can it some time to play with his friends, but after a while you will notice that those wishing
to participate in the game with one of the parties is getting smaller. And then, when the strategy will be known to all, the game play will not
be none.
Let's look at some examples of mathematical games.
Task 1. There are two piles of stones, 8 pieces each. Two persons play the following game: in one move is allowed to take a handful of any
number of stones. Who can not make a move - lost. Find a winning strategy for one of the players.
Before embarking on the search strategy, we advise you to play this game with someone else, or at least one of the two players at once. If you
have formed a definite opinion about who wins in this game, try to formulate a rule that should guide the player, ie, strategy. After that, read
on and compare their solution with ours.

Note that each turn number of stones in one of the piles decreases. So the game can not last indefinitely, no matter how the players play.
Solution 1. The winner is the second player. To do this, he must adhere to the following strategy (called "symmetric strategy"): after each of
the first player moves need to equalize the number of stones in a pile.
Why such a strategy is called symmetric? Not only in this game, but also in many other positions, some are symmetrical. So, the symmetric
strategy is that each turn a player achieves a symmetrical position. In this case, the position symmetrical, if the number of piles of stones in
the same way.
So check that the symmetric strategy is the solution of the game and provides a second player win.
The first question is whether the player will always follow it? To answer this question, think about the position in which the second player can
not equalize the number of stones in a pile? Only if the stones in a pile ALREADY equally. But this situation to move the second player can
not occur! Indeed, initially have a symmetrical position. The first player to take a certain number of stones, and the position becomes
asymmetric.
Now the second question: whether the second player wins, sticking symmetric strategy? As already mentioned, the number of stones in the
pile is constantly decreasing. The course can not be done when there are no stones. But this position is symmetrical, and there she was after
the move of the second player.
Objective 2. There are a handful of stones 4 to 9, 10, 12 and 18 stones respectively. At its course the player can take one of a handful of 1 or
2 stones. Who can not make a move - lost. Determine the outcome of the game (ie, to find a winning strategy for one of the players).
This task is more difficult to last. However, try to solve it, and then read the solution given here.
Solution 2. The situation in which it is impossible to make progress in this game is also unique stones --- all players. It turns out that the first
player can make sure that he took the last stone. A winning strategy for the first player: do such a course, the number of stones in each pile
divisible by 3.
Is it always the first player can follow this rule? Note that on the first turn possible: take one stone from the heap, in which 10 stones. The
second player has to take its course from one of a handful of 1 or 2 stones. If its course in every pile of stones was the number divisible by 3,
and if the first player follows the preceding rule, this is true, then before the next move of the first player in exactly one pile of stones is not
the number is divisible by 3. It is clear that the first player can follow the preceding rule, taking from this heap remainder of dividing the
number of stones on the 3.
So, after the move of the first player the number of stones in a pile to be divided by 3. But the game ends when the stones in piles - 0 0 0 0 ie the first player wins.
Competition in Physics
Assignments
Lower grades (6-7)
1.

Why icicles are usually located only one side of the roof?

2.

Air and water are transparent. Why is opaque fog?

3.

Sergei Archimedes believes that he is lucky with good weather. More than once he noticed that when the
clouds surrounded the entire horizon, namely the sun shines on it. Does Serge rights?

Upper grades (8-9)


1.

Sergei Archimedes believes that he is lucky with good weather. More than once he noticed that when the
clouds surrounded the entire horizon, namely the sun shines on it. Does Serge rights?

2.

Wider or narrower than the moon moonlit path on the water (meaning angular dimensions)?

3.

Rats and other rodents some teeth stay sharp all the time, despite the constant stitching. This is because
their teeth consist of materials with different hardness. Explain how such self-sharpen your teeth and
how they are distributed substances with different hardness.

Competition in Astronomy

Questions
1.

When people built the oldest observatory? Why is it needed?

2.

Eratosthenes in the 2nd century BC, knowing nothing about other countries, to measure the size of the
Earth. How is it managed?

3.

How to change the speed of the satellite, if it starts to slow down?

4.

They say: "light and not heat." About what I may say so, and why? And whether there is other way
around?

5.

Why is the planet in the sky after the Sun go?

6.

To the sky wants to make all well verree great impression, and in fact quite small and nevzrachnenky?

7.

What constellations in the sky most ancient? Do you know why they are so named?

8.

Why do not we see how stars are born? Can we see how they die?

9.

When humanity begins to travel to other planets that we will be interested in it?

10. What are the similarities between the universe and the cotton candy?
Answers to questions of competition on astronomy
1.

The answer to this question depends on what is meant by the observatory. A. If this religious building, at
least to some extent related to astronomy, the oldest known structure - Stonehenge - the remnants of a
giant megalithic stone structure, built at the turn of the Stone and Bronze Ages (1900-1600 BC) on the
territory of modern England. At Stonehenge carried out not only ritualistic ceremonies, but also, due to
the special orientation of the individual parts of the structure, was conducted through calendar days,
marks the beginning of the seasons. B. If you have in mind for the construction of regular visual
observations of stars, it is an observatory of ancient Babylon (1st half of the 2nd millennium BC). There
were widespread predictions of important events occurring on the basis of the heavenly signs, and
thought: the more precise observations, the more certain prediction. It appeared in Babylon associated
with astronomical events mathematical time-scale (8th century BC). B. If we talk about observatories
equipped with tools, then we should talk about ancient Greece, about the age of antiquity.

2.

Eratosthenes lived in Alexandria in northern Egypt. And in the south of Egypt was the city of Siena, which
was a wonderful well. He was vertical and deep, and at noon, the longest day the sun illuminates its
bottom, ie was at its zenith. Eratosthenes measured figured in one of those days the sun zenith angle
(angle to the vertical, it was 7.2 O ), imagine the Earth ball, put Siena and Alexandria on one meridian
and ascribe 7.2 O meridian arc distance between cities. Knowing this distance and the fact that a full
circle is 360 O , it is not difficult to find the proportion of the circumference of the Earth's
meridian. Eratosthenes was 46,000 km. Excellent accuracy!

3.

If the satellite will begin to slow down, it will spiral closer to the ground. Hence, it has a circular orbit at
an initial vertical velocity component to appear. If braking is slow enough, then for each new round
circular orbit can be considered, but a smaller radius H. Since circular velocity V = (GM / (R + H)) 1/2 (R radius of the Earth, H - height above the earth, M - mass of the Earth, G - gravitational constant), we see
that V cr increases with decreasing H.
ie with slow growing and normal braking, and the tangential component of the velocity, and hence the
full velocity vector. The growth rate is due to the completion of the kinetic energy of the satellite mv 2 / 2
due to the potential energy mgH.

4.

"Light, and does not heat" - can be said about any source that emits in the optical range at low
power. On the contrary, you can say about the powerful sources of electromagnetic radiation with a peak
in the infrared region of the spectrum. Example - iron.

5.

The solar system is highly flattened. Planets very slightly spaced from the central plane in which they
revolve around the sun, all in one direction. This plane is called the ecliptic. Therefore, the observer on
Earth, and it would seem that the planets and the Sun on the background of distant stars move in the
sky on the same line.

6.

Solution of this problem is not given, because the collector could not learn what it had in mind the
authors.

7.

It is difficult to say whether the names endowed beautiful configuration brightest stars Adam and biblical
patriarchs. Ancient texts mentioning the constellations date from the second half of the second
millennium BC Perhaps the oldest known - Old Babylonian text (1700 BC). This is the prayer to the starry
gods. Referred to the constellation - participants myths (monthly myths, according to the lunar
cycle). Obviously, the first in this regard, attracted the attention of circumpolar constellations, especially
the Big Dipper and the North Star in Ursa Minor, because the rotation of the sky takes about an axis
passing near the North Star. These constellations are the first to receive sustainable titles (based on
mythological), which in most cases are not associated with modern, inspired later periods (Antiquity, the
Middle Ages, the era of geographical discoveries).

8.

Stars are born when compressed gas and dust clouds, with a sufficiently high degree of compression
when warming up in the center is sufficient to "ignite" thermonuclear reactions. Death of a star occurs
when the star is no longer in a stable condition under normal parameters (density, size, somehow
comparable with solar). In this sense, the stars in store for several kinds of deaths, according to their
initial weight.
When M <1,4 solar mass star "dies", becoming the new white dwarf,
with 1,4 <M <3,3 dying corresponds to the birth of a neutron star as remnant of a supernova explosion.
When M> 3,3 death marks the formation of a black hole. All deaths can be observed, and all of them the result of the depletion of nuclear fuel.
Neutron stars and black holes manifest themselves in a particular form of the spectrum of the radiation
and the nature of the impact on normal companion stars.

9.

It will not happen soon. Will also be of interest to energy and mineral resources.

10. Observations suggest that the universe has a honeycomb porous structure. In most large-scale clusters
of galaxies are distributed uniformly in space, but on a smaller scale homogeneity structured
"emptiness" surrounded by "thin wall". And in cotton candy.
Competition for Chemistry
Assignments
1.

When processing of ultrasonic radiation mixture of starch, an aqueous solution of potassium iodide and
carbon tetrachloride observed a blue color.
Suggest an explanation for this phenomenon.

2.

The reaction of the two gases mixed in a molar ratio of 1: 2, in a closed volume leads to the formation of
hydrofluoric acid solution with a mass fraction of 69%.
What is the gases. Answer motivate.

3.

Graphically the dependence of the conductivity of aqueous solutions of sulfuric acid and sodium sulfate
concentration and explain et dependence.

4.

Some alum (hydrated A 1 + B 3+ (SO 4 ) 2 * 12H


hydrogen. Determine their composition.

5.

Alcohols do not conduct electric current while alkalis, also containing a hydroxyl group in a molten state
or in an aqueous solution the electric current is performed.
What is explained?

6.

The inorganic compound contains 12.28% nitrogen, 3.50% hydrogen, 28.07% sulfur and oxygen.
What this compound and its properties are briefly characterize.

O) containing 51.78% oxygen and 4.53%

Competition in Mathematics
Tasks
1.

On the sides of the hexagon was recorded six numbers, and each vertex - a number equal to the sum of
the two numbers on the allied side. Then, all the numbers on the sides and top of a single number
erased.Is it possible to restore the number on the top?

2.

Vertices A, B, C of a triangle are connected to points A 1 , B 1 , C


Can the midpoints of AA 1 , BB 1 , CC 1 lie on the same line?

, which lie on opposite sides (not top).

3.

Three chess players A, B and C played a match tournament (everyone with everyone played the same
number of games). Could it be that by the number of points won first place A, C - last, but by the number
of victories, on the contrary, A finished last, C - the first (for the victory is awarded one point for a draw a half-point)?

Problem solving in mathematics competition


1.

Yes I Am. Assume that a number of 1 , a 2 , a 3 , a 4 , a 5 , a 6 are recorded on the ribs 6 in order of the
polygon counterclockwise. Then, six-sided polygon vertices will be recorded (counterclockwise) of the
following:
A 1 + a 2 , a 2 + a 3 , a 3 + a 4 , a 4 + a 5 , a 5 + a 6 , a 6 + a 1 . We can assume without loss of generality
that the only erased in the top number - is a 1 + a 2 . Then it can be recovered by the formula:
(A 2 + a 3 ) - (a 3 + a 4 ) + (a 4 + a 5 ) - (a 5 + a 6 ) + (a 6 + a 1 ).
After removing the parentheses it turns out that all the numbers except a 1 and a 2 , enter the right pane,
double, and with different characters.

2.

The middle of said segments lie on the respective average lines of the triangle (as an average parallel to
the base line). They also lie inside the triangle (point A 1 , B 1 , C 1 can not celebrate the vertices). Since
the line can not cross the three sides of the triangle, the midpoints of the segments and can not lie on
the same line.

3.

Yes I Am. Place in the match-tournament determined by the difference between the number of wins and
the number of lesions. Here is a table of the tournament in 8 circles (as in match-tournament contenders
in the 50s and early 60s) that meets the specifications (in the row corresponding to the player and the
specified number of victories over the other players):
A
A
B

glasses

victory

8.5

7.5

Competition for mathematical games


Terms Games

1.

Honeycomb (1x1 squares) are arranged in a 9x9 square. In the central box instead of honey - tar. In one
move is allowed to make vertical or horizontal incision (squares on the sides) and eat one of the cut
pieces, in which there is no tar. Play two of the moves made in turn, loses the one who has to eat
tar. Who's winning? (You can consider the case where the tar is not in the center.)

2.

On a strip of size 1x8 in the 4th, 6th and 8th cells are chips. Each move any of them can be pushed to
the left by any number of cells, so as not to jump over another chip and not get to the occupied
space.Played by two, take turns. The player who can not make a move. Who's winning?

3.

The arrow on the clock shows twelve hours. In one move is allowed to move in a clockwise direction to
two or three hours. The winner is the one who put the arrow on the eleven. Who will win?

4.

The cake is in the shape of a regular pentagon. In one move is allowed to make an incision along the side
or diagonally. The winner is the one whose turn is after the cut a piece containing cherry, located in the
center of the cake. Who will win? Consider a plywood table on which is painted a regular pentagon in the
center of which stands a vase, and the loser is the one whose turn after vase falls.

5.

In a series of written seven minuses. In one move, you can fix on the plus one or minus two
adjacent. The player who can not make a move. Who will win? Consider the eight minuses in a row; daisy
with eight or nine petals.

The story of what math games


In this section, we did not give solutions of games offered at the competition. Otherwise, you will not be interesting to play them. But it is
useful to discuss what a mathematical game .
With conventional games like chess, checkers, or of them have in common is that there are two players chasing opposing goals. A difference
is that chess would be a fatal blow, if it turned out that the white, for example, with the right game always win. A mathematical games just

interesting to invent the correct ways to play. Mathematics, however, are able to prove that the right way to play there in the ring games. But
to find the way to chess and Go is not yet possible, even using super-computers (for checkers situation is somewhat different). But those
games that we'll play a lot easier, and so they manage to find out who wins the game at the right.
And how do understand what the right play? After a sequence of moves in the game alternates. Suppose for example, I wrote down the right,
winning sequence of moves for me. After all, the enemy is not obliged to follow it!
However, if it is possible to formulate a rule, following which one player always gets a prize at all possible moves of the second, it is clear
that the outcome of the game is defined (it should be, of course, know this rule). Mathematics in such cases we say that the first player has a
winning strategy . It is the search of a winning strategy and one of the players is a mathematical game.
Of course, if the rules of the game you like, you can it some time to play with his friends, but after a while you will notice that those wishing
to participate in the game with one of the parties is getting smaller. And then, when the strategy will be known to all, the game play will not
be none.
Let's look at some examples of mathematical games.
Task 1. There are two piles of stones, 8 pieces each. Two persons play the following game: in one move is allowed to take a handful of any
number of stones. Who can not make a move - lost. Find a winning strategy for one of the players.
Before embarking on the search strategy, we advise you to play this game with someone else, or at least one of the two players at once. If you
have formed a definite opinion about who wins in this game, try to formulate a rule that should guide the player, ie, strategy. After that, read
on and compare their solution with ours.
Note that each turn number of stones in one of the piles decreases. So the game can not last indefinitely, no matter how the players play.
Solution 1. The winner is the second player. To do this, he must adhere to the following strategy (called "symmetric strategy"): after each of
the first player moves need to equalize the number of stones in a pile.
Why such a strategy is called symmetric? Not only in this game, but also in many other positions, some are symmetrical. So, the symmetric
strategy is that each turn a player achieves a symmetrical position. In this case, the position symmetrical, if the number of piles of stones in
the same way.
So check that the symmetric strategy is the solution of the game and provides a second player win.
The first question is whether the player will always follow it? To answer this question, think about the position in which the second player can
not equalize the number of stones in a pile? Only if the stones in a pile ALREADY equally. But this situation to move the second player can
not occur! Indeed, initially have a symmetrical position. The first player to take a certain number of stones, and the position becomes
asymmetric.
Now the second question: whether the second player wins, sticking symmetric strategy? As already mentioned, the number of stones in the
pile is constantly decreasing. The course can not be done when there are no stones. But this position is symmetrical, and there she was after
the move of the second player.
Objective 2. There are a handful of stones 4 to 9, 10, 12 and 18 stones respectively. At its course the player can take one of a handful of 1 or
2 stones. Who can not make a move - lost. Determine the outcome of the game (ie, to find a winning strategy for one of the players).
This task is more difficult to last. However, try to solve it, and then read the solution given here.
Solution 2. The situation in which it is impossible to make progress in this game is also unique stones --- all players. It turns out that the first
player can make sure that he took the last stone. A winning strategy for the first player: do such a course, the number of stones in each pile
divisible by 3.
Is it always the first player can follow this rule? Note that on the first turn possible: take one stone from the heap, in which 10 stones. The
second player has to take its course from one of a handful of 1 or 2 stones. If its course in every pile of stones was the number divisible by 3,
and if the first player follows the preceding rule, this is true, then before the next move of the first player in exactly one pile of stones is not
the number is divisible by 3. It is clear that the first player can follow the preceding rule, taking from this heap remainder of dividing the
number of stones on the 3.
So, after the move of the first player the number of stones in a pile to be divided by 3. But the game ends when the stones in piles - 0 0 0 0 ie the first player wins.
Competition in Physics

Assignments
Lower grades (6-7)
1.

Why icicles are usually located only one side of the roof?

2.

Air and water are transparent. Why is opaque fog?

3.

Sergei Archimedes believes that he is lucky with good weather. More than once he noticed that when the
clouds surrounded the entire horizon, namely the sun shines on it. Does Serge rights?

Upper grades (8-9)


1.

Sergei Archimedes believes that he is lucky with good weather. More than once he noticed that when the
clouds surrounded the entire horizon, namely the sun shines on it. Does Serge rights?

2.

Wider or narrower than the moon moonlit path on the water (meaning angular dimensions)?

3.

Rats and other rodents some teeth stay sharp all the time, despite the constant stitching. This is because
their teeth consist of materials with different hardness. Explain how such self-sharpen your teeth and
how they are distributed substances with different hardness.

Competition in Astronomy
Questions
1.

When people built the oldest observatory? Why is it needed?

2.

Eratosthenes in the 2nd century BC, knowing nothing about other countries, to measure the size of the
Earth. How is it managed?

3.

How to change the speed of the satellite, if it starts to slow down?

4.

They say: "light and not heat." About what I may say so, and why? And whether there is other way
around?

5.

Why is the planet in the sky after the Sun go?

6.

To the sky wants to make all well verree great impression, and in fact quite small and nevzrachnenky?

7.

What constellations in the sky most ancient? Do you know why they are so named?

8.

Why do not we see how stars are born? Can we see how they die?

9.

When humanity begins to travel to other planets that we will be interested in it?

10. What are the similarities between the universe and the cotton candy?
Answers to questions of competition on astronomy
1.

The answer to this question depends on what is meant by the observatory. A. If this religious building, at
least to some extent related to astronomy, the oldest known structure - Stonehenge - the remnants of a
giant megalithic stone structure, built at the turn of the Stone and Bronze Ages (1900-1600 BC) on the
territory of modern England. At Stonehenge carried out not only ritualistic ceremonies, but also, due to
the special orientation of the individual parts of the structure, was conducted through calendar days,
marks the beginning of the seasons. B. If you have in mind for the construction of regular visual
observations of stars, it is an observatory of ancient Babylon (1st half of the 2nd millennium BC). There
were widespread predictions of important events occurring on the basis of the heavenly signs, and
thought: the more precise observations, the more certain prediction. It appeared in Babylon associated
with astronomical events mathematical time-scale (8th century BC). B. If we talk about observatories
equipped with tools, then we should talk about ancient Greece, about the age of antiquity.

2.

Eratosthenes lived in Alexandria in northern Egypt. And in the south of Egypt was the city of Siena, which
was a wonderful well. He was vertical and deep, and at noon, the longest day the sun illuminates its
bottom, ie was at its zenith. Eratosthenes measured figured in one of those days the sun zenith angle
(angle to the vertical, it was 7.2 O ), imagine the Earth ball, put Siena and Alexandria on one meridian
and ascribe 7.2 O meridian arc distance between cities. Knowing this distance and the fact that a full
circle is 360 O , it is not difficult to find the proportion of the circumference of the Earth's
meridian. Eratosthenes was 46,000 km. Excellent accuracy!

3.

If the satellite will begin to slow down, it will spiral closer to the ground. Hence, it has a circular orbit at
an initial vertical velocity component to appear. If braking is slow enough, then for each new round
circular orbit can be considered, but a smaller radius H. Since circular velocity V = (GM / (R + H)) 1/2 (R radius of the Earth, H - height above the earth, M - mass of the Earth, G - gravitational constant), we see
that V cr increases with decreasing H.
ie with slow growing and normal braking, and the tangential component of the velocity, and hence the
full velocity vector. The growth rate is due to the completion of the kinetic energy of the satellite mv 2 / 2
due to the potential energy mgH.

4.

"Light, and does not heat" - can be said about any source that emits in the optical range at low
power. On the contrary, you can say about the powerful sources of electromagnetic radiation with a peak
in the infrared region of the spectrum. Example - iron.

5.

The solar system is highly flattened. Planets very slightly spaced from the central plane in which they
revolve around the sun, all in one direction. This plane is called the ecliptic. Therefore, the observer on
Earth, and it would seem that the planets and the Sun on the background of distant stars move in the
sky on the same line.

6.

Solution of this problem is not given, because the collector could not learn what it had in mind the
authors.

7.

It is difficult to say whether the names endowed beautiful configuration brightest stars Adam and biblical
patriarchs. Ancient texts mentioning the constellations date from the second half of the second
millennium BC Perhaps the oldest known - Old Babylonian text (1700 BC). This is the prayer to the starry
gods. Referred to the constellation - participants myths (monthly myths, according to the lunar
cycle). Obviously, the first in this regard, attracted the attention of circumpolar constellations, especially
the Big Dipper and the North Star in Ursa Minor, because the rotation of the sky takes about an axis
passing near the North Star. These constellations are the first to receive sustainable titles (based on
mythological), which in most cases are not associated with modern, inspired later periods (Antiquity, the
Middle Ages, the era of geographical discoveries).

8.

Stars are born when compressed gas and dust clouds, with a sufficiently high degree of compression
when warming up in the center is sufficient to "ignite" thermonuclear reactions. Death of a star occurs
when the star is no longer in a stable condition under normal parameters (density, size, somehow
comparable with solar). In this sense, the stars in store for several kinds of deaths, according to their
initial weight.
When M <1,4 solar mass star "dies", becoming the new white dwarf,
with 1,4 <M <3,3 dying corresponds to the birth of a neutron star as remnant of a supernova explosion.
When M> 3,3 death marks the formation of a black hole. All deaths can be observed, and all of them the result of the depletion of nuclear fuel.
Neutron stars and black holes manifest themselves in a particular form of the spectrum of the radiation
and the nature of the impact on normal companion stars.

9.

It will not happen soon. Will also be of interest to energy and mineral resources.

10. Observations suggest that the universe has a honeycomb porous structure. In most large-scale clusters
of galaxies are distributed uniformly in space, but on a smaller scale homogeneity structured
"emptiness" surrounded by "thin wall". And in cotton candy.
Competition for Chemistry
Assignments
1.

When processing of ultrasonic radiation mixture of starch, an aqueous solution of potassium iodide and
carbon tetrachloride observed a blue color.
Suggest an explanation for this phenomenon.

2.

The reaction of the two gases mixed in a molar ratio of 1: 2, in a closed volume leads to the formation of
hydrofluoric acid solution with a mass fraction of 69%.
What is the gases. Answer motivate.

3.

Graphically the dependence of the conductivity of aqueous solutions of sulfuric acid and sodium sulfate
concentration and explain et dependence.

4.

Some alum (hydrated A 1 + B 3+ (SO 4 ) 2 * 12H


hydrogen. Determine their composition.

5.

Alcohols do not conduct electric current while alkalis, also containing a hydroxyl group in a molten state
or in an aqueous solution the electric current is performed.
What is explained?

6.

The inorganic compound contains 12.28% nitrogen, 3.50% hydrogen, 28.07% sulfur and oxygen.
What this compound and its properties are briefly characterize.

O) containing 51.78% oxygen and 4.53%

Tasks
1.

During each ball boy danced a waltz with a girl, or more beautiful than the last dance, or smarter, and at
least one - a woman at the same time more beautiful and more intelligent. Could this be? (Boys and girls
at the ball were equally.)

2.

On the plane, given two circles, one inside the other. Compass and straightedge to construct a point O
such that the outer circle is obtained from the inner tension with center O. ("Tension with center at O"
means the transformation of the plane, leaving the point O and all passing through it right on the spot
and increases (or decreases ) all distances in one and the same number of times.)

3.

In Prostokvashinskoy elementary school student in all 20 children. Do any two of them have a common
grandfather. Prove that one of the grandfathers in this school has at least 14 grandchildren.

Rescheniya competition problems in mathematics


1.

Yes, it can. We describe a ball that meets the requirements of the problem. Let during the first waltz girl
Alla Bella and Val danced with boys Anton, Vadim Borisov, respectively. In this case, Valya - the most
beautiful and the most stupid of the girls, and Bella - the most intelligent and beautiful Alla. Then, if the
next dance Anton will dance with Bella (more beautiful and smarter than Allah), Boris - Valya (which is
beautiful Bella) and Vadim - Alla (which is smarter Wali), the task conditions are met.

2.

First construct a ruler and compass data centers of the circles O 1 and O 2 . This can be done, for
example, a triangle inscribed in a circle and built a middle perpendicular to its sides - and their
intersection point will be the center of the circle.
Draw parallel radii O 1 A 1 and O 2 A 2 . Seeking the point O is the intersection of lines O 1 O 2 and A 1 A 2 . Let us prove this. We
affirm that the circle with center O 1 is obtained from the circle with center O 2tension in the k = | OO 1 | / | OO 2 | times. From the
similarity of triangles OO 1 A 1 and OO 2 A 2 implies that the ratio of the radii of the circles and is equal to k. Take the second circle
point B 2 and draw the radius of the first circle O 1 B 1 parallel O 2 B 2 . Triangles OO 1 B 1 and OO 2 B 2 are similar, then the
angle / B 1 OO 1 equals the angle / B 2 OO 2 , that is, the point O, B 1 , B 2 are collinear and | OB 1 | / | OB 2 | = k.This means that the
point B 1 is obtained from the point B 2 at the center of stretching O and coefficient k.

3.

Every student has no more than two grandfathers. Draw a diagram, where each point is indicated
grandfather, and every student - a line connecting his two grandfathers. According to the problem, any
two lines in this diagram have a common end. This can occur in two cases: either there is a common
grandfather for all students (and then the assertion is true), or there are exactly three of his
grandfather. In this case, there is a pair of grandparents who have no more than 6 general grandchildren,
and other disciples (them at least 14) - third-grandchildren grandfather.

1.

(3 points) at the ball every youth danced a waltz with a girl, or more beautiful than the last dance, or smarter, but the majority (at
least 80 per cent) - with the girl at the same time more beautiful and more intelligent.Could this be? (Boys and girls at the ball were
equally.)

2.

(4 points) Prove that the six edges of the tetrahedron can be folded two triangles.

3.

(4 points) Let a, b, c, d - are real numbers such that a 3 + b 3 + c 3 + d 3 = a + b + c + d = 0. Prove that the sum of any two of these
numbers equal to zero.

4.

(5 points) strip 1 * 10 is divided into unit squares. In squares recorded numbers 1, 2, ..., 10. First, one square any write the number
1, number 2 is then written in one of the adjacent squares, and then the number 3 - one of adjacent already busy, etc. . (Arbitrary
choice are the first choice of square and neighbor at every step). In how many ways this can be done?

Competition in Physics
Tasks
1.

(8kl. And under) in a glass poured sugar solution, and gently poured on top of the water and placed on a
hotplate. The solution begins to boil and simmer for a while, and the water - no, although the difference
in the boiling point of water and the solution is small. How Come?

2.

(8 cells. And under) at the end of the 40s, academician AN Frumkin made a bet with the Americans that
he will get the purest water in the world. One step in producing such it was the evaporation of water and
subsequent condensation of water vapor. Expected results from Frumkin did not, because at reflux went
splashing, spray formed, which was dissolved material installation more active than the liquid
itself.Academician I.V.Petryanov-Sokolov suggested that to achieve evaporation without boiling, you can
move the heater. Prompt and you Academician Frumkin, where it is necessary to move the heater, and
why would not boil.

3.

(All classes) Estimate the total surface area (do not forget the inside) piece of foam rubber.

4.

(All classes) Solid disc and ring roll down an inclined plane. Why do they have at the end of a plane
different speed?

5.

(9kl. And over) The air, for the face, cools it. Air, for the spacecraft during launch and landing, heats
it. What was going on?

6.

(9kl. And older) Why debris thrown from the window of a moving train, strives to immediately fly back
through the adjacent window?

Problem solving competition in physics


1.

In the condition expressly agreed that the difference in boiling points of water and the solution is
low. Thus, the assumption that the water temperature and the same solution and at the boiling
temperature of the solution below (here it boils and before) immediately disappears. Hence, the water
solution and the temperature during the various experiments. Why is this happening? In fact if only the
water glass was, the temperature in the entire volume would odinkovye. Let's see how the heat is
distributed. From the source - electric - it by conduction, the bottom cup is passed, and then the lower
layers of the water and walls of the beaker. The lower layers of water are heated, expands and rises
upwards, where the heavier cold water. That is lowered and heated by the bottom of the beaker. Thus,
the main mode of transmission of heat here - convection.
Even the solution heated to boiling and heavier cold water however is not convection mechanism works! Heat can be transferred
only through thermal conductance between water and a solution of water glass and walls.
Glass wall heat slightly - just below the heat source, and heat conductivity of the glass is not very large. That is why the problem is
mentioned stove, and not the gas burner, as the flame can directly heat the glass wall.It is clear that when he slowly boiling solution
is mixed with water and some time in the glass will boil the diluted solution.

2.

It is necessary to eliminate the boil - that was not boiling liquid bubbles rising from the bottom and spray
in all directions drops - this caustic spray. And why do they rise from the bottom? Because they are
formed there - there just hot, there is a heater. So, if we raise the heater above the fluidized bed is
reduced, and if the heater is positioned above the water surface - not quite boiling. Indeed, as soon as
the top layer will be heated to the boiling point - it evaporates. The lower layers, only the heat receiving
heat transfer through the water will cool. The lower, the cooler and hence denser. And no
premeshivaniya and seething. It is this and advised academician Petryanov-Sokolov. It is certainly
possible, and leave the heater in place, but not to bring water to a boil. She still evaporate. The only
question is when. And even if sufficient exposure to bring eksperementatory experiment until the end is
unknown, is not dissolved during this time (many) more installation material than at reflux.

3.

Piece of foam is much related to each other small air chambers. Total area - is the amount of inner
surface area of the bubble chamber. For simplicity, we will count all these cameras are identical and

estimate the size of a single-chamber bubble. Count how many bubbles account for any amount much
larger than the size of the bubble - 1,2 or 3 cm (depending on your patience when counting
bubbles). Natural to assume that our standard of bubbles at the same length will fit the same. This
means that the partition is selected on the calculated length counting vials, we obtain L - size of our
standard bubble. Now we can calculate that such a piece of foam bubbles with dimensions A, B and C on
each side fit respectively A / L, B / L and C / L of bubbles and their total number of - (A / L) * (B / L ) * (C /
L). The area of the inner surface of the bubble can be calculated in different ways - depending on what
form it has a (hypothetical because it is dependent on us). Naturally, the bubbles have an irregular
spherical shape, but if we assume that the standard vials - cubic, we err slightly (a few tens of%), but to
ease the count. So the surface area of the bubble - 6 * L * L. A total area - 6 * A * B * C / L. (At the
tournament were given pieces of foam with a total surface area of about 1 sq. M.)
4.

The reason is not that the disc is heavier ringlet (could be used to demonstrate the ring, more severe
than the disk), and that the potential energy is the same for mass unit and the disc, and ringlet is
distributed differently. Neglecting friction losses, we can assume that the energy "lander" (disk or ringlet)
at the beginning and end of the inclined plane of the same. But in the first case is the potential energy,
and the second - the amount of kinetic energy and rotation around its own axis. In ringlet fraction
attributable to rotate more kinetic energy and consequently the velocity at the end plane is smaller than
the disk. To understand why this share more fairly easy. It is clear that, at the same angular rotational
velocity, the velocity of the body part that is farther from the center of rotation is greater than the parts
located near the center. And consequently, the rotational energy is also greater. But ringlet whole mass
is concentrated away from the center, while the disc part of the mass is close to the center and takes
little energy "on the rotation." By the way, these considerations it becomes clear why the experiment
were taken disk and ring with approximately equal radii.

5.

When air flows around something friction heats the air, and that is blown, as both dissipates kinetic
energy. But why a person is cooled, how it differs from the spacecraft? There are several reasons. Firstly,
the wind carries the layer of own entity air - air layer, which is close to the surface. Generally, it is more
warm and saturated air with water vapor. His place is cooler and less saturated with water vapor and
cools the face.Second, the sheer face moist and water while maintaining the equilibrium vapor pressure
over the face, and also evaporates cools it. The second reason may lead to the fact that even the wind
with a temperature of 36.6 degrees Celsius to be cooled face. Consequently, we found that when air
blows any subject, there are two factors that contribute to both heat (air friction, positive temperature
difference between the air-subject) and cooling (evaporation from the surface, the negative temperature
difference between the air-thing). And in each case it is necessary to make an assessment of these
factors.

6.

The more air flow rate, the lower the pressure in it - this is a consequence of the Bernoulli
equation. Therefore, in the air stream, breaking into the car through the open windows, the pressure is
less than in the surrounding train more calm air. And if the rate at which waste thrown out of the window
is small, the pressure difference between the air and the calm airflow crosses its back in the car, where
the pressure is less.

Competition in Chemistry
Tasks
1.

In some solution lowered inert electrodes and electric current is passed. Both electrodes began to
differentiate gas density 22. Which hydrogen solution was taken for the experiment?

2.

In the hydrolysis test, 7.42 g of ester obtained 3.22 g of a monobasic carboxylic acid and 6.72 g of a
monohydric alcohol. What ester hydrolysis was taken?

3.

Some alum (hydrated composition A 1 + B 3+ (SO 4 2- ) 2 * 12H


hydrogen. Determine their qualitative composition.

4.

A g of calcium oxide introduced in B ml of water to give a clear solution. Calculate the mass fraction of
substance in the resulting solution.

5.

Over hot copper oxide missed excess hydrogen. The sample is then cooled and weighed. How many
times has changed its mass?

6.

Suggest a way to color the ball ping-pong table inside (then the ball must be suitable for the game).

O) containing 51.76% oxygen and 4.53%

Problem solving competition in Chemistry


1.

If not involve complex solutions embodiments, it is obvious that both electrodes releasing carbon
dioxide, and chemical reactions is rather difficult to explain. Well, if you remember that the electrodes
during electrolysis is also very hot, as a variant solution emerges, for example, seltzer (salted soda) boiling soda, as we all know to be pushed into oblivion machines with water results in the release of gas.
As for the non-trivial solutions to the tournament there were none, and are interested can reflect what happens in the electrolysis
solution NO 2 (or more precisely, N 2 O 4 ) in concentrated nitric acid - not whether there will be the same effect; satisfied, can write
the equation of electrolysis NO 2 BF 4 in some conducting an inert solvent. After a problem is not asserted that the water solution!

2.

Task to verify the inertia of thinking. Who entered a decisive straightforward - write the equation
RCOOR '+ H 2 O = RCOOH + R'OH, and he received an absurd result. A careful reading of the problem
leads to doubts - because it does not say that it is an ester, and even the one that is described in school
textbooks. Scrabble (Read as much as 2 books in addition to the textbook) once (or poskripev brains) will
remember that in addition to ordinary esters, there are just as common orthoesters - when 1 acid residue
has to balance the alcohol 3 - RC (OR ') 3 . Witted physicist (It is generally considered that the chemistry the branch of physics, and it can be fully described on the basis of general physical considerations) will
begin to vary a single parameter - the number of water molecules (which, we note, at a rate of 1 to ether
may be a fraction), and will also be right.
If we substitute the expression referred to the 2 H 2 O, to alcohol have also put 3, and then the equation looks like this:
RC (OR ')
M

(x) 3M +

R'

(3Y) +60

2H

--->

RCOOH

36

3R'OH

x + 45

3y + 51

3.22

6.72

0.07 mol

0.21 mol

7.42
0.14 mol

We hope that a solution to this proportion did not take long at any of the aforementioned scholar nor a physicist, and they will find
that M R acid - 46, alcohol - 32, and saponified orthoester of formic acid - HC (OMe) 3 .
3.

Again - the logic does not work honors. The calculation for ordinary metals A and B gives the answer, not
provided for periodic system:
the mass fraction of hydrogen

24 / (a + b + 192 + 216) = 0.0453

a + b = 121,8

the mass fraction of oxygen

320 / (a + b + 192 + 216) = 0.5176

a + b = 210,3

4.

(Atomic mass A and B taken as a and b, respectively). And it does not happen. Hence, not only the hydrogen contained in water of
crystallisation.

5.

It is assumed that one of the "metals" - not metal, but something containing either oxygen or hydrogen (in this case rectilinear
invalid payment). Scrabble recalls that happens also ammonium, behaves quite "metallic", a physicist calculates it
logically. Therefore, ammonium alum. If a = 18 (ammonium), then b = 210-18 = 192 - is iridium Ir. Formula alum NH 4 Ir (SO 4 ) 2 * 12H 2 O. Well, those who are afraid of words that are not in the school curriculum, it would be better not to go to
the Olympics (especially since there is nothing new in the properties of iridium with respect to aluminum casting is not required).

6.

We form the equation for the reaction:


CaO + H 2 O = Ca (OH) 2 ;
56 g CaO react with water to form a 74 g Ca (OH) 2 ,
CaO A g react with water to form (74/56) A g Ca (OH) 2
Thus, in the solution will contain 1.32 g A Ca ( OH) 2 Since the weight of the solution (on the basis of the reaction equation) is equal
to the sum of the weight of the starting materials and (A + B) r, starting material is completely soluble (no precipitate) have
= 132 * A / (A + B) (%).

7.

We form the equation for the reaction:


CuO + H 2 = Cu + H 2 O;
1 mol of CuO - 1 mol Cu, 80 g / mol - 64 g / mol, 80 g of CuO give 64 g of Cu.

Hence, the mass of the sample taken after passing over it hydrogen decreases in 80/64 = 1.25 times.
8.

Methods color celluloid based on his restoration various nitrogen-containing compounds. On this and
many other interesting things in organic synthesis is well written in the book Shpaustsusa "Journey into
the world of organic chemistry."

Tasks
1.

(7-11) on the plane is a square, and invisible ink applied point. Man in special glasses sees the point. If
you draw a straight line, he said, is whether a point on the line, and if not lies, he says, on which side of
this line is invisible point. What is the minimum number of lines necessary to carry out, to see whether
the invisible point lies inside a square (not outside and not on the border)?

2.

(7-9) Is there a 100 natural numbers such that their sum is equal to their least common multiple?

3.

(7-9) The rectangle ABCD (AB = a, BC = b) It so happened that turned the pentagon area S (C laid down
in A). Prove that S < 3 / 4 ab.

4.

(10-11) M A , M b , M C - midpoints of the sides, H A , H b , H C - base heights of triangle ABC area S. Prove
that the segments M A H b , M b H C , M C H A triangle can be formed, find its area.

5.

(10-11) a) there are four different natural numbers such that the sum of any three of them is a prime
number?
b) Are there five of these numbers?

Problem solving in mathematics competition


1.

(7-11) Answer: a necessary and sufficient number of questions - three.


Sufficiency. Let our square - ABCD. In the first step we perform a direct AC. Man in special glasses (for brevity - "man") indicates
one of the half-planes or very straight. If it is - the half-plane containing the vertex B, then the second and third step of conducting
lines AB and CB. The same straight wire and if it points to the very direct AC. If the second and third step a person indicates both
times the half-plane, which contains the triangle ABC, invisible point lies in the interior of the square; if once he points to the halfplane, and the second time - on the line, or both times - on a straight line, the point lies on the boundary of a square. If at least one
time, he points to the half-plane, in which there is a triangle ABC, then the point is outside the square. Case when a man pointing to
the half-plane containing the point D, similar.
Necessity. If you specify only one or two questions, it held less than three lines (two or one). Whatever the answers, we can learn
only if a point belongs to those parts into which the plane smashed the drawn line. But these parts are unbounded, and belonging to
them can not be proof that the point belongs to the square.

2.

(7-9) These numbers exist. Here is one example:


1, 1, ..., 2, 2, ..., 4, 5, 6 (89 units, eight pairs). The sum of these numbers ( ) is equal to 120, and the
least common multiple (LCM) - 120 too.
The problem can be understood, and so that all the numbers must be different. Here is an example of such a kit:
1, 3, 4, 8, 16, ..., 2 * 3 98 2 99 (successive powers of two, but in the second place is the 3 and 99-m - 3 2 * 98 ). For these numbers
NOC = = 3 * 2 99 .

3.

(7-9) Draw a rectangle on the line l, for which it is laid. To coincide with the point C to point A, you need
to l passes through the middle of the diagonal AC and it was perpendicular. It is known that turned
pentagon, therefore the rectangle - not square. Assume for definiteness that AB <CD. Then l intersects
sides BC (at point M) and DA (point N). AMN = CMN, since they coincide when applied. Hence, CM = MA,
NA = CN, /= MCA / MAC, / = NCA / NAC. However, / = DAC / BCA, since AD || BC. Consequently, CMA and
CNA - equal to each other isosceles triangles. Hence, MA = ND, / = CND / AMB and hence, CND = AMB.
We divide our quadrilateral into four triangles: ABM, AMN, CMN and CND. Area of the first and fourth of which are equal and the
first and second areas are also equal, with the S ABM <S AMN (they have a common height AB, AN said base triangle AMN BM more
base triangle ABM). Hence, the area of triangle AMN is greater than a quarter of the area of the quadrangle.
Pentagon, which received after the quadrilateral is complex, composed of triangles ABM, CND and AMN (or CMN). We can see
that from the quadrangle threw a piece that area more than a quarter of the quadrangle. This means that the remainder is less than
3/4.

4.

(10-11) Triangle BCH b - a rectangular, the point M A - the middle of its hypotenuse, which is known to
coincide with the center of the triangle described BCH b circle. Therefore, | M A H b | = | M A B | = 1 / 2 BC.
Analogous considerations with respect to two other of said segments in a condition indicates that
| M b H C | = 1 / 2 AC, | M C H A | = 1 / 2 AB. Thus, the segments of the triangle can be formed, which is half the side of the original
triangle sides. Such a triangle similar to the original scaling coefficient 1 / 2 , and its area is S / 4.

5.

(10-11) a) The answer is - there. Here is an example of a four numbers: 1, 3, 7.9. From these numbers we
can form four triples; their sum - 11, 13, 17 and 19 - the number of ordinary.
b) A - not exist. Integer when divided by three can yield one of three residues: 0, 1, 2. The amount of the three integers that give the
same remainder when divided by three, itself divided into three. If there are three integers such that their residues when divided by
three are all different, then the sum of these numbers is also divided into three. Assume now that there is a set of N distinct positive
integers such that the sum of any three of them - a prime number. First, we note that this amount is never less than 6, since the
number of all positive and distinct, so that the amount was a prime number, it should not be divided by three. From this it follows
that when divided among residues at all three sets of numbers may not meet all three options (shown above), and each of the other
two embodiments can be represented by not more than two numbers. It follows that N< 4.

Competition for mathematical games


Terms Games
1.

The two take turns writing the numbers, starting with a non-zero, with the most significant bit (in order
until the youngest). - 6 digits. If the number is divided evenly divisible by 7, then made the last move
wins, otherwise - beginner. Who wins the game with the right?

2.

The two take turns writing the numbers, starting with a non-zero, with the most significant bit (in order
until the youngest). Total - k digits. If the number is divided evenly divisible by 11, then made the last
move wins, otherwise - beginner. Who wins the game with the right? Consider the following cases:
a) k - even; b) k - is odd, one more.

3.

The two take turns putting their horses on a chess board. You can not put a horse to the beat at least one
of the previously supplied. Who can not put the horse loses. Who wins the game with the right?

4.

Each turn of the sweets on the table can take any number strictly less than half, or even a piece of
candy. The player who took the last piece of candy. Who will win the game at the right if the table - 50
chocolates.Note: better to deal with 2, 3, 4, ... candy.

5.

There are a couple of apples and 2 people. They take an apple and begin to eat at the same time. Each
iozhet take the following apple only after previous ate. Speed of eating (in grams per second) is the
same for both players. What Apple should choose first came up to the table to eat more in the following
cases:
a) 3 apples: 300 g, 400 g, 500 g,
b) 300, 500, 700, 900;
c) 300, 600, 700, 800;
g) 10, 20, 21, 30, 31,
etc.) 100, 201, 202, 203
e) 201, 299, 400, 600, 900;
g) a, b, c, d, where a <b <c <d and d <a + b.

6.

B cells and rectangle) k on k; b) 2 k in turn put a spotlight illuminating all cells located to the left and not
above. In this case, each move is necessary to cover at least one new cell. Who put the spotlight in the
most upper left corner, loses. Who can secure a win, and how it should play?

7.

Thun and philosophers began to dispute Bol, pulling each one thesis. Next to each queue can either
repeat, without a permit, absolutely all of its previously spoken phrases or repeat just heard it a friend or
add to it one of his sentence. The player who was first in his next speech will deliver more than 9
consecutive sentences. Who argue?

Competition in Physics
Tasks
(In parentheses are the classes for which the recommended task, but counted all solved problems.)

1.

(7-9) Recently, the media has passed the message on the giant iceberg, breaking away from the
Antarctic ice sheet. The iceberg about 100 km in length, a width of 10 km and a 1 km
thick. Environmentalists are sounding the alarm. As far as sea level rise, when the iceberg melts?

2.

(7-9) The famous inventor of perpetual motion Present completes assembly of the latest model. As you
know, the capillaries in the trees sap can rise very high. Take timber, immersed in water, the water
rises. At the top of the water on the specific cut side (remember to collect birch sap) enters the grooves
and from there flows to the mill wheels, causing them to move. Why is this engine still does not work?

3.

(7-11) What time of day is shown in the picture Vrubel's "Pan"? Determine the possible Acc.

4.

(10-11) Celebrating a successful landing on the moon, lunar rover crew, right in the cockpit, where they
were without spacesuits, opened a bottle of "Borjomi". Different than the behavior of bubbles in this
bottle of bubbles in a bottle, but open on the Earth?

5.

(10-11) The Nobel Prize in Physics for 1993 was awarded to Dzh.Teyloru and R.Halsu for the discovery in
1974 of the first binary pulsar. They found that the interval between the pulses from the pulsar (rotating
around the axis of the neutron star, which has a radiating "hot spot", dissenting from the pole, as soon
as the beam from the "hot spot" fell to the ground, fixed pulse) periodically varies little and explained it
the fact that the emitted pulse neutron star rotates around another neutron star. Explain how it is
changing the interval between pulses as reference stars relative to each other and how we can estimate
the radius of the orbit of the rotation.

6.

Evaluate how much temperature may rise in the audience of your stay in it.

Problem solving competition in physics


In the solutions proposed by the students, the jury valued above all the ability to see the main features of the
phenomenon and neglected secondary. What is the main and secondary that is initially determined intuitively. To
confirm or not confirm the accuracy of intuitive assumptions, requires a closer look at the phenomenon. On
closer examination often requires new intuitive assumptions, and so, strictly speaking, to infinity. Almost have to
stop at the moment when interest in the task at hand is satisfied. Thus, the requirements to address the student
are not strictly defined, and plays a significant role match or mismatch of flavors and traditions of students and
members of the jury. Therefore, estimates of the solutions are not strictly objective. However, this is not
important. In the selection of tasks, we tried to give food for thought, which would help to better understand and
feel the real world around us, and it would be useful to students regardless of what kind of activity they choose
for themselves in the future.
1.

Messages in the media referred to in the problem, actually took place. We do not call the newspaper in
which they were published, and the authors did not specify (though they could, but why?). Note that
"environmentalists" are mentioned in the problem, it is "environmentalists" in quotes, that is often poorly
educated in the sciences representatives of social and political movements. I write in order apology to
these environmentalists (without quotes) - specialists in a particular and very interesting biological
discipline.
In order apology to students and teachers, I note that the proposed problem is as old as the world. In many books of problems it is
offered in the form: "In a glass of water floating piece of ice ..." and so on, obviously continued. The language is clear, are
equivalent, but there is a solution that is more natural to look for a glass.

In response to this problem - the level will not change. This is easily obtained from the law of Archimedes. Weight equal to the
weight of displaced water ice floes. As a result, the weight of the ice melts it will not change, and the melted water will take the
same amount, which is engaged in the underwater part of the iceberg.
Another solution I formulate for the glass. Assume for simplicity that the glass - a cylindrical. Put a glass of water and pieces of ice
on the scales. They will show the total weight of these items. It is understood that when the ice melts, the weight does not
change. By weight water with floating objects therein equal to the pressure at the bottom of a glass of water multiplied by the area
of the bottom. The pressure is determined by the height of the water column. Once the pressure has not changed, then the height of
the column has not changed.
This is - the solution to a first approximation. Specify that it must be assumed that this decision was correct. We hypothesized that
a) the presence of air above the water is negligibly;
b) the temperature of the water ice and is identical and equal to 0 degrees,
c) the clean water and ice.
Detailed consideration of the problem, when these conditions are not met, leading to a very minor clarifications answer: it turns out
that the real values of air pressure, temperature and salinity of the water level is almost unchanged.
Asks students to evaluate the impact of these factors. Can you think of any more factors that can change the answer?
2.

If sap in trees raised only under the influence of capillary forces, then, first of all, he raised would be very
low, and secondly, he never would have poured from the upper end of the capillary (remember, as a
force lifting the liquid in the capillary vessels - for the occurrence of lift necessary to continued above the
meniscus of the vessel wall). In a living tree sap rises under the influence of several mechanisms. One osmotic pressure that occurs due to high concentration of salts inside the tree, while the concentration
of salts in the soil are significantly lower. This mechanism can lead to pouring the juice out of the vessel,
but it does not allow to create a perpetual motion, as the concentration of salts must somehow
maintain. In the live tree, it is supported by the biochemical reaction. Other mechanisms for lifting juices
associated with the presence of special organs located on a tree trunk. These bodies also work only on
living trees.
However, the Present, the test has not yet finished his car, so that the last word has not yet been told.

3.

Time of day can be determined by knowing the position and shape of the moon. It is known that the
moon revolves around the Earth, which, in turn, orbits around the Sun, as well as on its axis (Fig.
1). Figure 1. the plane of rotation of the Earth around the Sun and the Moon around the Earth are
practically the same. The plane of rotation of the Earth around the Sun and around its axis does not
coincide (the angle between 23 o 27 '). Direction of rotation is the same for all three. The sun always
shines half moon, addressed to him (except in the case of a lunar eclipse).

Watching the Moon from the Earth, we see only a piece of the illuminated portion (sickle). The thickness of the crescent depends on
the relative positions of the Moon, the Earth and the Sun as:
D = R (1 - cos ) (see. Fig. 2). Figure 2. In the month in the picture Vrubel d = R / 2, where we find that = 60 O .

It is obvious that Vrubel on his picture painted landscape middle latitudes of the northern hemisphere.
At a known angle (that is, at a certain shape of the moon), there are two positions of the Moon, indicated in Figure 1 by Roman
numerals I and II. To find out which of them is the case, we note that (when viewed from the northern hemisphere) in position I
ends sickle necessarily directed to the right (waning moon) in position II - the left (waxing Moon). Since the sickle in the picture is
on the right, the moon is in position I.
Figure 3 Figure 3. paper plane coincides with the plane containing the centers of the Sun, Moon and Earth, but does not coincide
with the equatorial plane of the Earth. Inside the circle depicting the projection of the Earth on the plane of the drawing, drawn by a
dotted circle, which is the north pole of the Earth. At what point of the circle it is, depends on the time of year.

Further, the moon is on the horizon, that is the direction it touches the surface of the Earth at the point of observation. Hence, the
effect of the picture occurs somewhere on the line AB is a drawing of a circle and dotted on the north side of the earth or from the
point A or point B side (segments AA 'and BB', respectively). If an action film takes place on the segment BB ', the axis crescent
(the line connecting the ends) should be rotated clockwise through an angle; if the segment AA ', this axis is rotated
counterclockwise (see. Fig. 4). Figure 4. In the picture it is turned counterclockwise. Therefore, the painting depicts a place that lies
on the segment AA '(segment BB' will not do as well because there - day and therefore the moon can not be seen clearly enough).

It is easy to calculate what would be the time, if the north pole coincides with the point O (that is, the equator would be in the plane
of the figure 3). At point D - 12 pm local time, at the point M - 0 hours. Earth rotates counterclockwise. Angle AOM = 90 O - =
30 O . Local time is linearly dependent on the angle. Therefore, A is 12 * 30/180 = 2:00.
The discrepancy between the planes of rotation of the Earth around the Sun and around its axis leads to a deviation from this value
response. Summer and spring time is reduced, winter and fall - increased by an hour and a half. Than the north area, the greater the
discrepancy. These estimates can be made by considering Figure 3 and the various options for the location of the North Pole on the
dotted circle.
Determine the time with greater precision is not possible, solely on the basis of the data of the problem.
4.

Cabin pressure was atmospheric, but the weight of subjects was less than on the earth. This means that
the surface of the water in the open bottle was atmospheric pressure, and at the bottom of the bottle less than what it would be in the world at that point. Therefore, on the Moon bubbles in the water column
will be allocated more rapidly. At the same time they will rise slowly, as the buoyancy force will be less
than on Earth. As a result, the water in the bottle more froth than the Earth.

5.

Of two neutron stars, we are interested in that which radiates. It moves around a common center of
mass of the two stars orbit for some; For simplicity, assume that the orbit - circular. We also assume
initially that the observer is located in the plane of rotation and the distance from the rotation center to
the point of observation (to ground) is constant and equal D. The linear velocity of the star denote V, the
time between successive pulses - through d.
Let A be the point of the orbit, in which the star is moving at maximum speed on us; it happens when the angular distance of the star
from the center of rotation as possible (from the point of view of the observer);approach speeds star to the observer at this point is
equal to V. Consider the momentum that rejected the star at point A (the time it will take for radiation origin of time), and the next
pulse emitted by the time d. The time during which the first pulse has reached the observer is D / c (c - velocity of light). The time
during which the next pulse is reached observer smaller as the radiation source is closer to the viewer by an amount Vd.This time is
equal to (D-Vd) / c. Thus, the pulses recorded by the observer at point: D / c, and d + (D-Vd) / c. The difference between these
moments is the observed time between pulses D - . It is equal to D - = D + (D-Vd) / C - D / C = D (1-V / c).
Consider the point in the orbit in which the star with a top speed of moving away from the observer. A similar calculation shows
that the observed time between pulses in the position of the star D + D = (1 + V / c).
The average value of D + and D - is equal to (D + + D - ) / 2 = d. D + and D - - observable quantities, thus the value of d can be
considered known. D + - V = D / C; thus the value V can be computed. T treatment period stars around a common center of rotation
of the two stars, we also know, as it is - observed value. The length of the orbit is equal to s = VT. At the same time, 2 s
= R. Hence we find R.
Note that examined the effect of varying the frequency of the radiation when the source moves relative to the observer has a wellstudied and analyzed in the textbooks of the Doppler effect.

Thus, some additional assumptions, the problem is solved. These two assumptions: the fixed stars with respect to the observation
point (or more precisely - the constancy of the distance to it), and the location of the observer in the plane of rotation of the star. If
we reject these assumptions, the results of the calculation will change.
Let the star, for example, is approaching us with speed v. Then, the same reasoning we find that the observed interval between
successive pulses at the points A and B respectively is d (1- (v + V) / c) and d (1- (vV) / c), and the average value of these intervals
equal to d (1-v / c). The same happens when a double star is moving away from us (in the formula v is negative). If the speed is
closer or farther from the double star observer somehow found, it is possible to find the value of d. If it is unknown, it is impossible
to find d.
Now drop the second assumption. Let the angle between the segment joining the point of observation and common center of a
double star, and the axis of rotation of the star radiating from the center of rotation is different from direct and equal to . Then in
our calculation V is replaced by
V cos . The rest of the payment will continue. But if is unknown, to bring the calculation to a numerical result is
impossible. We note only that cos is never greater than 1, so we get to the radius of rotation of the lower bound. If you do not
know the angle , we will take it for / 2, we obtain the radius of rotation, which can be more true, but no less.
6.

Discard humorous responses like this, which begins with the words: "As the State Academy of Oil and
Gas does not drown, I'm frozen and not able to generate heat ...".
In this problem, the main thing - to find a hook that allows to obtain at least inaccurate, but the numerical answer. Many students
noticed that the main mechanism of heat is breathing. Rate heat through breathing is relatively easy. Let us assume that a person
makes in a calm state of 20 breaths per minute, each time inhaling 1 liter of air. This means that he exhales hour 20 * 60 * 1 = 1200
liters of air. The temperature of the exhaled air is almost equal to the body temperature, i.e. 36 O C. Assuming for simplicity that the
air temperature in the audience is 16 O , we find that 1200 liters of air heated to 20 O . If the audience has a volume of 1,000 cubic
meters (each student can on the eye to estimate the size of its audience), we find that 1/800 of the air heated to 20 O , so if all the air
mix, its temperature will rise by 20/800 = 1 / 40 degrees.
Let us now discuss how legitimately assume that the breath - the basic mechanism of heat transfer. Most likely - this assumption is
still wrong. If heat loss from the body surface can be neglected, then there was no need to dress warmly. But can we assume that the
heat transfer through the surface of the body - the main mechanism of heat transfer body, and breath can be ignored? Recall how
behaved dogs in hot weather. Even those who have very warm fur, can achieve for themselves a normal temperature. To do this,
they often breathe, his tongue hanging out. This example shows that breathing in some cases may be substantially and almost the
only means of cooling the body. It can be concluded that for a person both mechanisms likely give the same order of magnitude
effect.
So, the answer to the problem of 1/20 of a degree (for the received size of the audience).

Competition in Chemistry
Tasks
1.

Chemical analysis of the nitrogen compounds with hydrogen, having a relative weight of 32 revealed that
the mass fraction of nitrogen in the compound is 66%. Prove that the test results are incorrect.

2.

In a closed volume placed two identical glasses at 25 to C with distilled water and sulfuric acid solution
(mass fraction of 80%). What can be said about the temperature of each of the glasses after a while?

3.

When processing the mixture ultrasvetovym radiation starch aqueous solution of potassium iodide and
carbon tetrachloride appearance of a blue color is observed. Suggest an explanation for this
phenomenon.

4.

Are the two solutions of salts formed of the same acid change violet litmus, one blue and one - red? Give
a reasoned response and discuss options with concrete examples.

5.

Is it possible, with heavy water, get LiAlD 4 and (ND 4 ) 2 SO


response. Try to get the products with the highest yield.

6.

An equimolar mixture of nitrogen and oxygen in a closed vessel subjected to prolonged exposure to
electric discharge, after which the reaction products are brought to normal conditions and under vigorous
stirring to an excess of sodium carbonate; wherein the solution after mixing (solution A) nitrate ions were
not detected. The volume of gas remaining after the operation, was 5/8 the volume of the feed mixture
(measured under the same conditions).
1) In any form of nitrogen compounds in the solution is A? Write the necessary equation reaRktsy.

* FeSO

* 6H

O? Give an appropriate

2) Determine, if possible, some of the nitrogen react when exposed to the discharge. Solubility of gases
in water ignored.
7.

You must discover the "chemtrails" on combat positions submarine, still lying at a depth of 60 m. Boat
jettisons 2 kg / h of waste lubricants and 10 m 3 / h of hydrogen are produced simultaneously with the
crew used for breathing oxygen. As a result, corrosion of the boat losing 0.1 kg / h of iron as Fe (II) and
0.5 kg / h is used for protection against corrosion of zinc. What analytical tools and equipment you see fit
to establish anti-submarine ship, whose task is the discovery of the submarine on its "chemtrails"?

Problem solving competition in Chemistry


1.

It is known that the number of atoms in general, and in the molecule, in particular - an integer (the word
"atom" in Greek means "indivisible").
According to the analysis
N (N) = 0.66 * 32 * 1 / 14 is approximately 1.5
The number of nitrogen atoms can be fractional, therefore, incorrect analysis.

2.

A glass of water will have a temperature lower than the glass with a solution of sulfuric acid. Glass with
H 2 SO 4 , which is a strong hygroscopic substance, ie absorbs water will be heated by absorption vapor
H 2 O (that vaporize and then diffuse out of the glass with H 2 O).
Glass with H 2 O will be slightly cooled by the evaporation of water.

3.

Let's write a series of chemical equations to explain this phenomenon:


KJ -> H K + + I 2I - -> I 2
CCl 4 + K + -> KCl + CCl 3 * (free radical)
2CCl 3 * -> C 2 Cl 6
Starch staining occurs under the action of free J 2

4.

Possible options. 2 cases are fundamentally different.


1) KH 2 PO 4 and K 3 PO 4
H 2 PO 4 - = H + + HPO 4 2- , -PH <7
PO 4 3- + H 2 O = OH - + HPO 4 2- , -PH> 7
2) KCH 3 COO and Al (CH 3 COO) 3
KCH 3 COO: CH 3 COO - + H 2 O = OH - + CH 3 COOH, -pH> 7
Al (CH 3 COO) 3 : Al 3+ + H 2 O = H + + Al (OH) 2+ , pH <7

5.

Get LiAlD 4 can. We present the corresponding reaction:


D 2 O -> D 2 + 1 / 2 O 2 (electrolysis)
D 2 + 2Li -> 2LiD
4LiD + AlCl 3 -> LiAlD 4 + 3LiCl
Get (ND 4 ) 2 SO 4 * FeSO 4 * 6H 2 O is not possible. The reason is that Mohr's salt can receive co-crystallization of ammonium
sulfate and iron. As before ion ND 4 + will be in solution in the presence of H 2 O (which, by assumption, enters the crystallization,
but water), then exchange will occur
ND 4 + + H 2 O = ND 3 H + + HDO.

6.

Possible reaction
N 2 + O 2 -> 2NO
2NO + O 2 -> 2NO

In the absorption solution CO 3 2- :


1) H 2 O + NO + NO 2 + 2CO 3 2- -> 2HCO 3 - + 2NO 2 2) 4NO 2 + O 2 + 2H 2 O + 4CO 3 2- -> 4HCO 3 - + 4NO 3 -

Case 2) is impossible. Hence, the gas mixture before absorption - NO, NO 2 and, possibly, N 2 . After absorption - N 2 + NO.
Suppose, in terms of 1 mole of a mixture (0.5 mole of N 2 O and 0.5 mol 2 ) to get the x mol NO 2 .
We have:
Number N: 2x + y + 2z = 1
Number O: 3x + y = 1
The volume ratio of y + z = 5/8
Hence, y = 11 / 20 , x = 3 / 20 , z = 3 / 40 .
Reacted 1 / 2 - 3 / 40 = 7 / 40 mol N 2 of 0.5. Hence, reacted 70% N 2 .
7.

You can use any reasonable method of analytical determination of H 2 , Zn 2+ and Fe 2+ . For H 2 determination of redox-potential of water for Zn 2+ sulfide method for Fe 2+ - [Fe (salt)] 3- . The most
promising method is based on detecting the presence of lubricants, for example, using adsorbents or
analyzing a surface layer of water on the presence of organic impurities.

Competition in Mathematics
Tasks
Grades 7-9
1.

From the hot tub faucet filled in 23 minutes, from a cold - 17 minutes. Mary opened the first hot tap. How
many minutes it should open the cold, so that when filling the bath with hot water nalilos 1.5 times more
than the cold?

2.

The length of the height of the rectangular trapezoid ABCD AB equal to the sum of the lengths of the
bases AD and BC. In what respect bisector of angle ABC divides side CD.

3.

At each kilometer of highway between the villages of Elkin and Palkino should post a sign on one side of
which it is written, how many kilometers to Elkin, on the other - to Palkino. Boris noticed that each
column is equal to the sum of all digits of 13. What is the distance from Elkin to Palkino?

10-11
1.

Any more numbers among all the numbers from 100 to 999: those in which the average number of both
more extreme, or those in which the average figure is less than both the extreme?

2.

Circle held several (finitely many) different chords so that each of them passes through the middle of any
other of the studies chords. Prove that all these chords is the diameter of the circle.

3.

Is there a convex polyhedron has 12 ribs, which are, respectively, 12 and parallel to the diagonals of the
faces of a cube?

Additional tasks (for all classes)


1.

Find the sum of angles of MAN, MBN, MCN, MDN and MEN, drawn on graph paper as shown.

2.
3.
4.
5.
6.
7.
8.
9.

*
|
*
|
*
|
*

-ABCDE
| | | |
- * - *
| | | |
- * - *
| | | |
- * - *

|
- * - * - *
|
- * - * - *
|
- * - * - *

10. | | | | | |
11. * - * - * - * - * - *
12. | | | | | |
13. MN - * - * - * - *
14.

15. Dan paper circle. Can I use the scissors to cut it into several parts that make up the square of the same
area? (Cut permitted under direct and arcs).
16. In the bazaar selling fish, big and small. Today, three large and one small stand together as much as
three large and one small yesterday. Is it possible to find out from these data that the more expensive:
one large and two small today, or five little yesterday.
17. Peter cut a rectangular piece of paper in a straight line. Then he cut in a straight line from one of the
resulting pieces. Then he did the same with one of the three pieces of the resulting etc. Prove that, after
a sufficient number of cuts can be selected among the 100 pieces of the resulting polygons with the
same number of vertices (e.g., triangles 100 or quadrangles 100, etc.).
Problem solving in mathematics competition
Grades 7-9
1. To the hot water in the bath turned out 1.5 times more than the cold, cold tap must fill 2/5 baths and cold - 3/5 (to find out, it was possible
to identify the volume of the bathroom for 1, the amount of cold water for x; if x + 1,5x = 1, where x = 2/5). But if the hot tap must be open
all (2/3) * 23 = 69/5 minutes, and the cold - for (2/5) * 17 = 34/5 minutes. Hence, the need to open the cold tap over (69/5) - (34/5) = 35/5 = 7
minutes.
2. We prove that the bisector of angle ABC chord divides DC half.
Let O - midpoint of the side DC. Draw through the points A and O line, let M - the point of intersection of this line with the line BC. Then
triangles AOD and MOC will be equal to the second feature (DO = OC by hypothesis, / AOD = / COM as vertical; / = ADO / OCM, as the
lines AD and BC are parallel). This means that AO = OM, t. E. The triangle ABM - isosceles. But bisector at the vertex of an isosceles
triangle is the median and the height from which the bisector of angle ABC passes through the middle of AM, that is, the point O, and then
divides DC into equal parts (ie. A. O - mid-DC).

3. Let the distance from Elkin to Palkino n kilometers (by hypothesis, n-integer). We number the columns from Elkin to Palkino
order. Consider the 9th column, t. E. A pillar, spaced from Elkin 9 kilometers (clearly, n >10). Then, one side is written 9, and on the other: n9. The next column is written on one side 10 and the other: n-10. If n does not end at 9, then the n-9 would not have ended with at 0, and
hence the amount of digits of n-9 and n-10 would be equal, but then on the 9th column sum of the digits would be 8 more than on the 10th,
which is impossible. Hence, n ends at 9.
If n> 49, then the sum of the digits in the 49th column is greater 13. This means that for n is the only such features: 19, 29, 39, 49.

If n = 19, then on the 9th column sum of the digits is equal to 9 + 1 + 0 = 10 - a contradiction.
If n = 29, then on the 9th column sum of the digits is equal to 9 + 2 + 0 = 11 - a contradiction.
If n = 39, then on the 9th column sum of the digits is equal to 9 + 3 + 0 = 12 - a contradiction.
There is only one possibility: n = 49. It is easy to check that in this case, the sum of all columns of numbers will be equal to 13 (it is sufficient
to check only for 9, 19, 29, 39, and 49 poles, think why).
10-11
1. Note that if a number greater than the average number of both extremes, then the number of 999-a figure less than the average of both
extremes. Therefore, some of the numbers from 100 to 999-100 = 899 the same amount of numbers with the highest average figure and the
lowest average number (which can be divided into pairs). But there are still a number from 900 to 999. It is clear that none of them are
numbers with the greatest average number, but there is the lowest, such as 901. Therefore, greater numbers of those who have less than the
average number of both extremes.
2. Note that the smaller the distance from the center O to the circumference of the chord, the greater the chord length. Since the final number
of chords, among them is the smallest in length, for example, AB. By hypothesis, it passes through the midpoint of chord K of some other,
say, CD. If the point of intersection of AB and CD is not well and the middle of AB, the distance from point O to CD will obviously be greater
than the distance from O to AB (t. To. OK will be greater than the length of the perpendicular from the point O to AB) therefore, the chord has
a smaller CD than AB, length - contradiction. So, CD goes through the middle of AB, where the perpendicular from the point O on these
chords are the same. This is possible only if the AB and CD are the same, or if the AB and CD intersect at the center. The first is impossible,
then AB and CD - diameters.Likewise, it is proved that all the other chords pass through O.

3. The answer is: there is.


Example. Construct a cube whose edges are twice more than the initial, middle note and its faces. It is clear that one can construct an
octahedron with vertices at the marked points, with all its edges are equal halves of the diagonals of the cube faces constructed. Thus, this is
the desired octahedron.
Additional tasks (for all classes)
1. Note that / = MAN / RES, / = MBN / QER, / MCN = / PEQ, / MDN = / NEP. Hence we see immediately that / MAN + / + MBN / MCN
+ / + MDN / MEN = / MES = 45 o

2. Let us assume that this can be done. Then the edge of each piece is made up of lines and arcs of circles. For each arc define its "angular
measure", equal to the ratio of length to the radius of the arc, with taken with a positive sign if the radius vector drawn from the center of the
circle, sticks "inside" our paper circles and negative if it sticks in outside.

Now we assign each piece of the amount of angular measures its arcs. It is easy to see that if we "sostykum" together two pieces A and B, the
sum of the angles measures piece A + B is equal to (the sum of the angular piece measures A) + (sum of the angles measures piece B). This is
because when two arcs are glued, the amount of angular steps equal to 0.
Hence we see that the sum of the angles measures of the arcs on the circle should ovpadat with the sum of the angular measures of the arcs on
the boundary of the square. But this is not the case for the circle is equal to the number 2 , and for the square - 0.
3. Let 'fish prices ": today the big fish worth b C , and small m C . Yesterday most cost b v , and the little - m v . Then from the conditions of the
problem we have two equations
3b C m + C = 5b v , 2b C m + C 3b = v m + v .
Hence, we obtain:
5m v = (2b C m + C -3b v ) 10b 5 = C + 5m C -3 (3b C m + C ) = b C 2m + C .
That is five little yesterday cost as much as one large and two small today.
4. Note that when Petya dissections obtained only convex polygons. In addition, we note that the cutting convex n-gon in a straight line, we
get da new polygon with n 1 and n 2 and n vertices, respectively 1 + n 2 < n + 4 (this follows from the fact that in the reverse bonding convex
n 1 and n 2 -gons the "gap" may be no more than 4 vertices). Hence, after Peter m-1 will cut all m obtain polygons with the total number of
vertices less than 4 + 4 (m-1) = 4m. Let Peter did 3 * 99 + 1 cut. We show that there are 100 or triangles or quadrangles 100.
Assume that this is not the case. Indeed, we let (3 * 99 + 1) of the polygon have n 1 , n 2 , n 3 * 99 + 1 vertices, respectively, while
4 (3 * 99 + 1) = n 1 + ... + n * 3 + 1 99 > 3 * 99 + 4 * 99 + 5 ((3 * 99 + 1) -99-99)
(Ie. A. Of the numbers n
5). Then

, ... is at most equal to 99 3 to 99 4 equal, and the remaining number is not less than
12 + 4 99 * > 99 * 7 + 99 * 5 + 5

therefore 4 > 5 - a contradiction. So, after 3 * 99 + 1 Petya's sections there are 100 polygons with the same
number of vertices.
Competition in Physics
Tasks
1.

From the tower height H jumping stunt tied cord. The length and rigidity of the cord are chosen so that
stunt stops near the ground. Made some hesitation, stuntman hangs above the earth at an altitude of L.
Find the maximum speed that the stunt was during the jump.

2.

Water is poured into the bottle to the point where the bottle begins to narrow, and the top was poured
kerosene (kerosene lighter than water and not miscible with it). The bottle is then shaken several times
and put in place. How to change the pressure on the bottom of the bottle?

3.

Ball from pind-pong table falls from the Ostankino Tower and elastically strikes the ground. Determine
the acceleration of the ball immediately after take-off. Qualitatively draw a graph of acceleration versus
time.

4.

Narrow plate of length L lies on a rough horizontal table. Plate starts to rotate at a constant low speed by
applying a force (finger) on the side at one end of the plate. Determine where the point around which the
plate.

5.

Consider the following model of a clock. The light beam is successively reflected by two mirrors arranged
parallel at a distance L. Each time the beam is reflected from the mirror 2, the electronic circuit for
increasing the clock 2L / c, c - velocity of light.
Show that if these clocks give some speed v in a direction parallel to the plane of the mirror, the speed
of the clock is reduced, and find how many times (this is - the famous "relativistic reduction of the
time").

(Reminder: the speed of light in any frame of reference is the same and equal to c = 300,000 km /

sec) .
6.

As you know, on a moonlit night on the water forms the light path (where should be a reflection of the
moon). Explain (qualitatively) its shape. As it depends on the width of the wave heights (or ripples) on
the water?

7.

Before you caliper. Determine (with his help) what color - red or green corresponds to the shorter
wavelengths of light. Note: one of the first who vysskazala the idea that the color is determined by the
wavelength was Euler. He believed that the red color corresponds to a smaller wavelength than the
green. Come up with a simple way to check this. Was Euler rights?

Problem solving competition in physics


1. The speed stunt will be a maximum at a time when the maximum of its kinetic energy. The kinetic energy at any given time is equal to the
work committed over stuntman external forces at a given moment. External forces that should be considered in this problem are the force of
gravity and the force of the elastic cord. Let the cord length is l, stiffness k, mass stunt m, the gravitational acceleration g. If we denote by x
the distance from the top of the tower, which is flying stunt to this point in time, the work force of gravity at this point is mgx, and the work of
an elastic spring -kx 2 / 2. Work elastic force is negative, since this force is directed against the movement stunt. We obtain the expression for
the kinetic energy stunt
mv 2 / 2 = mgx-k (xl) 2 / 2 = - (k / 2) (x - ((m / k) g + l)) 2 + (m 2 g 2 / 2k) + mgl
The value of this expression is maximal and equal to (m

g 2 / 2k) + mgl, when (x - ((m / k) g + l)) = 0, i.e.

m (v Max ) 2 / 2 = (m 2 g 2 / 2k) + mgl.


From Where
v Max = ((m / k) G 2 + 2GL) 1/2 .
It remains unknown to us here to express the magnitude of the problem through data. To do this, we rewrite the
conditions of the problem in the form of equations containing these unknown quantities. What stunt stops at the
ground, means that the work of the external forces is equal to 0 when the tensile cord still Hl, ie
0 = - (k (Hl) 2 / 2) + mgH.
That stunt eventually freezes at a height L from the ground, means that the elastic force of the pinch adjustment
equal to the force of gravity, i.e.
k (HLl) = mg.
For computational convenience we denote u = k / m. Then our terms and conditions can be rewritten as two
equations
u (Hl) 2 = gH, u (HLl) = g
with respect to two unknown l and u. Solving this system and substituting the solution into the expression for the
maximum speed, we get the answer:
v Max = ((G 2 / u) + 2GL) 1/2 = G (H + L-(2H (L (H / 2))) 1/2 ).
2. Choose a column of liquid from the surface to the bottom of the bottle. Shaking up to a column made up of clean water column and the
column of kerosene over the water column. Pressure on the bottom after shaking changes so as to change the weight of the selected column,
which at the same size will now consist of water and kerosene. So, find out the column will be easier or harder. First consider that a very
narrow neck. Then the kerosene fraction in a mixture with water is very small, and the weight of the column of our practically become equal
to the weight of the column of pure water. This column will be so severe initial composite column as the column weight of kerosene less than
the weight of the column of water, which replaced it in the neck after shaking. Hence, the bottom pressure will increase. Then it remains to
note that the column weight of a mixture of kerosene and water can be equal to the weight of the composite column to shake only when the
width of the neck of the bottle becomes equal width.

3. The ball is very light, so you need to take into account air resistance to its motion as well as it is necessary for the fall in the air a feather
light. As the air resistance increases with increasing speed of the object in the air, then falling with a high tower, the ball eventually attains a
speed at which the air resistance force becomes equal to the gravity of the earth ball - its weight. After that, the speed of the ball stops
changing, because the sum of forces acting on it is equal to 0. With this speed and it will reach the ground. The ball is very elastic, therefore,
after hitting the ground, the magnitude of its velocity will not change much, but the speed will be directed up now. The force of air resistance,
remains the same in size, that is equal to the weight, will be directed down now. At this point the value of the sum of the forces applied to the
ball, will be twice the strength of his attraction Earth, and thus the acceleration of the ball is twice the gravitational acceleration g.
4. We assume that a moving plate portion of a fixed length s of the friction force acting ks, where k - coefficient of friction which is
independent of the position on the plate portion, that is the same throughout its length and is independent of the velocity plot. Then, in order
to maintain uniform motion applied finger force F must uravnoveshivapt total force of friction:
F = kL.
That the movement is a uniform rotation around a fixed center means that no external force with respect to this
center is 0. Let l be the distance from the pin to the center of rotation. Then the value of the moment of the force
F is equal to Fl = kLl. We calculate the friction torque acting, for example, on a segment from the center to the
end plate, which acts on the finger. Coordinates of a point on this segment can be regarded as the distance from
it to the center of rotation. We divide the interval into N small plots points with coordinates x 0 , x 1 , ..., x N (in
this case it turns out that x 0 = 0 and x N = L) and add the value of the friction torque acting on these sites . The
magnitude of the frictional force applied to the portion of x i to x i + 1 , k is equal to (x i + 1 -x i ). Distance from the
center of rotation to the point of application of this force can be approximately regarded as the distance to the
middle of the plot (x i + 1 + x i ) / 2. Then the value of the friction force at the moment of this portion is equal to k
(x i + 1 -x i ) (x i + 1 + x i ) / 2. Adding points in all areas, we obtain
((1/2) k (x N ) 2 - (1/2) k (x N-1 ) 2 ) + ... ((1/2) k (x 3 ) 2 - (1/2) k (x 2 ) 2 ) + ((1/2) k (x 2 ) 2 - (1/2) k (x 1 ) 2 ) + ((1/2) k (x 1 ) 2 - (1/2 ) k (x 0 ) 2 ) =
= (KL 2 / 2) - (k0 2 / 2) = kl 2 / 2.
Likewise we find that the magnitude of the friction torque acting on the segment on the other side of the center
is equal to
k (Ll) 2 / 2.
Given the direction of the forces, we write the condition for the vanishing of the sum of the moments of the
external forces
0 = Fl - (kl 2 / 2) - (k (Ll) 2 / 2) = kLl - (kl 2 / 2) - (k (Ll) 2 / 2).
From this equation we find that the distance from the finger to the center of rotation
l = (1- (2 1/2 / 2)) L
5. Let us find the time between two successive reflections of the beam from the mirror to the observer 2 K, which sees our model moves
with velocity v parallel to the plane mirrors. To do this, the length of the path traveled by the beam between the reflections, divided by the
speed of its movement. The speed of the beam in the reference frame K (as in any other frame of reference) is equal to c = 300,000 km /
sec. It remains to find the path length (see. Figure), which is equal to A 1 B + BA 2 = 2A 1 B. You can write the equation
A 1 B 2 = A 1 C 2 + BC 2 .
where BC = L, A 1, C = A 1 A 2 / 2. Express the members of this equation in terms of the data of the problem and
the required time . A 1 A 2 - is the distance at which moves our model in the time between two reflections,
A 1, A 2 = v . On the other hand, A 1 B + BA 2 - is the distance that light passed during the time between two
reflections, A 1 B + BA 2 = c , and hence, A 1 with B = / 2. Substituting these expressions in our equation, we
obtain the equation for :
(C / 2) 2 + (v / 2) 2 = L 2 ,
from where
= 2L / (C (1- (v / C) 2 ) 1/2 ).
Thus, an observer noted K time between two reflections
= '/ (1- (v / C) 2 ) 1/2 .
where '= 2L / C - the time between these two events marked by an electronic circuit, for which our model is
stationary. This means that the speed of the clock, driven by an electronic circuit, after imparting a velocity

model v, will be less than the clock speed observer ostavshekosya stand still in / '= 1 / (1- (v / c)

1/2

times.

6. If the water surface was mirror smooth, no track, we would not have seen. Water would create a clear reflection of the moon, visible in one
specific point on the surface of the mirror. If we see a track, this means that the surface waves act as small mirrors which are due to the
different tilt send us moon light from points that are away from the place of its reflection mirror smooth. Variation in position of the points on
which falls to us moonlight, i.e. the track width, the greater the slope of a strong possibility of small mirrors, that is, the higher the water
waves. If viewed as a wave of convex reflecting surface is incident to the beam on the surface of its reflection from all points on the surface of
a cone, which is wider, the more convex surface, or higher than the wave (in the case where the surface is flat, i.e. zero wave height This cone
is compressed into a reflected beam).Adjudged whether reflected from the water space of the light to the observer, i.e. whether it is in place
within the lunar tracks depends on whether the observer is located within the cone of rays created at that location with a maximum wave
height. If we consider the maximal cones of reflected rays at different locations of the water within which enters the observer standing on the
shore, it turns out that the path widens when approaching the observer.
7. All that we need in this experiment from the calipers - a narrow slit that can be made between his wings. To zaintereovatsya this gap, we
must remember that the intensity of the received light waves passing through the slit depends on the angle distribution after the gap. For
waves of a given frequency, this dependence appears in the form of alternating-receiver on the screen (or the retina of the observer) of light
and dark bands parallel to the slit. This phenomenon is called diffraction. The fact that the waves arriving at a given point from different
angles slit may amplify or attenuate each other depending on the difference of the lengths of the paths that they passed.If the wavelength is
much greater than the width of the gap, the waves coming from different points of cracks practically do not differ from each other and
reinforce each other znachittolko regardless of the propagation angle. For an observer it looks as one broad band light. Dependence of the
intensity on the angle becomes stronger, and therefore the width of the strips and the distance between them is smaller, the shorter the
wavelength.Thus, if the distance between the strips of one color is less than the other, the colors have a wavelength of shorter length. And if
you look at the bands of red and green colors of the light transmitted through the leaf calipers, it becomes clear that Euler was wrong.

Competition in Chemistry
Tasks
1.

Masa piece of chalk and 50 g of calcined residue was dissolved in 1 liter of water. Through the solution at
room temperature was omitted chloro obtained by reacting 250 ml 20% HCl solution ( = 1.1 g / cm 3 )
with an excess of potassium permanganate? a) Which substances contained in the solution? b) Evaluate
the content in it the main reaction product. c) Determine the mass fraction of calcium carbonate in the
original piece of chalk, considering that chlorine has reacted completely.

2.

The combustion oxygen unknown substance formed 3.6ml water and 2.24 liters of nitrogen (standard
conditions). Relative vapor density of the substance to hydrogen is 16. Determine the molecular formula
of the substance. What properties it possesses? Where to use?

3.

Determine how many grams of a 10% solution of sulfur oxide (6) into pure sulfuric acid and 60% solution
of sulfuric acid is necessary to prepare 480 g of a 90% solution of acid.

4.

For a complete combustion of the organic substances required twice less oxygen than for complete
combustion of the next member of the homologous series. It can be any compound.

5.

Four chemical element designated by the letters A, B, C, D. Pick such reactions, which can be encrypted
as follows:
a) 2CA + A 2 = 2CA 2
b) D 4 C 3 + 6B 2 A = 3CB 4 + 2D 2 A 3
a) CB 2 A 2 + CB 4 A = C 2 B 4 A 2 + B 2 A

6.

Metal plate weighing 50 g after staying in a solution of hydrochloric acid decreased in weight by 1.68%,
with 0.336 liters of gas stood out. From what can be made of metal plate?

7.

How to change the rate of reaction between the molecules of nitric oxide (2), and oxygen, if a sealed
vessel filled with a mixture of these gases in a ratio of 2: 1, the pressure increase is 3 times?

Problem solving competition in Chemistry


1. Assuming that the chalk is pure CaCO 3 , we have 0.5 moles of calcium carbonate. For equation
CaCO 3 = CaO + CO 2
amount of CaO and 0.5 mol amount. After the process of
CaO + H 2 O = Ca (OH) 2
obtain 0.5 g of 37 mol or lime.
In 250 ml of 20% HCl solution sorderzhitsya 250 * 1.1 * 55 = 0.2 g of hydrogen chloride, which is 55 / 36.5 = 1.507 mol. For equation
2KMnO 4 + 16HCl = 2MnCl 2 + 5Cl 2 + 2KCl + 18H 2 O
obtain 1.507 * 5/16 = 0.471 mol or 17.19 g of chlorine.
a) reaction of lime with chlorine at room temperature proceeds according to the equation
Ca (OH) 2 + Cl 2 = CaOCl 2 + H 2 O
It is evident that the lack of chlorine, and may be present in solution in addition to the main product CaOCl
Ca (OH) Cl, CaCl 2 , Ca (OH) OCl, Ca (OCl) 2 , Ca (OH) 2 .

and

b) The maximum content CaOCl 2 solution is


(0.471 mole * 127 (g / mol)) / (100 g + 37 g + 17.9 g) = 0.0567 or 5.67%
c) If reaction (1) passed quantitatively, the lime, and accordingly, the oxide and calcium carbonate was 0.471 mole. In this case, the mass
CaCO 3 : m = 0,471 * 100 mole (g / mol) = 47.1 Then, the (CaCO 3 ) = 47.1 / 50 = 0.942.
2. The substance may comprise hydrogen, nitrogen, kislolrod. However, the relative molecular mass of the substance 16 * 2 = 32 limits the
number of oxygen atoms to one or eliminates it. By the condition of nitrogen formed 2.24 liters, which is 0.1 mol. Hydrogen 0.4 mole
water; 0.1 mol of N 2 were 0.2 moles of nitrogen atoms.
The ratio of nitrogen and hydrogen in the compound is 0.2: 0.4 = 1: 2. Simple formula - NH 2 . M R (NH 2 ) = 16; M R (N 2 H 4 ) = 32 to
coincide with the defined problem statement. From the formula of the starting material H 2 N-NH 2 . This hydrazine at room temperature it is a
colorless liquid that is used as one of the propellant components.
3. 10 g of 10% oleum contains 10 g of SO 3 and 100 g of 90% sulfuric acid. If the count content of SO 3 , then 90 g of H 2 SO 4 is contained in
all (18/98) * 90 = 16.53 g of water, the rest 100-16,53 = 83.47 g oleum is SO 3 . Similarly, in the case of 60% sulfuric acid: 40 + 60 * (18/98)
= 11,02 + 51,02 = 40 g of water and 48.98 g of SO 3 . In 480 g of 90% sulfuric acid contains 0.9 * 480 = 432 g of P 2 SO 4 or (80/98) * 432 =
352.65 g SO 3 and (480-352,65) = 127.35 g Water . Suppose we must take x g oleum, then a 60% acid - (480-x) In the oleum contains (x /
100) * 83.47 g SO 3 , and acid ((480-x) / 100) * 48.98 g of SO 3 , for a total of 352.65 g of SO 3 . Hence:
(X / 100) + 83.47 * ((480-x) / 100) * 48.98 = 352.65.
Solving the equation, we get x = 340,8 g of oleum, and a 60% acid, respectively 480-340,8 = 139.2 g
4. For the oxidation of the group -CH 2 - (homologous difference) need three oxygen atoms. Therefore, the starting material must also be
oxidized by three oxygen atoms, then the next member of the homologous series will be oxidized by six oxygen atoms.
C x H y + (x + (y / 2)) O 2 = xCO 2 + (y / 2) H 2 O,
C x 1 + H 2 + y + (x + (y / 2) +1,5) O 2 = (x + 1) CO 2 + ((y / 2) +1) H 2 O
Examples of substances oxidized by three oxygen atoms
CH 3 OH
HOOCCH
(OH)
COOH
CH 2 N 2 HO-CH 2 -COOH
HO-C ( COOH) 3 - trikarboksimetanol.

(the first term

gamologicheskih series):
methanol,
gidroksimalonovaya
acid,
diazomethane,
hydroxyacetic
acid,

5. A - oxygen, B - is hydrogen, C - carbon, D - aluminum.


6. Reaction equation: M + nHCl MCl = n + (n / 2) H 2 . The weight loss was 50 * 0.0168 = 0.84 g of metal x is isolated (n / 2) * 22.4 L H 2 ,
and 0.84 g - 0.336 H L 2 , x = (0,84 * n * 22.4) / (2 * 0,336) = 28n of
If a monovalent metal, m. E. N = 1, it is the relative atomic weight 28 (silicon). However, silicon does not dissolve in hydrochloric acid and
may be monovalent. For n = 2 the relative atomic mass is 56 tons. E. Metal - iron.You can check the options with n = 3 and 4, and make sure
that the correct answer - only iron.
7. Reaction equation: 2NO + O 2 = 2NO 2 .
Denote the concentration of NO letter a, O 2 - b, then to the pressure change speed reuktsii V 1 = ka 2 b. With increasing pressure, the
concentration increased by 3 times, skrost is V 2 = k (3a) 2 3b = 27kab; increasing the speed V 2 / V 1 = 27 times.
Competition in Mathematics
Tasks
Grades 7-9 (or younger).
1.

By the banks of the Nile came a company of six people: three Bedouin, each with his wife. At the shore is
a boat with oars, which stands only two people. Bedouin can not allow his wife was without him in the
company of other men. Can the whole company to cross to the other side? See comment on the
condition of the jury of this problem.

2.

In triangle ABC, angle A is 120 , the point D lies on the bisector of the angle A, and AD = AB + AC. Prove
that the triangle DBC - equilateral.

3.

Written in a circle 7 natural numbers. It is known that in each pair of adjacent numbers divided by one
another. Prove that there exists a pair of adjacent numbers and not with the same property.

10-11.
1.

Number of 1/42 spread in infinite decimal fraction. Then crossed 1997 th digit after the decimal point,
and all the figures standing to the right of strikeout numbers have shifted to the left one. How many
more: new or original?

2.

Is it possible to cut an equilateral triangle into five distinct isosceles triangles.

3.

Antique dealer bought 99 identical in form of ancient coins. He was told that exactly one of the coins - a
fake - this is easier (and real weigh the same). How using pan scales without weights for weighing 7 to
identify the counterfeit coin, if an art dealer does not solve any coin weigh more than two times?

Problem solving in mathematics competition


1.

(7-9).
The jury's comment on the condition of the problem 1 for grades 7-9
The jury believed that the wife can not "be in the company of another Bedouin without her husband"
even infinitesimal time. That is the situation when a woman swims in a boat to the shore, where there is
her husband, but there is another Bedouin, and without leaving the boat sails away, was, according to
the jury, contrary to the statement of the problem. However inadvertently, this condition explicitly in the
final text of the problem has not got, and when checking for his failure to score was not reduced. Any
other decision of the jury students could rightly challenge, since the concept of "being with" uniquely
defines
the
above
situation.
Back
to
the
condition
of
the
problem.
We introduce the notation: B1, B2 and B3 - Bedouins, x1, x2, x3 - their wives, [} - boat.
Bank of the Nile

Nile

The other bank of the Nile

B1 + x1, x2 + B2, B3 + R3

[}

There is no one

B1, B2, B3 + R3

[X1, x2}

There is no one

B1, B2, B3 + R3

{X1]

R2

B1, B2, B3

[X1, R3}

R2

B1, B2, B3

{X1]

R2, R3

B1 + x1

[B2, B3}

R2, R3

B1 + x1

{B2 + x2]

B3 + R3

X1, x2

[B1, B2}

B3 + R3

X1, x2

{R3]

B1, B2, B3

R2

[X1, R3}

B1, B2, B3

R2

{R3]

B1 + x1, B2, B3

There is no one

[X2, x3}

B1 + x1, B2, B3

There is no one

[No one}

B1 + x1, x2 + B2, B3 + R3

2.

(7-9). Note on the segment AD point K such that AK = AB. Then by hypothesis KD = AC. ABK is
equilateral triangle, since it has two equal sides and one of the angles of 60 . Therefore, the triangles
ABC and KBD are equal on both sides (AB = KB and KD = BC) and the angle between them (the angle
BAC = angle BKD = 120 ). So the sun = BD and angle DBK = angle CBA. Adding to both sides of the last
equality angle KBC, get the angle DBC = angle KBA. Thus, triangle DBC - isosceles 60 at the vertex, so
equilateral. 1

3.

(7-9). Connect pairs of adjacent numbers so that the arrow came from the fold (so-called number that is
divisible by the divisor) to the splitter (if neighboring numbers are equal, then the direction of the arrow
is chosen arbitrarily). The total number of arrows is odd (7), so they can not alternate directions. Hence,
any two adjacent arrows in the same direction: x -> y -> z. This means that x is divided by y, and y is
divisible by z. That is, x = ay and y = bz (a and b - integers, a = x: y and b = y: z). Therefore, x = ay = a
(bz) = abz. It follows that x divided by z (potomuchto x: z = (abz): z = abz: z = ab (z: z) = ab * 1 = ab).

1.

(10-11). Number 1/42 can be represented in the form of a periodic decimal fraction. The period begins
with two decimal places (two) and consists of 6 numbers: 238095 (1/42 = 0.0238095238 ...). Set it is
possible, for example, by dividing by 42 one "column". Since 1997, when divided by 6 gives the
remainder 5, 1997, I figure recorded the same number as the fifth - zero, and the next - a nine. Hence,
the new number is greater.
Infinite periodic decimals are indicated as follows: 1/42 = 0.0 (238095).

2.

(10-11). You can. Let ABC - this equilateral triangle.


Note on the AC side point D, so that AD = DC.
Note on side BC point E, so that BE = CE.
Note on the segment BD point F, so that BF = AF.
Note on the segment AF point G, so that AG = GF.
DEC (equilateral), DGF (equilateral), BED (BE = ED), BFA (BF = FA), AGD (AG = AD) - the required
equilateral triangles.
The possibility of constructing the conditions specified are distinct and equilateral triangles are almost
obvious from the figure, a rigorous proof of these assertions is long and uninteresting and is not included
here.
There are other options for cutting.

3.

(10-11). First put on the two scales to 13 coins, then (if the balance is in equilibrium) uberm them and
put on 11 more of who took no, then 9,7,5,3 and 1 to as long as one of the bowls do not outweigh .
If this does not happen, then after the seventh weighing (when the scales will be only one coin) will be only one coin, which was
not involved in weighing. It is false.
If some kind of weighing bowl outweighed, then the false coin is in the other side. The total number of coins in this bowl denote 2k
+ 1 (every time we we place on one side of an odd number of coins), and we've used 7-k weighings, with each coin weighed no
more than once. Therefore, it remains to find a false coin in a group of (2k + 1) ones in k weighings, each coin weighing not more
than once. This can be done to break all the coins in the group, except one, divided into pairs k and the weight of the coins
successively compare each pair. If some weighing balance is violated, then the lighter coin is counterfeit. Otherwise, false coin - the
remaining single.

Competition in Physics
On a sheet with the task was given this explanation: After the task number in parentheses indicates how classes,
this problem is recommended. To solve problems not of their class is allowed. Pay attention to the quality of
physical explanation: the work, which is good, with explanations resolved two or three tasks will be highly
appreciated - higher than the work in which there are fuzzy reasoning on many tasks.

Assignments
1.

(7-9) Compares two experiences.


1.

On the table is laid wooden ruler thickness of 1-2 mm and a length of 50 cm so that slightly more
than half the line lay on the table, and the rest hung. If you click on the overhanging part, the
line will bend easily.

2.

Line also is laid, but the portion of the line that is on the table is laid a sheet of paper. Now, if you
quickly press the overhanging part, ruler became like heavier.

Explain why this is so and describe the phenomena that occur when pressing on the line.
2.

(7-9) It is possible to imagine such a possibility, that in some countries the duration of the day was
defined as the time between adjacent sunrise. What time of year in this country, "day" was the longest,
and in some - the shortest.

3.

(7-9) Why eventually radius bend of the river slowly increases?

4.

(9-11) a) Try to estimate the order of magnitude of the coefficient of friction end of your pen on the desk
or paper as follows: first put pen almost vertically, propped on her finger, and then tilt the handle up
until it begins to slip.
b ) Estimate the order of work on writing your work at the competition in physics. There is a view only
the mechanical work on the printing on the paper; costs of other forms of energy are not counted. Note:
If the task requires something to evaluate in order of magnitude, the greater accuracy is required; need
just an order of magnitude when the error is 10 times is quite acceptable.

5.

(9-11) Contemporary Galileo, professor at the University of Padua Sanktorius invented the first
thermometer, which was a glass ball filled with air and the glass tube is connected to the
atmosphere. The tube is placed in a drop of water, which overlaps message cavity vessel with the
external environment. With this device Sanktorius are diagnosed his patients (he was a doctor). The
patient took the ball in his mouth, and a minute later on the Status of water drops Sanktorius determines
what the patient fever. Depend on whether such a thermometer readings of the weather, in particular
whether they will be different in clear weather and rain?

6.

(10-11) Try to invent a device that creates a magnified image of the object and it does not use a curved
surface.

7.

(10-11) Try to come up with such an experience, in which the charge is moved, and the magnetic field
does not arise.

In summing up all the tasks are considered equivalent (paragraphs a) and b) of the problem 4 were
considered separate tasks).
Reply to the competition in physics
1.

The thing is, of course, a piece of paper. However, the line has become "harder" is not due to the extra
weight of a piece of paper (its mass is very small), and atmospheric pressure (this is known enough
experience to demonstrate the atmospheric pressure). When pressing the sharp edge of the line hangs it
rotates around the edge of the table by lifting the middle of the sheet of paper, whereby a vacuum is
created under the sheet (air pressure below atmospheric) and pressures above the plate - atmospheric
(or slightly more). As a result, a paper sheet, and, consequently, the line force directed downwards. The
resulting space region with reduced pressure is communicated with the atmosphere only through the
"channels" along the edges of the ruler, in which also the pressure is less than atmospheric (for two
reasons: due to the vacuum existing in the fact that the air moves through "channels" on the paper a
high speed). If the force due to the pressure difference in the channels and air would be enough to drag

the table pressed him atmospheric pressure edges of the paper (with a strong impact on the ruler),
channels slam shut, the air inlet under the sheet of paper at a time to stop and line will seem heavy.
This description has been compiled on the basis of the jury's own observations. In various literature and
the works of the participants of the tournament there are other explanations, slightly different than
shown. All explanations, correctly reflecting the essence of the phenomenon, the jury will be counted as
correct.
2.

It is clear that since the "Earth rotates once on its axis in 24 hours, at any time of the year" (as written
almost everything, but a lot on it and stopped), the difference between adjacent sunrise will be less than
24 hours when the day is increasing and more 24 hours increased when the night. Ie from the date of the
winter solstice (21 or December 22) until the day of the summer solstice (21 or June 22) night in this
country will be less than 24 hours, and at other times - more. It is known that the highest intensity
change of day length reaches during the spring and fall equinoxes. (You can check this on the calendar,
but you can find a purely geometrical reasons - think what.) So, the shortest day in this country - the
vernal equinox (March 21 or 20 depending on the year), and the longest - on the day of autumn equinox
(September 23). (Recall that the tournament was held on September 28.) The exact values of the times
of sunrise and sunset for Moscow can be found, for example, in "Diary of Moscow
schoolchildren." Equinox (spring and autumn) - the moment of passing the center of the Sun in its
apparent motion on the celestial sphere through the celestial the equator. At this time, the length of day
and night is the same. Solstice (summer and winter) - moments when the height of the sun above the
horizon at noon, the maximum and minimum.

3.

This task (question) the jury stuck in option at the last moment as a problem for younger students (as
problems for the lower grades in the variant lacking), hoping to get around this answer: "Because of the
centrifugal force of the water interacts strongly with the outer coast than with internal and stronger blurs
it, thereby increasing the turning radius (bend) " . However, immediately after the commencement of the
audit work it became clear that the issue is not as simple as it was supposed to, and clearly not for the
junior. Briefly turning process River is as follows. On the right section of the river is known, the maximum
speed at the center ("Bistrica") and decreases as it approaches the shore due to the friction of them. At
the turn of water tends to inertia continue straight for her turn must apply a force, with the larger, the
more its speed (by rotating the ball of mass m on a string in a circle of radius R with a velocity v thread
tension force is mv 2 / R). The result is a sort of water velocity and flow rate in the outer bank of rotation
is greater. We assume the obvious, that the intensity of erosion of the coast increases with increasing
flow rate. Also, we assume that a straight section of the river, this rate is an equilibrium, since there is a
form of the shore does not change with time (speed increases the width of the river increases, which
leads to the erosion of beaches, t. E. To increase the width of the bed. But because of the this rate of flow
is reduced, leading to the shores of namyvaniyu (deposition of sedimentary rocks). And vice
versa.). Then, if a straight section of the flow velocity at both banks was an equilibrium, then when you
turn after the "sorting" on the outer coast of the speed is greater than the equilibrium and it will be
eroded, and at the inner - less than the equilibrium, and it will pan out. In addition to "sort" the water
velocity at the turn of the river there is another effect: after the "slow" the water was near the inner
bank, and "fast" - from the outside, "slow" the water begins to "crush" the "fast", which leads to an even
greater increase in the rate of the latter. On the other hand, the "fast" water after the interaction with
the external bank becomes turbid (there are eroded from the shore rock particles) and loses its
speed. Then she pushed to the inner bank of the available water flows faster. At the inner bank, in turn,
loss occurs "haze" in the precipitate, i.e. additional namyvanie it. The above reasoning is a simplified
description of the process at the level of the school curriculum in physics, which obviously does not take
into account many important circumstances, so it is certainly not correct in terms of geology.
As it turned out, this task was interested in even the Albert Einstein, in which "Reasons Education
meanders in rivers and so-called Beer law "can find an explanation for the many features of the flow of
rivers, for example, why the rivers of the northern hemisphere blur mainly the right bank, and the south left. And in 1997 on this topic has been defended his doctoral dissertation in which the problem is not
completely solved (so far!).

4.

Somehow, this problem has caused discontent decisive (maybe because we are the most essential part
of their work, not mechanical, not interested): "if so try often, all the tables in the school will be
inscribed."
Nevertheless, many approached correctly to estimate the friction coefficient. (Although at the last
moment, when the tasks have already been handed out, the jury became worried - because if you
specify k tr = 0.1, in a wide range, from 1 to 0.01, the error does not exceed a given accuracy. However,
students on this temptation did not fall . And rightly so - because the jury will evaluate the argument, not
the final figure.) In the figure all the forces acting on the pen are indicated by a bold arrow. R x and R y ,
which are not real forces, and the projections R, shown for convenience only and therefore designated by
arrows a lower fat content).The vertical component of the forces of reaction finger and paper should be
equal (for example, from the condition that 0 torques about the center of the pen - or pen begins to
rotate, we believe that the mass m pens distributed uniformly along its length). Together, they must
balance the force of gravity mg (otherwise the handle will fail under the table), so they are both equal to
mg / 2 (N = mg / 2 and R x = mg / 2). Pen fall when the friction force F on the paper tr becomes smaller,
than the horizontal pressure force component pen on paper. But where does this component, because

the handle simply presses down its gravity? No, it still relies on a finger to solve this problem and the
response of this support R and gives us the horizontal component of R x . Two angle indicated in the
figure symbol a (alpha) as well as angles with mutually perpendicular sides: the reaction force of the
finger R, acting on pen, fountain pen perpendicular and the vertical projection of the horizontal force
perpendicular to the paper surface. By definition, the tangent of an acute angle in a right triangle is
equal to the ratio of the leg opposite the angle to an adjacent. Then TG = A R x / (mg / 2) (as defined by
the tangent to the angle a, which "rests \" finger). Hence, R x = (mg / 2) tg a. This force (R x ) can be
balanced only by the force of static friction F tr.p. pen on paper, which, in turn, can not exceed the force of
friction F tr = kN = k (mg / 2}. Therefore, at the time, when the pen in a continuous decrease in the angle
of a fall, R x = kN (mg / 2) tg a = k (mg / 2) k tr = tg a Thus, estimating the angle of a tangent and finding
it (and even better, you can experiment by definition, building a right-angled triangle containing the
angle a, measure its legs of improvised means, such as the edge of a sheet of graph paper, and
calculating their ratio), we obtain an estimate of the coefficient of friction. Now just find the mechanical
work - it is the product of the frictional force on the handle passed way. It remains to estimate the weight
of the handle and assess the overall length written by you (for example, assuming that all of the letters is the height of squares corresponding to your handwriting) - and the problem is solved.

5.

Actually Sanktorius are diagnosed "for 10 pulse beats," but to solve it is not essential, but only adds to
the long text of the problem. Maybe because the problem could not read to the end, it was making less
than we expected.
Since the first thermometer was connected with the atmosphere, it influenced his work. Moreover, the
thermometer affect atmospheric conditions at the time of its manufacture. Indeed, if the pressure inside
the tube is increased due to increased temperature of the patient, the doctor could see that the droplet
has moved by a greater amount. But if the air pressure in the day was more than daily calibration of the
thermometer, the droplet could remain in place or does not reach a normal level of temperature. But of
the many messages Gidromettsenrta all already known link between atmospheric pressure and clear or
rainy weather. So the weather affect the reading of the thermometer.
But on the other hand, do History has brought to us information about the doctor-charlatan, inaccurately
diagnosed patients with fever? Rather, Sanktorius before measuring the temperature of the patient, the
thermometer calibration is performed by measuring the temperature at, and only then - in a patient. Jury
these details are not known for certain, but indirect evidence of them is the fact that the patient briefly
("for 10 pulse beats") held a thermometer at home, because he was already heated professor!
This example shows that it may not be the right answer monosyllabic (effect-no effect), is important to
correctly reasoning that, depending on the additional assumptions lead to different (but good!)
conclusions.

6.

A list of devices that satisfy the conditions of the problem, these students: the camera obscura, crossed
prisms, holography (hologram recorded at a single wavelength and reproduced at the other), the
diffraction grating with concentric rings, different ways to create optical density inhomogeneities in the
medium (lighter in the air is dissolved in water, salt crystals). There are other options, but they are not
offered to the students and are very hard to describe, so no more description.

7.

Take the spherical surface (e.g., a balloon) at which electric charge is uniformly distributed. To begin to
change its radius without moving her center. Located on the surface charges will obviously move, with
the process of moving is centrally symmetric. Consequently, all of the physical phenomena that result
from this process should also have a center of symmetry. * The only possible (from geometrical
considerations) in this situation, the magnetic field configuration is this: the magnetic field lines diverge
uniformly in all directions from the center of the sphere. But this is impossible, since Magnetic lines of
force may not be endless. *Obviously, the condition of the problem satisfy any centrally symmetric
motion of the charges and the superposition of such movements. The jury is not known whether such a
situation in the other cases. Note: For an asterisk claims in the scientific world there is no single
opinion. We rely on the school curriculum and are not going to contribute to the debate on these issues
in modern physics

Information about the competition jury in Physics


On estimates of the competition in physics tournament them. Lomonosov 1997
Competitions in physics and mathematics tournament it. Lomonosov Moscow Olympiad in mathematics, Tournament cities and some other
similar events are held the same korrellirovanoe a lot of people, so in all these events is used for many years has developed a system of
ratings:
0 - -. - + + / 2 + - +. +!
0, the problem is not recorded in the
- In the solution contained only wrong, have no relation to the correct solution or contained in the problem statements
-. the problem is not solved, but the decision has at least one nontrivial true statement pertaining to the right decision and not contained in (all
estimates are reported to participants, so stamped assessment "minus" in this case may give the student a misconception that in his work
incorrectly absolutely everything, including this very statement)
- + Problem is solved correctly, but the decision has significant progress in the right direction. For the right decision, but their absence,
usually put + - + / 2 in the problem can not be solved sorting options - the correct answer without explanation. In the task, where necessary,
for example, to prove the necessity and sufficiency of something - for the right solution to only one of the parts. In other problems - if the
solution has a sufficiently large number of true moves, but to build one right solution still can not.
+ - The right decision with a significant failure.
+. the problem is solved correctly, but there are insignificant flaws
+ Problem is solved correctly
+! the problem is solved correctly and in the decision contains the original and / or substantially beyond the school curriculum (corresponding
class) points that the jury considers it necessary to emphasize.
Spelling errors are not taken into account (because it is a competition in physics, not in Russian, but also because the primary school children
in solving complex problems of the Contest can not do without grammatical constructions that they have not learned in school). Although,
unfortunately, there are so many errors ...
Arithmetic errors do not affect the meaning of the solution, not taken into account (is reasonable to assume that students 7th grade and older
are able to perform arithmetic operations, such errors are not caused by lack of knowledge, and other factors).
If the work has only correct strikeout decision, it is counted as correct.
If the solution of the problem can not be obtained by brute force of a small number of options (yes / no increase / no change / decrease), and
her work has several options, including the right, count the correct option (even if it is crossed out).
Available for solving incorrect statements that have no semantic connections with the rest of the decision, the assessment is not reduced.
The criteria for determining the winners
Grade 6 and under
Diploma if one + / 2 or better
Officially, the tournament is held for students grades 7-11, but talented schoolchildren also successfully participate in the tournament.
This year, the students of class 5 and younger participate in the competition in physics is not taken.
The competition was attended by two sixth-grader:
Gaifullin Sergey school '10 Zhukovsky, Moscow Region,
evaluation - +. +. 0 0 0 0 0
(This is a very good result for grade 6)
Kolychev Alexander School '58 Moscow
evaluation 0 + / 2 + 0/2 0 0 0 0
(The discussion of the jury also decided to award him a diploma, that is the criterion for grade 6 is purely formal)

7 and 8 classes
Around if one + / 2 or better
Around if the three - + or better
Diploma if one + - or better
Grade 9 and older
Around if one + - or better
Around, if three - + or better
diploma if the two + - or better
The phrase "or better" does not mean subjective view "better or worse", and the presence in the specified number or those estimates or higher.
Any formal justification for the above criteria for determining the winners do not exist. The decision was made after checking all the works
under discussion based on the experience of last year's results, the actual situation at work, personal opinions of the members of the jury.
Check
The audit involved 7 (NN Konstantinov, undergraduate and graduate students, physics). Each work independently verified first 2 times, then
recheck again (usually the third person), which, when there is a discrepancy between the estimates and the estimates and its own opinion to
hold discussions with the other reviewers (sometimes quite tense), and set the final evaluation. At the closing ceremony the students could
challenge them (this year, wishing there was in the past - were).
When checking the work of the different classes were presented the same requirements for the physical side solutions, but takes into account
different levels of intellectual development of students of different ages in the evaluation of logical correctness and validity of the
decision. The difference between the classes into account in the criteria for determining the winners.
Of course, when checking have been developed certain formal criteria, but each work still tested individually, in accordance with common
sense, and the criteria themselves are often formulated in the form of words or phrases such as "prism", "centrifugal force". Bringing them to
the formal compliance with all the assessments require breaching a lot of work for what the jury had no opportunity now (see. The last
paragraph), and the publication of the criteria without such a reduction can cause massive resentment among the tournament participants, their
teachers and parents. Therefore, the criteria are not published.
7 checks - to put it mildly, is not enough (in mathematics at about the same amount of work they were 37). But more than just was not
wishing, most physicists were limited excuses and valuable comments and innovations.If any of the readers willing to make a personal
contribution to the work tasks Tournament next year, its implementation and verification work, the jury happy to support this initiative.
Selection problems
At the competition were invited to a lot of problems (7, one of them with two points), but to get high score was enough to solve a small part
of them, as can be seen from the above-stated criteria. For each task, indicate which classes students recommended to solve it (for each class at least 3 tasks to provide student the opportunity to choose a favorite subject for the task and to avoid a situation where the student can not
solve any problem, because I do not know exactly the formulas and definitions that are needed for this). Students can solve problems and not
the class, summarizing the results of all the tasks are taken into account equally. This rule came as follows: The tournament has been held for
more than 20 years and its rules have changed over time. In particular, in 1992/93, and a few years before the tournament formally held for
grades 7-9, and for 10 and 11 classes at the same time and in the same places Tournament held cities in mathematics. But the ubiquitous high
school students still leaked on Turlov. Combat it was useless and in the 1993/94 school year, they were invited to the event "officially", and
for them was made a separate option in physics. Due to the fact that it successfully solved and several (about 15) schoolchildren in HS
1995/96 the jury decided to conduct an experiment and offer a "senior" mladsheklassnikov all, it was found that almost half of
mladsheklassnyh degrees in physics (mainly in part) due to problem solving "older" version. It became apparent that their division would be
extremely unfair to such students.
The complexity of the task (for the class) approximately corresponds to the level of regional Olympiads (Juniors - fans challenges - can solve
the problems of high school, and high school students, especially for fans of a problem is the university level ( 7, about the movement of the
charge and the magnetic field). Relatively low complexity of most tasks and assessment criteria are due to lack of time (7 competitions for 5
hours) and low (unfortunately) the general level of school physical education in Moscow (much lower than the mathematical or
humanitarian).
The content of the competition problems in physics this tournament is somewhat different from those of the school, district, city and
university competitions. Jury tries to avoid formal synthetic problems, a key element of the decision which is the formal knowledge and
combination formulas and definitions (such problems and so much on the above activities and entrance examinations), mathematical
problems "in the physical Skin" (in the tournament have a competition in mathematics) very simple (comforting) problems (students should
not be in vain to spend their decision time and effort required in other competitions). Preference is given to quality problems and issues, a
simple experimental tasks (which are shown or in the audience, and each participant can do them in place), the tasks for which it is necessary
to understand the content of the physical process or phenomenon, evaluation tasks. The jury also fundamentally waived task, which is difficult
to check. Back to the list of contests

Competition in Chemistry

Tasks for competition in chemistry provided by the Moscow Chemical Lyceum House of scientific and technical creativity of youth in
Moscow.
Solving problems of the above-mentioned organization, unfortunately, are not provided for that tournament organizing committee apologizes.
Participants of the tournament that the jury expects to receive from them three of the proposed solutions of problems of choice. For each task
to grade on a scale from 0 to 5. The final evaluation is the sum of scores for the three tasks otsennnye highest number of points than the other
(points a, b, c, objectives 5 assessed on a 5-point scale individually and summarizing the results were considered separate tasks;
unfortunately, it has not been announced to participants in advance). If such problems but three student solved yet sufficient other, the amount
increased by 1.
Tasks
1.

(Grades 9-11) According to the information preserved in folk tradition, in the first century of our
millennium there was the idea that there is metal as much and planets. Write the chemical symbols of
these metals.

2.

(Grade 9-10) As, using as the starting materials iron filings, salt, baking soda, chalk acid battery for
refueling, water and air, to obtain various iron compounds. The equations of the corresponding reactions,
indicating the conditions of their implementation.

3.

(9-10 grade) proposed a formula of inorganic substances A and for which you have the following
transformation:
A + B -> B
D T -> B + L
E + F -> W
A + F -> And
G + H -> R + D
+ E B -> D
B + D -> D
A + H -> R + E
B+W->I+D

4.

(Grades 9-11) Suggest a pair of colorless solution (no more than five), which is formed by mixing colored
solution.

5.

Suggest a way to distinguish between substances, based on the quality of chemical reactions.
a) (Grades 9-10) FeS, PbS, coal, MnO 2 , CuO (black powder);
b) (grades 9-11) NaNo 3 , Zn (NO 3 ) 2 , Mg (NO 3 ) 2 , HNO 3 , NaOH (colorless aqueous solutions);
c) (class 10-11) C 6 H 4 (OH) CH 3 (o), C 6 H 5 OCH 3 , C 6 H 5 CH 2 OH, C 6 H 5 C (O) H, C 6 H 5 C (O)
CH 3 (colorless liquid). The structural formulas shown below.

6.

(Class 10-11) Some elements form na ' ture of compounds of formulas A x B y , where x does not equal
y. Examples of such compounds to indicate a method of preparation.

7.

(Grades 10-11) Give the structural formulas of the isomers of C

8.

(10-11 class) A mixture of three gaseous substances under normal conditions has the following property:
for complete combustion of one volume of this mixture is required 3/4 volume of oxygen, and in the
result is only water and 1 volume of carbon dioxide. Suggest variants of this mixture (not more than
five), indicating volumetric ratio of its components.

Cl.

Mathematics
In parentheses after the number of tasks specified classes for which the recommended task. To solve problems not of their class is allowed.
1. (6, 7) in a row without gaps discharged all even numbers from 12 to 34 to get the number 121416182022242628303234. whether it is
divided by 24?
2. (6, 7, 8) On the graph paper draw the shape (see. fig.) in the top row - one cell, the second from the top - three cells, in the following order 5 cells, and so on, all series - n. Prove that the total number of cells is the square of a number.

_
_ | _ | _
_ | _ | _ | _ | _
_ | _ | _ | _ | _ | _ | _
| _ | _ | _ | _ | _ | _ | _ |
.....................
_ _ _ _ _ _ _ _
| _ | _ | _ | _ | ....... | _ | _ | _ | _ |
3. (8, 9) Out of all if you can cut a convex quadrilateral parallelogram, three peaks coincide with the three vertices of the quadrilateral?
Explanation: The parallelogram - a quadrilateral whose opposite sides are parallel. Rectangle, square and diamond - also parallelograms.
4. (8, 9, 10, 11) six identical parallelograms Square 1 plastered cube with edge 1. Is it possible to say that all parallelograms - squares? Can
we say that all of them - rectangles?
. See note 3 to the problem.
5. (10, 11) A triangle ABC is inscribed in a circle. Point D - middle arc AC, point K and L are selected on the sides AB and CB, respectively,
so that KL parallel AC. Let K 'and L' - the point of intersection of DK and DL, respectively, with the circle. Prove that the quadrilateral
around KLL'K 'can be described as a circle.
6. (9, 10, 11) has the shape of a square table with a side length n. In the first line of the table is one number - 1. In the second - two numbers the two deuces in the third - three four, and so on:
1
February 2
4 4 4
8 8 8 8
16 16 16
32 32
64
(Here draws a square 4 * 4, but you need to solve the problem not only for this particular case, and it is desirable
for any positive integer n). In each of the following line stands next power of two. The length of the lines first
increases and then decreases, so that they form a square. Prove that the sum of all the numbers of the table is
the square of an integer.
Mathematics - solutions
1. This number is not divisible by 4, since the number made up of his last two digits - 34 - is not divisible by 4. And, then, the number is not
in the condition and divided by 24.
Note. Divisible by 3 will not be able to use, t. To. this number is divisible by 3.
2. The solution is shown in Figure 1. Figure 1.

3. The answer is yes, of all. Consider an arbitrary quadrilateral ABCD . Since the. / A + / B + / C + / D 360 = O , then either / A + / B > 180 O ,
or / C + / D > 180 O . Similarly, a / B + / C > 180 O , or / D + / A > 180 O .Suppose, without loss of
generality, / A + / B > 180 o and / B + / C > 180 O . Then consider the parallelogram with vertices A , B , C and sides AB and BC . Let E - its
fourth top. Then / ABC + / BCE = 180 O < / ABC + /BCD . Similarly, / ABC + / BAE 180 = O < / ABC + / BAD . It follows that the

segments CE and EA lie inside or on the sides of the quadrilateral ABCD . Figure 2.

4. Examples of developments such cubes are shown in Figure 3. Figure 3.

5. Consider NM tangent at the point D (Figure 4). Have / K'KL = / KDM , / K'L'D = / K'DN . Hence K'KL + K'L'L = 180 O , and this means
that around quadrilateral KK'L'L can be described as a circle. Figure 4.

6. (1+2+4+...+2 n )+(2+4+...+2 n +1 )+...+(2 n +2 n +1 +...+2 2 n )=


=(1+2+...+2 n )+2(1+2+...+2 n )+...+2 n (1+2+...+2 n )=
=(1+2+...+2 n )(1+2+...+2 n )=
=(1+2+...+2 n ) 2 .

Math games

Competition Lomonosov Tournament mathematical games - unusual, significantly different from the rest. Alone
environment problems is not enough to understand what the competition and how it goes. As an explanation is
given here
Instructions conducting math games
Math games are held for students 7 - 9 (and under) classes. The main goal - to interest students in mathematics, invite them to mathematical
circles.
Sport side - in second place.
5 hours set aside for the tournament, you should spend a few (3 - 4) sessions mathematical games (each session for an hour and a half). After
filling your audience stop and start new children hang on the door of the announcement of the beginning of the next session. (If the children
very much, you can play games in two classrooms at the same time, if you have enough classrooms and conductive).
Each session runs as. First you have to offer students one of the games. They need to play it together with you. The goal is to explain to
students what a winning strategy, the examples show how you can prove that one of the players will always be able to win at any game
other. Can prompt students to play in the giveaway. If the student confidently offers a clear, but the wrong strategy, you can bet with him that
he will lose by using it (supposedly winning) strategy (and to do so). Games - a creative process.
Then (playing with students in one - two games) issued to them for the task of self-help. They can play this game with one another or with
themselves, and then everyone should own to write on a piece of paper which player has a winning strategy, how and why this strategy is
successful.
Collecting students' sheets (with the number of student cards, as well as his surname, name and number of the school and class), let them go
and get ready for the next session.
As a result you must provide a list of names of the organizing committee of the best pupils (indicating the card number, name, number and
class of the school), and specify the rating items.
Evaluations can be of two types:
"V" - put students who have successfully coped with the task. These students will receive a diploma for a victory in mathematical games.
"E" - put students who fared well, but not well enough to be considered the winner of mathematical games. Such students who received
another estimate "e" on some other items will be awarded diplomas for winning the all-around.
This rating you can take into account not only the written decision, but also the work of students during the session.
The following are options for some games.
Not necessarily limited to the following list. You can offer their games (on the divisibility, reverse, symmetry, etc., etc.), it is only important
that the objectives were not too complex.
Among the games issued a written decision to try to avoid games that are well known to pupils (and as examples of suitable and terrific
game). However, this is not very important.
Spend a fun and exciting competition (rather than as a test).
If possible, take the time to make the following announcement: 2 October and Saturdays at MSU starts mathematical circle for students 6 - 9
years, starting date at 16:00.
Terms of mathematical games
Games are divided by series (each series contains "similar" game).
Perhaps, some games will be complicated.
For seed safely use the most simple and well-known games:
Put a round table on the coppers who can not make a move - lost;
shift chips in a straight line on the 1 or 2 cells forward (who first comes to 15);
in the number of recorded 7 cons, each move one or two adjacent minus replaced by the pros (who can not walk - lost);

etc. etc.
Series 1.
1.1. The playing field is a rectangle of size 2 * n (where n - integer), divided into cells 1 * 1. Played by two, take turns. Each turn the player
fills one or more not shaded cell or two adjacent (horizontally or vertically) is not shaded cells. The player who can not make a move.
1.2 * . The same game as the 1.1 , but the player who made the last move.
1.3. Dan corner of 2n + 1 cells (see Figure 1). Played by two, take turns. Each turn is permitted to paint any one cell, or any two cells with a
common point (even the top). The player who can not make a move. Figure 1.

1.4. The same game as the 1.3 , but the player who made the last move.
1.5. The playing field has the form shown in Figure 2 (there 3n + 2 cells). Played by two, take turns. Each turn the player fills one or more not
shaded cell or two adjacent (horizontally or vertically) is not shaded cells. The player who can not make a move. Figure 2.

Note: The first play (be advised to play) in these games for small n (1, 2, 3, ...).
Series 2.
2.1 (known game). In a series of recorded n units: 1, 1, 1, ..., 1. Each course is allowed between any two digits with the character "+" or
"*". When all n-1 characters will be supplied, the value of the expression is evaluated (executed first all multiplication, and then - addition). If
you got an even number, then won the first, and if odd - the second.
2.2 (known game). The same problem as the 2.1 , but the first win if you get an odd number, and the second - if even.
2.3. In a number of recorded 2n numbers: 1, 2, 1, 2, ..., 1, 2. Each course is allowed between any two digits with the character "+" or
"*". When all 2n-1 characters will be supplied, the value of the expression is evaluated (executed first all multiplication, and then addition). If you got an even number, then won the first, and if odd - the second.
2.4. The same problem as 2.3 , but in a number 2n + 1 number 1, 2, 1, 2, ..., 1, 2, 1.
2.5 - 2.6. The same tasks as the 2.3 - 2.4 , but if you got an even number, then won the second, and if odd - first.
Note: The first play (be advised to play) in these games for small n (1, 2, 3, ...).
Series 3.
3.1 branch with leaves has the form shown in Figure 3 (leaves 2n + 1 ). Figure 3.

Two turns pluck leaves, each move is allowed to disrupt or one sheet, or any pair of leaves growing from a single point. The winner is the one
who picks the last sheet.
3.2 The same problem as that of 3.1 , but the branch has the form shown in Figure 4. Figure 4.

3.3, 3.4 The same tasks as the 3.1 and 3.2 , but the one who picks the last leaf loses.
Note: The first play (be advised to play) in these games for small n (1, 2, 3, ...).
Series 4.
4.1. (a famous game). The hour hand is set to 12 hours. Played by two, take turns. Each turn you can move the arrow on the 1 or 2 hours
ahead. Who first put the arrow again for 12 hours, wins.
4.2. (a famous game). The same problem, but each move can move the arrow to 2 or 3 hours in advance.
Note: variable length dial shifts themselves invented the game on reverse.
4.3. The hour hand is set to 12 hours. Played by two, take turns. Each turn you can move the arrow can be 2 hours ahead or back 1 hour. The
player who first moved the arrow on already met a number.
Series 5.
5.1 Dan convex n-gon. Played by two, each move can be performed is not conducted by the diagonal of a polygon. It is prohibited to another
diagonal had points in common with the already conducted diagonals. The player who can not make a move.
5.2 The same problem as that of 4.1 , but the player who made the last move.
Note: the first play (play advise) in these games for small n (4, 5, 6, 7, 8, 9, ...).

Physics
After the job number indicates which classes this object is recommended. To solve problems not of their class is allowed. Pay attention to the
quality of physical explanation: the works, which is good, with explanations resolved two or three tasks will be highly appreciated, higher
than the work in which there are fuzzy reasoning on many tasks.
1. (7-9) Athletes run a column length of 70 m at a speed of 20 km / h. Towards the coach runs at a speed of 15 km / h. Every athlete, coming
up with the coach, turns around and starts running back with the same speed of 20 km / h. What will be the length of the column, where all
athletes will unfold?
2. (7-9) The driver noticed that the raindrops on the windshield when driving the car behave differently: one slide down, and the other under
the pressure of a headwind going up (the car is moving, and the "wipers" do not work). But the wind drops for all the same; What's the
difference?
3. (7-11 grades) Someone looking into the laser lens (and grossly violating the safety rules; fortunately, the laser was turned off), I saw a
strange picture, very similar to a biological object. What could it be? This laser (if included) red light.
4. (7-9) vertical pipes of large diameter, a closed bottom (bottom) filled with water and inclined at an angle of 30 O . After that, the surface of
the water and let the little boat began slowly releasing water through the hole in the bottom. How will move the boat (or more vertically down
somehow?)
5. (10-11) A man sees in the ticker page notebook with one eye through a magnifying glass. The distance between the ticker on a sheet of
notebook equal to s, the distance between the eye and the notebook constant and equal to h. Construct a graph of the distance s 1 between the

ticker on the image of the page in the loop on the distance x between the magnifier and pages. The focal length of the magnifying glass is
equal to f.
6. (grades 8-10) The famous biologist Cephas Mokiyevich discovered a new species of mold, has a wonderful property completely absorb
heat. To test a piece of the mold is placed into a vessel to substantially completely insulated from the environment, the temperature inside was
monitored using a thermocouple. During the experiment, a special committee was convinced that with time, the temperature in the vessel
actually decreased by more than 1 o C. Thereafter, the vessel was opened and using the most modern equipment and precise, it was found that
the temperature inside the room is really below. After some discussion, the Commission confirmed the discovery of Cephas Mokiyevich. And
you?
7. (7-8 classes) Deviation volume of water using beakers considered equal to half of the scale interval of the beaker. Why, for division of the
scale can normally be divided into eye 2, and the larger number of parts (e.g. 3 or 4)?
8. (grades 8-10) Spiral electric included and measured its resistance. Then straightened, and turned again to measure the resistance. Are the
values obtained, and if not, what more and why?
9. (8-11 grades) has a long U-shaped tube filled with water. According to one of its vertical tubes floats large air bubble (radius of the sphere
with the same as that of the bladder, the volume greater than the radius of the tube.) In any of the two tubes over the water level?
10. (9-11 grades) Rough rod radius r 1 is free to rotate around a fixed axis (the axis passes through the center of the circle, which are the ends
of the rod). To it is pressed against the other, designated in the same manner and also rough, the core radius r 2 . The angle between the rods
(their axes) . The second rod is rotated about its axis with angular velocity 2 . What is the angular velocity 1 while rotating the first
rod?
11. (8-11 grades) "Comet rotated elliptical orbit around the star, with the star is in one of the foci of the ellipse" - read Dunno in popular
science book. "I wonder - he thought - if the other focus of the ellipse to put the exact same star, the shape of the orbit does not change,
because it will be correct from the point of view of the first stars, and the second." Correctly argues Dunno? If his reasoning there is an error,
specify which ones.
12. Someone suggested a new way to launch rockets. Rather than run them up, it is recommended to let down the missile guides (rails)
forming a large semicircle of radius R, arranged in the vertical plane. At some point, the movement along the guide should include the
engine. The author claimed that with this launch height H 2 lift rocket will be greater than the height H 1 , achievable under normal startup
(vertically upwards with the same engine).
a) (9-11) In finding missiles exactly where gutters should include the engine to the height H 2 lift rocket was the highest?
b) (8-11 grades) Where does the extra energy to height increases?
Physics - solutions
1. We define the time during which the column will unfold, t. e. the time between meetings with the first athlete to coach and coach of the
meeting with the last athlete. After the meeting with the first athlete to coach the coach and the last athlete running towards each other at
speeds of 15 km / h and 20 km / h, respectively, the initial distance between them - 70 m. Hence, they meet in a time T = 70 m / s (15 km / h +
20 km / h) = 70 m / s (35 km / h). During this time, the coach will flee from the venue to the first athlete to a distance T * 15 km / h = 70 m / s
(35 km / h) * 15 km / h = 70 m * (3/7), and the first athlete - to T * 20 km / h = 70 m / s (35 km / h) * 20 km / h = 70 m * (4/7). The difference
between these distances m * 70 (4/7) - m 70 * (3/7) = 70 m ((4/7) - (3/7)) = 70 m (1/7) = 10 m and has the length of the column after the turn.
2. The difference in size of droplets. The vertical component of the force acting on the droplet, the sum of components due to the counter air
flow (directed upward), and the force of gravity (directed downward). The first component is approximately proportional to the square of the
drop (in fact cross-sectional area drops perpendicularly to the flow of air, but it is not very important), and the second - the mass, and hence
the volume. That is the first grows as the square of the linear dimensions, and the second - as a cube. It is clear that there exists such a
machine speed (hence the oncoming flow vohduha), in which large drops slide down, and small creeping up.
Droplets falling from the top of the can for a while by inertia to move down, and then by the force of friction on the flow of air, turn around.
Many students wrote that the front of the windshield of a moving car chaotic turbulent eddies are created randomly drops move in different
directions. Such a response can not be considered entirely correct. All modern cars are designed in such a way as to avoid the formation of
these vortices, otherwise, for example, operation of the vehicle in the winter time would be impossible - Windshield immediately closed up
with snow.
3. The laser consists of an active emitting medium bounded by parallel transparent and semi-transparent mirrors. This one saw the reflection
of a fragment of his eyes in a transparent mirror. Since the laser beam and, therefore, the lens has a small diameter, it can be seen a fragment
of the eye, the image is not easy to guess about it. In many lasers are used mirrors, designed only for a certain wavelength of light (the one on
which emits laser). Therefore, the condition, and clarified that the color of the laser - red, it means that it mirrors reflect light in the optical
range.

4. The boat will move "downstream" along the pipe (parallel to its axis). Obviously, the water located near the walls will flow parallel to the
walls ("more it has nowhere to go"). For the same reason the layers of water that are a little further from the walls will move parallel to the
layers, immediately adjacent to the walls, and so on. D. In the condition specifically stipulated that the water released slowly, otherwise may
appear vortices and the movement of water and the boat will be much more complicated .
5. When the distance between the magnifying glass and notebook is 0, the loop does not have any influence on the image and we see it in full
size. (Really, t. To. Magnifier has a thickness if its put on exercise book, we see him with a slight increase). Then, as the distance from the
notebook magnifying glass magnification increases, then, at some point we cease to see the image at all, then there is a reduced inverted
image, as we approach the magnifying glass to the eye, it becomes more and more reduced.
6. Cephas Mokiyevich made methodological errors in the formulation of their experiment and correctly interpret the results.
The thing is that the thermocouple generates an electrical current (voltage on it and we judge its temperature) due to the thermal energy of the
medium in which it is located. Therefore, it is impossible with thermocouples to monitor the temperature of heat-insulated objects with small
heat capacity - as a result of their temperature may change significantly. Or the amount of heat absorbed by a thermocouple should be
considered when interpreting the results of experiments.
7. On the instrumentation (including beakers) scale is applied so that the absolute error of measurement was equal to half the price of division
Allocated scale. This very error occurs for many reasons, including of course there are errors of observation scale, but the cause is usually not
the primary. Beaker for example, more significant effects on the capillary walls and geometrical defects vessel from which it is made
beaker. In some cases, the measurement still can be performed with a smaller error than the one on which the instrument is designed. For
example, if we are in a beaker with water dripped a drop of water, and saw that the water level rose by less than 1/4 of the division, it can be
argued that the droplet size was actually less than 1/4 of the scale interval. But if, for example, we dolili into a beaker of water and the level
rose to 10.25 divisions, the claim that the amount of water topped up was equal to 10.25 scale value, you can not. The last division we are still
on the eye can accurately be divided into parts, but previous 10 divisions also contributed to the error, which together can be higher than 0.25
price division.
All of the above applies not only to the beaker, but also to many other measuring instruments. During the writing of this text on the table of
the Tournament jury happened to be package with new wooden ruler length of 50 centimeters (at least as written on them). Applying them to
each other, we found that the length scales they have almost the same (perhaps in fact equal to 50 cm), but in this package easily able to find
such a pair of lines that the opposite division "25 cm" was one line division "of 25 cm to 1 mm" another line. There was even found a pair that
has coincided division "25 cm" and "25 cm 2 mm." With such a range, perhaps, it may be concluded that the length of any segment lies
between 3.1 mm and 1.2 mm, but it can not be asserted that the length lies between 25 mm and 1/3 cm 25 cm 1 / 2 mm.
8. Raspryamlnnaya spiral has better heat dissipation than conventional (adjacent turns warmed to each other). Resistance of metals increases
with increasing temperature (at least "normal", of which make the helix electrotiles and specific to electrotiles temperature ranges).
Consequently, for the same resistance to conventional current flowing helix would be greater since, because of the lower heat transfer, its
temperature will be greater. However, most of the conditions of the problem, it follows that the same was the voltage applied to the coil, rather
than the current (if in the first and in the second case the spiral was connected to the same power source, for example, to the grid). Easy to see
that in this case the solution remains true. Assume the contrary (that swirling spiral resistance is less than or equal to the resistance
raspryamlnnoy). Then the temperature of swirling spirals will also be more (heat power, increases with decreasing resistance, not less, and
the heat transfer worse). Consequently, the resistance increasing with temperature must be greater than - contradiction.
9. The water level will be greater in that tube, wherein the bubble pops. Assuming that the bubble is fixed, the solution is obvious - each of
the water pressure at the bottom of tubes should balance each other, as well as the average density of the contents of the tube with the bubble
(air + water) is less then the volume (and hence the water level ) must be greater.
If the bubble pops up with a steady rate, it does not change the situation as compared to the case considered above. If it is because of gravity
moves upwardly with a positive acceleration (i.e., center of mass located in the tube system water + air moves down to the positive
acceleration), the pressure of the system at the lowest point is even lower (in comparison with the case where bubble
stationary). Consequently, the level will be even more.
We assume that the bubble is symmetrical - the upper and lower its borders have the same shape, therefore, the capillary forces acting on
these boundaries, cancel each other and therefore do not affect the levels of water in the tubes.
Theoretically, if the bubble will move up with large negative acceleration, this may alter the response to the opposite problem. But it is
obvious that if the bubble just let go from the bottom, it is all the time will move up with positive acceleration and speed will tend to
equilibrium. A brake force bubble, pre-disperse it, the jury could not - prevent the friction of water on the walls of the tube, which creates
turbulence and bubble breaks up into pieces, and there are variations in the levels in the tubes. Therefore, the situation described in this
paragraph, can not be considered.
10. The linear velocity of the surface of the second rod (in the reference frame where its axis is fixed) is equal to v 2 = 2 r 2 . In place of the
rod coupling component this velocity perpendicular to the axis of the first rod (and equal to 2 r 2 cos ) is passed to the surface of the
first rod (m. k. around its axis can rotate freely), and the parallel component is lost (occurs slippage t. To. along its axis first bar can not

move). Thus, the linear surface speed of the first shaft is v 1 = 2 r 2 cos , and the angular velocity of its rotation
respectively, 1 = v 1 / R 1 = 2 ( R 2 / R 1 ) cos .
11. Note that you need to find is an error (at least one) in the argument Dunno, not fix all of them, that the student - a very difficult task. One
of these errors - Dunno rightly believes that around each star individually comet will run on the same path , but in each of these cases, the
speed of the comet at different points of the trajectory is different (according to the law of conservation of energy the sum of potential and
kinetic comet energy is constant, and since changing the distance to the star, must change and speed). Thus, the velocity at each point of the
comet path determined by the distance to the star, so despite the fact that the path of movement around each individual stars on the same
speed at each point of the comet trajectory (except two - the intersection points of the ellipse with the perpendicular bisector of the segment
joining his tricks) will vary depending on what kind of stars around the comet rotates. Therefore, the rotation of the comet around the two
stars at once will not be overlaid movements, as mistakenly believes Dunno.
However, the imposition of two concurrent (trajectory and speed at each point of the trajectory) movements do not always give the same
movement. For example, suppose that the planet orbits the star in a circular orbit."Add" another same star, t. E. In fact, increase the mass of
stars available in 2 times. It is easy to see that after this planet will not be able to rotate on the same orbit at the same speed.
12. a) The jury was specifically not hard to formulate a condition. In particular, it says nothing about the engine is running. The simplest
solution to the problem (showing the main idea) is obtained by assuming that the engine is run for a short period of time.
It is obvious that where we have not included the engine, it will increase the speed of the rocket at the same value (denoted by v ), which is
independent of the initial velocity v 0 (to see this, just go to the reference frame in which v 0 = 0). We also assume that the mass of spent fuel is
small enough, then there are lots of rockets M during operation of the engine remains virtually unchanged. The difference of kinetic energies
before and after the rocket engine actuation E kin = ( M ( v 0 + v ) 2 / 2) - ( M v 0 2 / 2) = ( M / 2) ( v 0 2 +2 v 0 v + ( v ) 2 - v 0 2 ) =
( M / 2) (2 v 0 v + ( v ) 2 ) will be the greater, the larger v 0 . A v 0 is maximal at the lowest point of the semicircle.
b) Additional ("extra"), the kinetic energy of the missile is obtained at the expense of the potential energy of the fuel: the normal start that is
emitted at ground level, and in the case of the described embodiment - below this level.

Chemistry
You are invited to solve the problem of three or four electives. It is desirable, but not required, to choose the tasks intended for the class in
which you learn, or older.
1. Fill in the blanks (instead of the substance ... and instead of coefficients
7-8 class.
1) ... = CaO + ...
2) 3 ... + N 2 = Ca 3 N 2
3) ... + 2H 2 O = H 2 SO 4 + 2HCl
4) CaH 2 + H 2 O = Ca (OH) 2 +
5) ... = 2CuO + 4NO 2 + O 2
9-11. ( see explanation )
1) 1 ... 5 ... + = + 3 ... 4 ...
2) 4 ... + 11O 2 = 2 ... + 8SO 2
3) 3Cl 2 + 6 .. . = 5KCl + ... + H 2 O
4) ... + 6H 2 O = Mg (OH) 2 + 2NH 3
5) 3 + ... ... = 3HCl + H 3 PO 4,
6) FeSO 4 + KMnO 4 + H 2 SO 4 =

) in the equations of chemical reactions.

...

MnSO 4 +

... + K 2 SO 4 +

H2O

2. (7-9 grade) Chemical analysis showed that the mass fraction of sodium in the matter of (58 + 2)%. Suggest formulas of substances that
satisfy this condition.
3. (7-9 grade) It is known that the mass of protons and neutrons are expressed in whole numbers and electron mass is negligible. However, the
atomic weight of which are provided for the elements in the periodic table are nearly always fractions. How can this be explained?
4. (8-9 grade) marbles (CaCO 3 ) was placed in 100 g of hydrochloric acid solution with a mass fraction of 7.3%. After some time, the amount
of HCl in the solution was halved and the diameter of the ball is also halved.Determine the initial mass of the ball.
5. (class 10-11) 4.37 g of the alloy of the two metals dissolved in water. THe 7.84 liters of gas (STP). Upon careful acidification of the
solution obtained precipitate, which is dissolved in an excess of acid. Determine the qualitative and quantitative composition of the alloy.
6. (8-10 grade) Heavy water - water containing hydrogen isotope deuterium, D 2 O. Rate the density of heavy water.

7. (grades 9-11) Young Chemist Misha found three sealed vials with a shiny silvery liquid. He opened the vial and poured water into
them. Fluid from the first vial not react with water. Misha then heated it in a flask filled with oxygen and saw that this image of a red
crystalline solid. The contents of the second vial vigorously react with water to yield a colorless flammable gas. After completion of the
reaction vial was left on the bottom of the liquid silver indistinguishable in appearance from the original and chemical properties - on the
contents of the first vial. After Misha poured water into the third vial, there was a loud explosion with a flash of flame yellow with purple
highlights. What is in ampoules and what gross violations of safety allowed the young chemist?
8. (10-11 class) What is the structure may have a connection of C 3 H 3 Cl 3 ? Draw all the possible isomers.
9. (9-11 grade) Put the above formulas of chemical substances in accordance with their areas of application specified in the table. (The
solution can be written as "the number-letter").
1. AgBr

Abrasive

2. BN

chemical warfare agent

active agent in insulating masks

baby powder

Detergent components

component powders

filler bulbs

based on artificial gemstones

pigment oil paints

And

mordant in dyeing fabrics

photosensitive material (photo)

12. S

construction material

13. ZnO

Fixer (Photo)

Coolant

Oh

3. C

12

4. CaSO
5. CCl

25

Na

* 0,5H

SO

6. ClCN
7. KAl (SO

) 2 * 12H

8. Kr
9. Na

10. Na

S2O

11. Pb

14. ZrO

2
3

* 5H

10. (class 10-11) at full catalytic hydrogenation mixture propadiene, propylene, 1,4-pentadiene, 1-viniltsiklopentena absorbed hydrogen
amount equal to half of the volume of carbon dioxide (measured at the same conditions) produced during combustion of the same amount of
mixture. Determine the volume fraction of propadiene in pairs starting mixture.
Chemistry - Solutions
1. 7-8 class.
1) CaCO 3 = CaO + CO 2 ; or Ca (OH) 2 = CaO + H 2 O, and so on. d.
2) 3Ca + N 2 = Ca 3 N 2 .
3) SO 2 Cl 2 + 2H 2 O = H 2 SO 4 + 2HCl
4) CaH 2 + 2H 2 O = Ca (OH) 2 + 2H 2
5) 2Cu (NO 3 ) 2 = 2CuO + 4NO 2 + O 2
9-11.
1) C 3 H 8 + 5O 2 = 3CO 2 + 4H 2 O
2) 4FeS 2 + 11O 2 = 2Fe 2 O 3 + 8SO 2
3) 3Cl 2 + 6KOH = KClO 3 + 5KCl + 3H 2 O
4) Mg 3 N 2 + 6H 2 O = 3Mg (OH) 2 + 2NH 3
5 ) POCl 3 + 3H 2 O = 3HCl + H 3 PO 4,
6) 10FeSO 4 + 2KMnO 4 + 8H 2 SO 4 = 2MnSO 4 + 5Fe 2 (SO 4 ) 3 + K 2 SO 4 + 8H 2 O
2. Mass fraction solution (58 + 2)% means that the molar mass of compound NaX X should be from 23: 60 * 40 = 15.3 to 23: 56 * 44 = 18,
and in conjunction Na 2 X X molar mass ranges from 30.3 to 36.
Examples of compounds that are suitable: Na 2 O 2 , Na 2 S, NaOH, NaNH 2 , NaBH 2 D 2 .
3. Many of the elements may exist in the form of several isotopes (m. e. atoms of the element can contain different numbers of
neutrons). Isotopes differently distributed in nature. The experimentally determined atomic weight - a weighted average for all natural
isotopes of a given element. It is these values and are listed in the periodic table, so they are fractional.
There is a second reason - the so-called mass defect, that is, weight reduction that accompanies the formation of the nucleus of protons and
neutrons (mass equivalent of the energy that is released during the formation of the nucleus, E = mc 2 ).
4. A solution of hydrochloric acid containing 7.3 g of HCl, which represents 7.3 / 36.5 = 0.2 mol. In response entered half of this amount, i.e.
0.1 mol. Reaction equation CaCO 3 + 2HCl = CaCl 2 + H 2 O + CO 2 shows that while expending 0.1 / 2 = 0.05 moles of calcium carbonate.

Bead Diameter halved, then its volume and hence weight decreased in 2 3 = 8 times as V = (4/3) R 3 . In other words, 7/8 of the original
amount of reacted calcium carbonate. Thus, the initial amount of CaCO 3 0.05 7 * 8 = 0.057. M R = 100. The weight was 100 * 0.057 = 5.7 g
5. 4.37 g of the alloy to form a reacted, 7.84 / 22.4 = 0.35 mol gas. It is easy to see that we are talking about a very light metal, as the average
equivalent metal in the alloy is 4.37 / (0.35 * 2) = 6.24. In this unidentified metals must be active, since the alloy dissolved in
water. Obviously, one of the metal lithium. Equivalent (and molar mass) 7. Hence, the equivalent of the second metal should be even
smaller. This condition is satisfied beryllium. Its molar mass 9, but it is bivalent, so is the equivalent of 4.5. Beryllium is not soluble in water,
but dissolves in alkaline LiOH, formed by dissolving lithium. Now you need to find the quantitative composition of the alloy.
Let the mixture in the x and y mol Li mole Be.
Upon dissolution of the alloy is released (0,5x + y) moles H 2 .
7x + 9y = 4,37; 0,5x + y = 0,35. From x = 0,49 mole. y = 0,105 mol.
6. The density of heavy water can be estimated as follows: it is obvious that the amount of water molecules has not changed in comparison
with the volume of H 2 O. Means a unit volume (e.g., 1 cm 3 ) contains the same number of molecules. However, the increased mass of each
molecule (16 + 2 + 2) / (16 + 1 + 1) = 20/18 = 1.11 times.
Since the density of H 2 O 1 g / cm 3 , the density D 2 O 1,11 g / cm 3 .
7. The first vial was mercury, the second amalgam (mercury alloy) any alkali metal (e.g. sodium), and the third - the alloy of sodium and
potassium, which at room temperature is liquid. Tip is the color of the flame (yellow with purple).
With regard to safety regulations, frequent answers that nothing is impossible to pick up on the street, and it is impossible to conduct
experiments with unknown substances are certainly correct rules of behavior for students.However, safety in chemistry is somewhat
different. The fact that the chemistry is not impossible to carry out experiments with unknown substances. Just for such experiments must be
carefully selected small quantities of substances and be sure to wear protective goggles and mask better, and even better to conduct
experiments under the hood.
8. Linear isomers:
CH 3 -CCl = CCl 2
CH 2 Cl-CH = CCl 2
CHCl 2 -CH = CHCl (2 geometrical isomers)
CCl 3 -CH = CH 2
CHCl 2 -CCl = CH 2
CH 2 Cl-CCl = CHCl (2 geometrical isomers)
Cyclic isomers:
Cl H Cl Cl
\ / \ /
CC
/ \ / \
Cl-C --- CH HC --- CH
/ \ / \
H Cl Cl H
The first of these molecules can exist as two geometric isomers, depending on how directed chlorine atoms relative to each other. For a
second molecule can exist two optical isomers (ie. K. C atom linked to one chlorine atom and one hydrogen atom, is an asymmetric).
9. Solution:
1.

AgBr

photosensitive material (photo)

2.

BN

Abrasive

3.

Detergent components

4.

CaSO

construction material

5.

CCl

Refrigerant

Oh

6.

ClCN

chemical warfare agent

7.

KAl (SO

mordant in dyeing fabrics

8.

Kr

filler bulbs

9.

Na

active agent in insulating masks

12

SO

Na

* (1/2) H

25

) 2 * 12H

10.

Na

S2O

Fixer (Photo)

11.

Pb

pigment oil paints

And

12.

component powders

13.

ZnO

baby powder

14.

ZrO

the basis of artificial gemstones

* 5H

10. Links: C 3 H 4 , C 3 H 6 , C 5 H 8 , C 7 H 10 . Let the mixture comprises x, y, z and k mole of these substances, respectively.
Substan The number of moles The number of moles of H 2 for The number of moles of CO 2 emitted
ce
of the substance
complete hydrogenation of substance during the combustion of the substance
C

2x

3x

1y

3y

2z

5z

10

3k

7k

Condition of the problem allows to make the equation 2 (2x + y + 2z + 3k) = 3x + 3y + 5z + 7k here xyzk = 0 or x = y + z + k. Number
propadiene mixture (respectively, and the volume fraction in the vapor) 0.5 or 50%.
Explanation of the condition of the problem 1 for grades 9-11
In the context of this task, students handed out during the tournament, the jury has admitted as much as 3
errors. 1. instead of reacting 4) ... + 6H 2 O = Mg (OH) 2 + 2NH 3 was printed 4) ... + 6H 2 O = Mg (OH) 2 +
NH3 (coefficient omitted NH 2 before 3 , without it is not possible to equalize the response) 2. , instead of
reacting 5) 3 + ... ... = 3HCl + H 3 PO 4 should be 5). .. + 3 ... = 3HCl + H 3 PO 3 (H 3 PO 3 instead of H 3 PO 4 ) It is
understood the answer 5) PCl 3 + 3H 2 O = 3HCl + H 3 PO 3 As a result of this error, the problem remains correct,
but has become more difficult. 3. instead of reacting 6) FeSO 4 + KMnO 4 + H 2 SO 4 = MnSO 4 +... +
K 2 S0 4 + H 2 O was printed 6) FeSO 4 + KMnO 4 + H 2 SO 4 = MnSO 4 + ... + H 2 O (skipped K 2 S0 4 on the right
side of the equation)
Competition in Mathematics
Tasks
In parentheses after the number of tasks specified classes for which the recommended task. To solve problems not of their class is allowed.
1. (6-9) Can the arithmetic mean of 35 integers equal to 6.35?
2. (6-9) Peter bought a "Designer", which was 100 sticks of different lengths. The instructions to the "designer" is written that any of the three
sticks "designer" can make a triangle. Peter decided to test this assertion, made up of sticks triangles. Sticks lie in the constructor ascending
lengths. What is the minimum number of inspections (in the worst case) should be done Pete, to prove or disprove the assertion manual?
3. (10-11) There are 10 segments, with known that the length of each - a whole number of centimeters. Two of the shortest segment centimeter, the longest - 50 cm. Prove that there are three segments, from which you can make a triangle.
4. (6-11) from the point M inside the quadrilateral ABCD perpendiculars to the sides. Grounds perpendiculars lie within the parties. We denote
these grounds: that which lies on the side AB - through X , lying on the side BC- through Y , lying on the side of the CD - through Z , lying on
the side of the DA - through T . It is known that AX > XB , BY > YC , CZ > ZD , DT > TA . Prove that the quadrilateral around ABCD can be
described as a circle.
5. (6-11) The surface of the Rubik's Cube 3 * 3 * 3 consists of 54 cells. What is the maximum number of cells can be noted so that the marked
cells had no vertices in common?
Problem solving in mathematics competition
1. Answer : this can not be.
Assume that there exist numbers. Their sum is equal to the arithmetic mean of these numbers is multiplied by their number:
6.35 * 35 = 222.25.
Since the sum of the integers must be an integer, we obtain a contradiction.
2. The answer : one check.

Pete is enough to check whether you can make a triangle from the two most short rods and one of the longest. If the triangle is not made, the
statement refuted instructions. If you can draw a triangle, the sum of the lengths of the shorter of the two rods of the longest length of
more. But in this case, the sum of the lengths of any two rods long as any other set. (Indeed, the sum of the lengths of any two is not less than
the sum of the lengths of the shortest and the length of each rod is not greater than the length of the longest.) This means that any of the sticks
can make a triangle, t. E. The assertion proved instructions.
3. Assume the contrary, that from any of the three segments can not make a triangle. Consider the lengths in centimeters Ascending: L 1 =
1, L 2 = 1, L 3 , L 4 , ..., L 10 = 50. Since the shortest of the three segments can not make a triangle, then l 3 > L 1 + L 2 = 1 + 1 =
2. Similarly, L 4 > L 2 + L 3 > 1 + 2 = 3. Further,
l 5 > 2 + 3 = 5 l 6 > 3 + 5 = 8 l 7 > 5 + 8 = 13 l 8 > 8 + 13 = 21 l 9 > 13 + 21 = 33 l 10 > 21 + 33 = 55

Contradiction with the condition l 10 = 50 proves the assertion.


4. From the condition AX > XB should AM > MB . Indeed, in two triangles AMX and BMX with a common hypotenuse of the leg from the one
in which the length of the second leg. Similarly we BM > MC , CM > MD ,DM > MA . This is possible only if all four inequalities, the
equality: MA = MB = MC = MD . Hence, M - Center circumscribing quadrilateral ABCD circle, which proves the required result.
5. Answer : 14 cells.
Fig. Figure 1 shows how to mark 7 cells on three adjacent faces of the cube. On three "invisible" faces should be noted seven cells, these
symmetrical.
We now show that more than 14 cells as desired can not be mentioned. We do this in two ways.
The first method. Calculate the total number of vertices of the cells Rubik's Cube. There are eight vertices of the cube, another two vertices on
each of the rib 12 and still on the top 4 of each of six faces. Total:
8 + 12 * 2 + 6 * 4 = 56 vertices.
Each of these peaks is not owned by the condition of more than one marked cell. If the selected cells was greater
than 14, then the peaks would be greater than 4 * 14 = 56, since each cell has four vertices. Hence, the cells
selected to 14.
The second way. We cut the surface of the Rubik's Cube apart. Three adjacent faces we cut into 7 parts, as shown in Fig. 2. The three other
faces we cut to the same 7 parts. It is easy to see that any two cells, trapped in one part have a common vertex. Therefore, in each portion may
be not more than one marked cells. Hence, it can be noted in all no more than 14 cells.

Fig. 1

Fig. 2

Assignments
After the job number in parentheses indicates how classes, this problem is recommended. Works of the students grade 6 and younger are
measured at two , 7-8 classes - for three , grades 9-11 - on four best Solved problems. Solve more problems and challenges over its class - it
is possible. Solved problems class under its estimated much lower.
1. (6-8) During a thunderstorm, the distance from itself to the place where lightning struck, can roughly determine in this way.

As soon as sverknt lightning, start counting the seconds: "One, two, three, four, ...", as long as the thunder. Last called number is divided by
3 - Get the approximate distance from the observation point to the point of a lightning strike.
Based on what this method and why it is necessary to divide it into 3? Why this method is not very accurate? (List the main reasons for this.)
2. (6-8) In Moscow, a certain section of the Garden Ring (the length of 1 km), the average speed of vehicles in the right lane - 5 km / h, in the
middle two - 6 km / h, in the left - 7 km / h. In each row, the same number of machines placed. Find the average time during which the cars
pass this part of the Garden Ring.
3. (7-8) The passenger said that a cup of tea in the dining car of the train while driving cools faster than at the stops. A physical explanation of
this phenomenon.
4. (6-11) A clear spring day in Moscow went fine drizzle. Build a rough (qualitative) dependence of the concentration in the air poplar down
on the time since the start of a rain. Explain your proposed solution.
5. (9-11) The trolley rides on the horizontal surface of the earth with constant acceleration. On the trolley has two racks of the same height
(the segment connecting the base rack, parallel to the direction of motion). On these racks hanging piece of the chain (see. Fig.). A drawing to
determine the direction of motion and acceleration of approximately A , which is moving truck. Describe the method of determining the
acceleration pattern and lead its rationale. Read G = 10 m / s 2 .

6. (8-11) between terminals A and B are connected in parallel two identical chains of three series-connected resistors R , 2 R and R . Parallel to
each of the two resistors R connected two similar chains - R -2 R - R -, and themselves resistors 2 R , which is connected in parallel chains,
pointed (to avoid confusion). Parallel to each of the unmarked resistors 2 R again connected by two chains - R -2 R - R -, and themselves
resistors 2R, which is connected in parallel chains noted. This operation was repeated an infinite number of times. Find the
resistance R AB between terminals A and B .
7. (9-11) between the smooth stool and chair of the same height (horizontal) is the length of the board L mass M (weight is evenly distributed
along the length). The coefficient of friction between the board and the chair , between the board and stool - 0. stool sits mouse
mass m . Seeing a cat, a mouse runs across the board from the stool on the chair, hoping their actions so as to be in the chair as soon as
possible and do not fall down the board, based on the very edge of the chair. Find a job A , committed during perebeganiya mouse on the
board. Power mouse is not limited to, its paws on the board do not slip.
8. (10-11) On a clear sunny day Sherlock Holmes sat by the window, looking at the headline in your favorite big glass magnifier. It turned out
a larger image in the middle of playing a magnifying glass with almost no distortion, and the edges along the borders of the black letters
visible colored stripes: yellow from the edge of the magnifying glass and blue - from its center.
Explain the observed phenomena and fill (all or some) gaps in the table (please, why did you choose this color):
The object of observation (on Color
stripe
along
near to
white paper).
center of the image magnifier.
Black block letters.

Blue.

the Color stripe along near the edge of the


image magnifier.
Yellow.

Line from red felt-tip pen.


The line from the green felttip pen.
The line from
ballpoint pen.

dark

blue

Yellow (between the yellow stripe and


image - the thin red stripe).

(If you have a proper full explanation of the task is considered regardless of the solved correctly filled table, otherwise the table is taken into
account.)
Answers and solutions to the competition in physics
1. (6-8) During a thunderstorm, the distance from itself to the place where lightning struck, can roughly determine in this way.
Crazu as sverknt lightning, start counting the seconds "one, two, three, four ..." as long as the thunder. Last called number is divided by 3 Get the approximate distance from the observation point to the point of a lightning strike.
Based on what this method and why it is necessary to divide it into 3? Why this method is not very accurate? (List the main reasons for this.)
Solution. After the lightning flash light reaches us almost instantly (the speed of light C = 299,792.458 km / s), and the speed of sound in the
air
at a temperature of 0 O C - approximately 331 m / s = 1 / 3,021 km ... / c = about 1/3 km / s
at 20 O C - about 343 m / s = 1 / 2.915 ... km / sec = about 03.01 km / s.
In the temperature range of air 0 O C - 20 O C (during thunderstorms air temperature is usually in this range) we can assume that 1 second
sound travels 1/3 km, 3 seconds - 1 km. Therefore, to know the number of kilometers, the number of seconds to be divided by 3.
Causes of errors:
1) inaccurate measurement period ("one, two, three, ...");
2) to use an approximate value of the sound velocity;
3) the velocity of sound in air depends on temperature, humidity, the presence of water droplets, etc. causes which may vary (especially
during a thunderstorm);
4) in the place where the lightning channel shock waves that propagate through the air faster than sound, and at some distance (not yet
damped and not turn into sound) they determine the time after which the thunder thunder;
5) sound occurs not only at the site of lightning into the ground, but also along the entire length of the channel of an electrical discharge,
which may be several kilometers - first, we can hear the sound come from some intermediate point, not from the place hit in the ground;
6) the sound of the place of lightning into the ground, we can not hear at all (for example, when lightning struck into the woods or elevation),
and only hear thunder, come down to us from some other point of lightning, possibly more, and not in a straight line, as a result of multiple
reflections.
If a more accurate estimate would be that the reasons 1 and 2 are not the main, that is. E. The increase in accuracy of timing and use in
calculations more accurate (than 1/3 km / s) velocities of sound in air does not increase the accuracy of the final result . But in order to see the
storm approaching us, or, conversely, removed - described method is quite good.
2. (6-8) In Moscow, a certain section of the Garden Ring (the length of 1 km), the average speed of vehicles in the right lane - 5 km / h, in the
middle two - 6 km / h, in the left - 7 km / h. In each row, the same number of machines placed. Find the average time during which the cars
pass this part of the Garden Ring.
Solution. The arithmetic mean of the times of passage of this area in each of the rows t cp = (1/4) ((1/5) 2 (1/6) + (1/7)) h - the wrong
answer. The fact that the higher the speed of motion in a row, the number of machines bo'l`shimi thereon for a certain time. Note (easy to
check) that one row of cars passing by as many times greater than in another, how many times the speed of movement on it more than
others. Therefore, in calculating the arithmetic mean of the terms in the numerator and the denominator is multiplied by the coefficients so
that the appropriate number of cars this term accounted for correctly:
t cp = (1 / (1 + (6/5) * 2 + (7/5))) ((1/5) + (6/5) 2 * (1/6) + (7/5) ( 1/7)) = h
= (1 / (24/5)) ((1/5) + (2/5) + (1/5)) h = (5/24) (4/5) = h h 1/6 = 10 min.
3. (7-8) The passenger said that a cup of tea in the dining car of the train while driving cools faster than at the stops. A physical explanation of
this phenomenon.
Solution. As a result, the vibration of the train while driving surface of tea in a teapot it becomes uneven, increasing its area, which increases
the evaporation rate at which thermal energy is spent fluid (ceteris paribus conditions (temperature, humidity), the amount of liquid that
evaporates per unit time is proportional to the surface area).
Also, do not forget that while shaking the glass wall all the time wetted and tea with them too evaporates, the surface area is comparable to
wet the walls of the surface area of tea.
Shaking also affects the convection currents inside the cooling tea and air flows over its surface (moving "wave" on the surface "chase"
air). This effect may lead to an increase in the intensity of cooling (due to an increase of evaporation (from the surface) and cooling through
the walls due to the strong mixing of the liquid), and vice versa - as a result of vibration can be destroyed convection currents.
4. (6-11) A clear spring day in Moscow went fine drizzle. Build a rough (qualitative) dependence of the concentration in the air poplar down
on the time since the start of a rain. Explain your proposed solution.

Decision. The main reasons for feather may stop flying in the air:
1) a direct hit in a feather drop of rain or splashing droplets formed after hitting the ground;
2) an increase in mass and density fluff because of increasing with time (after the start of the rain) humidity, because of what the fuzz can
"land";
3) the movement of fluff down in a stream of air, was fond of raindrops, and then a feather may stick to the wet ground, or asphalt surface
puddles, or can be hit by spray at the surface of the earth;
4 ) fluff should have an electric charge (as any small particles floating in the air - such as dust adhering to this reason, the screen of the TV or
computer). At first glance it seems that, once all the poplar fluff about the same and are in the same conditions, they should have
approximately the same electrical charge and therefore repel each other, helping them to "hang" in the air.
In fact, most likely things work "smarter". For example, also seems to be the same raindrops [ 11 Here are the data for the strong (lightning) of
rainfall. ] charged in different ways - there are as negative (their average charge of -1.3 10 -12 cells), and positive (average charge + 1.1 * 10 12
TC) charged droplets, with positively charged droplets on average 1.5 times more than the negatively charged. The average electrostatic
potential of such drops - 40, maximum - 300 V (voltage in the household electric system, we recall, only 220). Charge drops "small blue
moon" less - an average of 10 -15 Cl - 10 -14 Cl, maximum - 5 * 10 -13 Cl.
Charge distribution pushinok likely also fairly complex. Data on the distribution of charges in the poplar fluff and even the approximate value
of the charge fluff we could not find (probably no physics or biophysics, biology or seriously not interested in this issue).
We can assume that a feather like snowflakes (average charge of 10 -12 Cl - 10 -11 Cl, maximum 5 * 10 -11 Kd). If these assumptions are correct,
the main effect of the presence of rain will just decrease the electrical resistance of the atmosphere, which may lead to flow of charges and
reduction of the stability of fluff (lint-concentration snowflakes and raindrops roughly the same, and the charge-fluff snowflakes
approximately 3 orders of magnitude (in 1000 times) more than raindrops, so the electrostatic interaction with raindrops fluff can be ignored).
In the rain can be seen an interesting paradox: the asphalt is completely wet (ie. E. A drop of rain has blocked the entire area of the air), and
the down is still flying (although in theory all the fluff has to be "shot down" drops).Pooh flying even after a time, 2-3 times more than when
it started to rain after the asphalt completely gets wet (Amendment to the fact that the diameter of the "blob" of drops on the pavement the
diameter of the droplets in flight). The point here is that the movement of raindrops in the air (to the point where it will be about the earth at
an altitude at which flies poplar fluff) is established, the air flow around the droplet streams, removing her way with a feather. But the
movement of the spray from the earth to establish not have time and they "knock" fluff is much more effective.
5. (9-11) The trolley rides on the horizontal surface of the earth with constant acceleration. On the trolley has two racks of the same height
(the segment connecting the base rack, parallel to the direction of motion). These racks hanging piece of chain (see. Fig. P. 78 ). A drawing to
determine the direction and approximately acceleration A , which is moving truck. Describe the method of determining the acceleration
pattern and lead its rationale. Read G = 10 m / s 2 .
Solution. The trolley travels from left to right (because that chain of "slack in the opposite direction"). Note that in fact correct to say that the
acceleration is directed from left to right (and the speed can be directed in the opposite direction - in this case, the truck moves
ravnozamedlenno).
Determine the numerical value of the acceleration a , you can use this algorithm:
1) put on the drawing line (or straight edge of the paper) so that it passes through the ends of the chain (the chain attachment points to the
posts);
2) to turn the line around the left end of the chain in the direction of clockwise until the line until the uncovered part of the image itself and
the chain line will not restrict shape having an axis of symmetry (perpendicular to the line);
3) build this symmetry axis (perpendicular bisector of the segment joining the points of intersection with the line of the chain);
4) noted on this axis arbitrary segment, to build on it as on the hypotenuse of a right triangle, one leg of which is vertical, and the other horizontal;
5) to find a proportion of
(Length of the vertical leg) / (length of the horizontal leg) = G / A .
Justification. [ vectors are designated with an arrow on top and bold ] It is known that the motion of the system in the gravity
field g acceleration a is equivalent to [ 12 Under the assumption that the rate of non-relativistic system, that is much less than the speed of
light C ] for finding the system its (uniform rectilinear motion at any speed c [ 13 much less than the speed of light C ], in particular, rest) in a
gravitational field G - A . Direction g is known (vertical) - means (in the selected scale) | G | = length of the vertical projection of segment =
length of the vertical leg.
Mentally, we fix the chain at the point where it intersects the illustration with a ruler. Obviously, the shape of the chain will not change (the
system is in equilibrium before and after will be). Now we have hung on the ends of a piece of chain that has a symmetrical shape. That's
what should hang a piece of string in a gravitational field, parallel to the axis of symmetry.
Thus, the axis of symmetry - is the direction vector of the difference G - A . We choose this direction interval and assume that we have chosen
the scale so that its length is equal to | G - A |.
The direction of a horizontally condition - "trolley rides on the horizontal surface of the earth with constant acceleration. ", so

| A | = length of the horizontal leg. Therefore, the ratio of the lengths of the vectors | G | / | A | is the ratio of the lengths of the legs, that is true
proportion
(Length of the vertical leg) / (length of the horizontal leg) = G / A ,
what was required to prove.
Help (this information is not necessary to solve the problem). The curve shape which is hanging in the gravity field uniform chain, is the
graph of "hyperbolic cosine" (It is important that the chain bends anywhere without any resistance, to rope it will not be so suspended form
the ends of a piece of rope will be different). Another name for the graph of "hyperbolic cosine" - Catenary (t. E. A hanging chain line):
( e x + e - x ); e = 2.718281828459045 ...
It is assumed that the coordinate axis x is directed horizontally, the axis y - vertically. If the ends of the chains are secured at coordinates ( X ;
CH ( X )) and (- X ; CH ( X )), it has a length
L = e X + e -X
Of course, between points ( X ; CH ( X )) and (- X ; CH ( X )) can be hung from the chain of some other length. Then form the chain will not
match the graph of the function CH ( x ). It is often said that the shape of a hyperbolic cosine have all the chains hanging in a uniform
gravitational field.
It turns out that all the hyperbolic cosine (geometrically) similar to each other: if we have two photos hanging chains, we can always increase
one of the photos so that when applied it to another photo image coincided chains (of course, may be "tails" which do not cover the image of
the chain on the other pictures, but overlapping areas coincide exactly).
Knowing the facts described after the word "help" and to this line, it is possible (though not easy) "school" methods to derive an equation for
the shape of any given length of chain hanging between any two given points. We suggest you think about it.
Note. The presence of the axis of symmetry in Sec. 2 is determined by eye. After that, the correct position of the line is better to check and, if
necessary, to clarify (for example, check that the distance from the point of intersection with the chain built the axis of symmetry to the ends
of the straight section of the border are the same figure). If it is not - just move the line. Two updates is usually sufficient.
6. (8-11) between terminals A and B are connected in parallel two identical chains of three series-connected resistors R , 2 R and R . Parallel to
each of the two resistors R connected two similar chains - R -2 R - R -, and themselves resistors 2 R , which is connected in parallel chains,
pointed (to avoid confusion). Parallel to each of the unmarked resistors 2 R again connected by two chains - R -2 R - R -, and themselves
resistors 2 R , which is connected in parallel chains noted. This operation was repeated an infinite number of times. Find the
resistance R AB between terminals A and B .
Solution. Let x > 0 - the resistance of such a scheme:

Consistently transform scheme as follows:

1 / x = (1 / (2 R )) + (1 / ( R + x + R )) + (1 / ( R + x + R ))

1 / x = (1 / (2 R )) + (2 / (2 R + x )) = (2 R + x 2 + 2 * R ) / (2 R (2 R + x )) = (6, R + x ) / (4 R 2 +2 R x )
x = (4 R 2 +2 R x ) / (6 R + x )
6 R x + x 2 = 4 R 2 +2 R x
x 2 +4 R x -4 R 2 = 0
x = (- 4 R + ((4 R ) 2 -4 (-4 R 2 )) 1/2 ) / 2 = <bR> = (- 4 R + (32 R 2 ) 1/2 ) / 2 = (- 4 R + 4, R 2 * 1.2 ) / 2 = 2 R ( + 2 1/2 -1)</b
x > 0, hence x = 2 R (2 1/2 -1)
Resistance x was the result of parallel connection of R AB and 2 R .
1 / x = (1 / R AB ) + (1 / (2 R ))
1 / (2 R (2 1/2 -1)) = (1 / R AB ) + (1 / (2 R ))
1 / R AB = (1 / (2 R (2 1/2 -1))) - (1 / (2 R )) = (1 / (2 R )) ((1 / (2 1/2 -1 )) - 1) = (1 / (2 R )) ((2-2 1/2 ) / (2 1/2 -1))
R AB = 2 R (2 1/2 -1) / (2-2 1/2 ) 2 = R ((2 1/2 -1) (2 + 2 1/2 )) / ((2-2 1 / 2 ) (2 + 2 1/2 )) 2 = R (2 1/2 / 2) = 2 1/2 R
Answer. R AB = 2 1/2 R .
7. (9-11) between the smooth stool and chair of the same height (horizontal) is the length of the board L mass M (weight is evenly distributed
along the length). The coefficient of friction between the board and the chair , between the board and stool - 0. stool sits mouse
mass m . Seeing a cat, a mouse runs across the board from the stool on the chair, hoping their actions so as to be in the chair as soon as
possible and do not fall down the board, based on the very edge of the chair. Find a job A, committed during perebeganiya mouse on the
board. Power mouse is not limited to, its paws on the board do not slip.
Solution. The mouse at each point of the board can not move with an acceleration greater than a certain number (depending on the point), and
its speed at the same time, obviously, can be arbitrary. Acceleration is determined from the condition that the horizontal force with which the
mouse operates on the board ( F = m A ), shall not be greater than the maximum static friction force of the board chair. This friction
force F tr = N , where N - the reaction force of the chair, acting on the end of the board, which is in this chair.
Let the mouse is on the board at a distance x from the stools. Was fixed to the board (as is required in the problem), it is necessary that the
sum of the moments of the forces acting on the board, the board relative to the end lying on the stool is equal to 0.
M G ( L / 2) + m G ( L - x ) - N L = 0
(Friction mouse on the board does not take into account, that is. To. The end of the board lies on the line along which the force acts, so the
moment of this force about the end of this equal to 0)
N L = M G ( L / 2) + m G ( L - x )
N ( x ) = ( M G ( L / 2) + m G ( L - x )) / L = (- m G / L ) x + (( M G / 2) + m G )
See that the dependence of the reaction force N (and hence the maximum static friction force F tr = N ) on the position of the mouse on the
board - linear.
Therefore, the mouse at each time point must move with the greatest possible acceleration. Since the acceleration is positive everywhere
(mouse never slowed down) and horizontal board committed mouse work equal to the kinetic energy (when the mouse is at the end of the
board at the beginning of the speed of the mouse on the condition was equal to 0).
We construct a graph of the acceleration of the coordinate. Since this relation is linear, area under the curve S is easy to find school
methods. Known from Formula 2 A x = v 2 - v 0 2 (or A x = ( v 2 / 2) - ( v 0 2 / 2) shows that this change in area is equal to half of the square of
the velocity of the movement, that is, (the so- as we have v 0 = 0) is simply v 2 / 2.
Work mouse that you want to find, A = m v 2 / 2 = m S .
F Max (0) = (M / 2) G ;

F Max ( L ) = (( M / 2) + m ) G .

A Max (0) = (1 / m ) F Max (0) = (1 / m ) ( M / 2) G ; A Max ( L ) = (1 / m ) F Max (L) = (1 / m ) (( M / 2) + m ) G .


S = L ( A Max (0) + A Max ( L )) / 2 = ( L / (2 m )) G (( M / 2) + ( M / 2) + m ) = L G (( M + m ) / (2 m )).
Answer: A = m S = m L G (( M + m ) / (2 m )) = (1/2) ( M + m ) G L .
Note. At first glance it may seem that the solution of this problem is easier to use the formula
Work = force * displacement
However, to justify the minimum time we still have to find the acceleration, and when we already found it - to bring the matter before the end
turns out to be easier The above method.
8. (10-11) On a clear sunny day Sherlock Holmes sat by the window, looking at the headline in your favorite big glass magnifier. It turned out
a larger image in the middle of playing a magnifying glass with almost no distortion, and the edges along the borders of the black letters
visible colored stripes - yellow from the edge of the magnifying glass and blue - from its center.
Explain the observed phenomena and fill (all or some) gaps in the table (please, why did you choose this color).
See. Table provided at p. 79 .
(If you have a proper full explanation of the task is considered regardless of the solved correctly filled table, otherwise the table is taken into
account.)
Solution. In the sunlight (and, of course, its reflection from the white paper) contains the entire color spectrum (rainbow). Glass has a
dispersion, i.e. refracts rays of different wavelengths (colors) at different angles.
Plain white paper through a magnifying glass, we also see white, t. E. From each point of the surface of the magnifying glass to our eyes
directed all the spectral components. However, they are reflected from differentportions of the surface of the paper (arranged one behind the
other in the order of colors of the spectrum (the order corresponding sequences rainbow color stripes, using easily remember phrases known
about pheasant and Hunter).
When, due to the presence on the paper surface of the drawing areas, these areas are recorded, not all spectral components in certain locations
spectrum is incomplete, and that contact is perceived as stripes.
Ordinary glass [ 14 there are glass and crystals that behave contrary, but the magnifier of them usually do not. ] refract more spectral
components with a shorter wavelength (in the "blue" part of the spectrum).
To explain what exactly happens when viewed through a magnifying glass strips of different colors, for simplicity, we assume that white light
consists of only three components: blue (the direction of the rays which varies most strongly magnifying glass), yellow and red (the direction
of the rays changes weakest). The white paper reflects all components, colored stripes on it reflect only "their" color and black do not reflect
anything at all.

, nm

Each

Hunter

Wishes

Know

Where

Cidit

Pheasant

By Red LED

About Ranjeva

F lty

W elny

Mr. oluboy

C iny

F ioletovy

510-560

480-500

600-760

570-590

380-470

Children's rhymes to remember the sequence of colors of the spectrum (rainbow)

Consider any point ( A ) on the white paper and the point A ' in the image of paper in the loop, which from the point A fall yellow rays, as well
as point O on paper whose image O ' is in the center of the image area of paper given by magnifying glass. Then the rays are yellow at the
point A ' fall from a point B , and blue beams - from the point C . Point A , O , B , C are on the paper on the same line in the order O - C (blue)
- A (yellow) - B(red). (Blue rays are refracted magnifying glass on the largest angle, red - on the smallest).
Spend now on paper the black border around the point A closer to the point O (point C was on this band, and the point A - a nearby kra'yu
magnifying glass edge 'of this band). Now the point of the image A ' will appear yellow (rays of red and yellow color of the points B and A to
point A hit it, and blue rays from point C - not, as now it is painted in black color and do not reflect any rays, the mixture yellow and red color
is visible as a yellow-orange). This explains the yellow coloring of the edges of the strip.

Similarly, if we draw a black band on the other side of point A (a point B ) - we see the point A 'in blue color (as against a black point B to
point A ' now does not enter the red rays).
As well explained and coloring the edges of the image colored stripes.
The above-described color distortion phenomenon in image formation lenses (or other optical systems) is known as chromatic aberration.
Properly completed table (these terms goals in normal text, empty cells are filled in italics ; hope that Sherlock Holmes had the same loop,
like the jury, even if not so - the differences are not very significant).
The object of observation Color stripe along near to
(on white paper).
center of the image magnifier.

the Color stripe along near the edge of the image


magnifier.

Black block letters.

Blue.

Yellow.

Line from red felt-tip pen.

Blue (purple).

Yellow.

The line from the green


The band is not observed.
felt-tip pen.

Yellow.

The line from dark blue


Blue (blue) .
ballpoint pen.

Yellow (yellow stripe between the visible and


the image of a thin red stripe).

Assignments
After the job number in parentheses indicates how classes, this problem is recommended. You are invited to meet four of the proposed tasks
of their choice (preferably solve problems designed for your high-end or more).
1. (8) By passing a mixture of hydrogen chloride and hydrogen bromide waterborne solution was prepared containing equal mass fractions of
hydrochloric and hydrobromic acids. What was the volumetric ratio of the gases in the mixture?
2. (8-9) The sample of limestone (calcium carbonate) was calcined to produce quicklime up until its mass stopped to change not. It was found
that the weight of the sample decreased by 22%. Determine the gangue content in limestone.
3. (8-10) After passing the carbon dioxide through a solution containing 3.7 g of calcium hydroxide was obtained 4 g of residue. Determine
the mass to absorb CO 2 .
4. (8-10) One of the most famous experiments in the history of chemistry was as follows. Swedish assistant pharmacist Carl Scheele made a
piece of burning phosphorus in a flask filled with air, and closed its stopper. At the end of combustion and cooling the flask he placed it
upside down in a container of water and opened the plug. The water rose to the flask by filling it to 1/5 volume. So Scheele discovered that
the air was previously considered a simple matter, consists of two main components - oxygen and nitrogen. What would change in the
observations Scheele, if instead he used phosphorus:
a) sulfur;
b) magnesium;
c) incandescent coal?
Answer explain the equations of the corresponding reactions.
5. (9-11) The equations of reactions, allowing for a chain of data transformations; specify the conditions of their occurrence. Transcribe
substances A, B, C, D (offer options).
N 2 -> A -> NH 3 -> B -> N 2 O -> N 2
Fe -> FeCl 2 -> G -> Fe (OH) 3 -> D -> Fe (NO 3 ) 3 -> Fe 2 O 3
6. (9-11) At your disposal are chlorine, sulfur, silver nitrate and water. Get as much as possible (not more than 20!) Of the new substances by
using these substances and their transformation products, as well as any laboratory equipment.
7. (10-11) In the production technology of copper there are terms "copper cathode" and "anode copper." What is the origin of these
terms? Record necessary to explain the response reaction equations. What kinds of copper cathode or anode, expensive, and why?
8. (10-11) of saturated hydrocarbons has a unique monohlorproizvodnoe and two isomeric dichloro. The possible structural formula of this
hydrocarbon.

9. (10-11) Chlorination isobutane chlorine izobutilhlorid the light received, and tertiary butyl chloride in the ratio 2: 1. What products and in
what proportion are obtained in the chlorination of 2,3-dimethylbutane under the same conditions? Write one reaction characterizing chemical
properties of each of the obtained isomers.
10. (9.11) Which of the following materials (in aqueous solution) or mixtures thereof are suitable for dissolving metallic copper
HCl, H 2 SO 4 , HNO 3 , FeCl 3 , H 2 O 2 , ZnCl 2 ?
Write the equation of the corresponding reactions and equalize them by electron or electron-ion balance. In cases where the reaction does not
go, explain why.
Brief solving competition in Chemistry
Challenges and solutions for the competition in chemistry prepared SE Semenov, director of the Chemical Lyceum N 1033 in Moscow, and ZP
Svitanko.
1. According to the law of Avogadro, volumetric ratio of gases is equal to their molar ratio. That is, in each molecule (HCl or HBr) gas have
the same volume (volumetric ratio so gases in the mixture ratio of the number of molecules of these gases). The molar mass M (HCl) = 36.5 g
/ mol, M (HBr) = 81 g / mol. By hypothesis, in an aqueous solution (and hence, the water passed through the gas mixture) containing equal
weight amounts of HCl and HBr, therefore ratio of the molecules of these compounds is the ratio of the masses of the molecules, which, of
course, equal to the ratio of their molar masses 81/36 5 = 2.22 molecules of HCl is easier therefore higher (2.22 times).
2.
CaCO 3 = CaO + CO 2 - reaction equation;
CaCO 3 - calcium carbonate (limestone)
CaO - calcium oxide (quicklime).
Molar mass of calcium carbonate M R (CaCO 3 ) = 100 g / mol, calcium oxide M R (CaO) = 56 g / mol. Calcining sample weight should be
reduced by 44% (((100 g / mole 56 g / mole) / (100 g / mol)) * 100% = 44%).Really, it decreased by 22%. Hence gangue content (in this case,
the impurities that do not decompose upon heating) is 22/44 = 0.5 (50%).
3. The passage of carbon dioxide into the solution of calcium hydroxide, the following reactions occur.
Ca (OH) 2 + CO 2 = CaCO 3 + H 2 O (1)
precipitation and an excess of CO 2 (when all the calcium hydroxide used in) the precipitate dissolves
CaCO 3 + CO 2 + H 2 O = Ca (HCO 3 ) 2

(2)

(As calcium hydrogencarbonate solution)


In solving the need to consider two cases.
1) Calcium hydroxide is in excess - are conducting an estimate on calcium carbonate. 4 g CaCO 3 up 4/100 = 0.04 mol. The amount of
CO 2 and 0.04 mol, which is 0.04 * 44 = 1.76 g
2) an excess of carbon dioxide. Then calcium hydroxide is exhausted. Its amount to 3.7 / 74 = 0.05 mole. In his deposition requires 0.05 mol
CO 2 . This should fall 0.05 mol precipitate CaCO 3 , which amounts to 0.05 * 100 = 5 g
As was actually obtained 4 g of the precipitate, the precipitate was dissolved 1 g of reaction (2).
It took (1/100) * 1 = 0.01 moles of CO 2 .
The total amount of carbon dioxide 0.05 + 0.01 = 0 06 mol, which is equal to 0.06 * 44 = 2.64 g
4.
1) In the case of carbon should be considered two factors. First, CO 2 , which is obtained by burning carbon - dioxide, in contrast to the oxides
of phosphorus. That is, it takes place that takes oxygen and which in the case of phosphorus immediately filled with water. The second factor
- is appreciably soluble CO 2 in the water, whereby the water still in the flask to rise, but less than 1/5.
2) Magnesium reacts not only with oxygen but also with nitrogen, so water can theoretically completely fill the flask. However, in practice
this does not happen, as it requires that the combustion of the magnesium in the flask was evacuated, which is unlikely. In any case, the bulb
is filled with more than 1/5.
3) In the case of sulfur, the situation is similar to carbon, but SO 2 solution is less than CO 2 , so the water in the flask will be even less.

5.
N 2 -> A -> NH 3 -> B -> N 2 O -> N 2
Fe -> FeCl 2 -> G -> Fe (OH) 3 -> D -> Fe (NO 3 ) 3 -> Fe 2 O 3
One of the options is as follows.

N 2 + 6Li = 2Li 3 N
Li 3 N + H 2 O = LiOH + NH 3
NH 3 + HNO 3 = NH 4 NO 3
NH 4 NO 3 = N 2 O + 2H 2 O
2N 2 O = 2N 2 + O 2 (at high temperatures) Fe + 2HCl = FeCl 2 + H 2 FeCl 2 + 2NaOH = Fe (OH) 2 + 2NaCl 4Fe (OH) 2 + O 2 + 2H 2 O = 4Fe
(OH) 3 Fe (OH) 3 + 3HCl = FeCl 3 + 3H 2 O FeCl 3 + 3AgNO 3 = 3AgCl + Fe (NO 3 ) 3 4Fe (NO 3 ) 3 = 2Fe 2 O 3 + 12NO 2 + 3O 2 (thermal
decomposition)
6. There are many possible reactions. Below are just a few of them for example.
2AgNO 3 = 2Ag + 2NO 2 + O 2 (term. decomposition)
2H 2 O = O 2 + 2H 2 (electrolysis)
S + O 2 = SO 2
SO 2 + NO 2 = SO 3 NO +
SO 3 + H 2 O = H 2 SO 4
H 2 + Cl 2 = 2HCl (to light)
AgNO 3 + HCl = AgCl + HNO 3
3NO 2 + 2H 2 O = 2HNO 3 + NO
SO 2 + Cl 2 = SO 2 Cl 2 , and so on. d.
7. In the process of copper production procedure is used its electrochemical refining
at the anode Cu - 2 e - = Cu 2+
on the cathode Cu 2+ + 2 e - = Cu
The crude metal is used as a soluble anode, the anode is called copper; pure electrolytic copper - copper cathode. From the point of view of
the additional energy cost in the preparation of cathode expensive copper anode in which (in the so-called anode mud) maximally
concentrated impurities present in original blister (Metal) copper. However, these impurities can be very valuable if they contain precious
metals which often accompany it in copper deposits, for example, silver or gold. In this case, the value of anode copper is able to exceed the
value of the cathode in the calculation of per unit mass.
8.
8. Examples of such substances
ethane CH 3 CH 3 ;
neopentane (2,2-dimethylpropane)

All the hydrogen atoms of the CH 3 can be considered to be equivalent (in fact, they are constantly changing places - CH 3 group of
"cool"). This statement may not be absolutely true if the environment of CH 3 -groups is strongly skewed, but in this case all the time as in the
order from ethane, obviously, the two CH 3 -groups lie on the axis of symmetry of the molecule, and are equivalent to each other. Therefore,
all the hydrogen atoms are equivalent and can be substituted by chlorine and any one of them - the result is the same molecule.
2,2-dimethylpropane molecule has tetrahedral form (in the center of an imaginary tetrahedron carbon atom located at the apices - methyl
groups -CH 3 ; cm. block diagram). All methyl groups in the molecule and therefore also equivalent monohlorproizvodnoe this molecule is
also unique.
There may be two ethane isomer dichloro - 2 chlorine atoms can replace hydrogen atoms are the same or different methyl groups. As is the
case with 2,2-dimetilpropanom - as in relation to the one selected top of a regular tetrahedron 3 others are equivalent, so the two chlorine
atoms can be placed in two different methyl groups only way.

9. When the chlorination of 2,3-dimethylbutane

can be obtained by the two products - by replacing respectively the primary and tertiary hydrogen atoms:

The same situation is observed in the case of isobutane. Wherein isobutane contains one tertiary and primary atoms nine H. If the rate of
replacement of these two positions were the same, the ratio of the reaction products would be 9: 1. Since the actual ratio of 2: 1, the rate of
substitution of tertiary hydrogen atoms in the 4,5 times faster than the initial atom substitution.
In the case of 2,3-dimethylbutane has 2 and 12 atoms of tertiary hydrogen atoms of the primary ratio 6: 1. Since the tertiary atoms are
replaced by 4.5 times faster, the ratio of products to (6 * 1) (* 1 4.5 ) = 4: 3.
Chemical properties of the different isomers. When interacting with the alkali solution of 1-chloro-2,3-dimethylbutane is converted to the
corresponding alcohol,

and in the case of tertiary chloride reaction product will be a major cleavage product, an alkene

(The location of the double bond - in accordance with Rule Zaitsev)


10.
1) Solutions of HCl, H 2 SO 4 (diluted) and ZnCl 2 is not dissolved copper (as copper is among a strained right of hydrogen, as well as the right
of zinc).
2) concentrated H 2 SO 4 reacts with copper
Cu + 2H 2 SO 4 = CuSO 4 + 2H 2 O + SO 2
3) Cu + 4HNO 3 = Cu (NO 3 ) 2 + 2NO 2 + 2H 2 O
(or a reaction to form NO)
4) Cu + 2FeCl 3 = CuCl 2 + 2FeCl 2
5) Cu + 2HCl + H 2 O 2 = CuCl 2 + 2H 2 O

6-7 classes
Of the proposed four tasks can be solved by any number of those that interest you. To get a premium correctly enough to solve any two tasks.
1. Is it possible to place the same 12 coins along the walls of a large square box so that along each wall lie flat
a) two coins;

b) three coin;

c) 4 coins;

d) 5 coins;

d) 6 coins;

e) 7 coins?

(Allowed to put a coin on the other.) In those cases where it is possible to draw, how to do it. In other cases, prove that coins can not be so
positioned.
2. In a group of 50 guys some know all the letters except "p", which simply omit the letter, and the rest know all the letters, but "a", which is
also passed. One day the teacher asked 10 students to write the word "cat", 18 other students - the word "mouth", and the other - the word
"mole". In this case, the word "cat" and "mouth" were written by 15 times. How many children have written their word right? Answer justify.
3. Lesch and Ira live in the house, each floor of which 9 apartments (one entrance in the house). Floor number equal to the number Leschi
Apartments Ira, and sum of the numbers of their apartments is 329. What is the apartment number Leschi? Answer justify.
4. Crazy cashier change any two coins on any three of your choice, and all three - on any two. Will you be able to exchange him 100 coins in
denominations of 1 ruble per 100 forint coins 1, giving him the exchange of exactly 2001 coin? Answer justify.
8-9 classes
Of the proposed four tasks can be solved by any number of those that interest you. To get a premium correctly enough to solve any two tasks.
1. Cut the active cell on the sides of the square into four parts so that all parts have the same size and the same shape, and that each part
comprises one cup and one sprocket.

2. On the table are four identical coins. Permission is granted to move the coin, not taking them away from the table. It is necessary to arrange
the coins so that you can put on the table the fifth coin of the same size, regarding these four. (In addition to these four coins to use other
objects and measuring devices is prohibited!)
3. Neznayka thinks that only an equilateral triangle can be cut into three equal triangles. Is he right?
4. Peter pulls out of the bag black and red cards and puts them into two piles. Putting the card to another card of the same color is
prohibited. Tenth and eleventh card posted Petya - red, and the twenty-fifth - black. What color is the twenty-sixth Laid card?
10-11
Of the proposed five tasks can be solved any number of those that interest you. To get a premium correctly enough to solve any two tasks.
1. The base of the pyramid of Cheops - square, and its lateral faces - equal isosceles triangles. Pinocchio climbed up and measured the angle
of the top of the Grand Prix. Get 100 O . Could it be?
2. On the table are four identical coins. Permission is granted to move the coin, not taking them away from the table. It is necessary to arrange
the coins so that you can put on the table the fifth coin of the same size, regarding these four. (In addition to these four coins to use other
objects and measuring devices is prohibited!)
3. The five-digit number is called indecomposable if it can not be decomposed into a product of two three-digit numbers. What is the greatest
number of indecomposable five-digit numbers can be a row?
4. Three equal triangle cut by oppositely medians (see. fig.). Can one get six triangles folded one triangle?

5. All the coefficients of the polynomial P ( x ) - integers. It is known that P (1) = 1 and that the P ( n ) = 0 for some positive
integer n . Find n .

Problem solving in mathematics competition


6-7 classes
1. Is it possible to place the same 12 coins along the walls of a large square box so that along each wall lie flat
a) two coins;

b) three coin;

c) 4 coins;

d) 5 coins;

d) 6 coins;

e) 7 coins?

(Allowed to put a coin on the other.) In those cases where it is possible to draw, how to do it. In other cases, prove that coins can not be so
positioned.
Answer: a), e) none; b), c), d), e) Yes.
a) Since by assumption all the coins you need to put along the walls, each wall and concerns exactly two coins, the total number of coins - not
more than 8.
b) -e) Examples desired position shown in Figure

e) Note that the coin can not touch the two opposite walls of the box. Therefore, the total number of coins related to the two opposite walls is
7 + 7 = 14> 12.
2. In a group of 50 guys some know all the letters except "p", which simply omit the letter, and the rest know all the letters, but "a", which is
also passed. One day the teacher asked 10 students to write the word "cat", 18 other students - the word "mouth", and the other - the word
"mole". In this case, the word "cat" and "mouth" were written by 15 times. How many children have written their word right?Answer justify.
Answer: 8. The word "mole" is not written correctly no one, because no one knows how to write both the letter "p" and the letter "n". The
word "mouth" or "cat" had to write 10 + 18 = 28 people. Note that only the words were written by the "mouth", "cat" and "off". The first two
words were written by 15 times, so the word "from" wrote 50-15-15 = 20 children from 28. This means that only 8 guys coped with the task.
3. Lesch and Ira live in the house, each floor of which 9 apartments (one entrance in the house). Floor number equal to the number Leschi
Apartments Ira, and sum of the numbers of their apartments is 329. What is the apartment number Leschi? Answer justify.
Answer: 296. Let Lesch lives on the floor with the number e in the apartment 9 E - K . Then Ira lives in the apartment E and the
condition E 9 E - K = 10E- K = 329. Since 0 < K 8, we obtain a unique solution K = 1, E = 24. Therefore Lesch lives in an apartment with the
number 9E- K = 296.
4. Crazy cashier change any two coins on any three of your choice, and all three - on any two. Will you be able to exchange him 100 coins in
denominations of 1 ruble per 100 forint coins 1, giving him the exchange of exactly 2001 coin? Answer justify.
Answer: No, it can. If two changes Peter three coins, the number of banknotes he increased by one. Let he produced N such exchanges. Gave
the ATM 2 N bills. In order to keep the total number of coins, Peter is forced to make the same amount of three exchanges of notes in two. At
the same time he will give another 3 ATM N coins. Total he gives thus two N +3 N = 5 N coins. 2001 but is not divided by 5.
8-9 classes
1. Cut the active cell on the sides of the square into four parts so that all parts have the same size and the same shape, and that each part
comprises one cup and one sprocket.

One possible way of cutting shown in the figure. Obviously, between two adjacent cells with the same icons (circles or asterisks) must pass
the cut (by hypothesis such icons must be in separate pieces). After the figure will be marked all such cuts, to come up with a final solution is
quite simple.

2. On the table are four identical coins. Permission is granted to move the coin, not taking them away from the table. It is necessary to
arrange the coins so that you can put on the table the fifth coin of the same size, regarding these four. (In addition to these four coins to use
other objects and measuring devices is prohibited!)
Put 4 coins "a diamond", as shown on the left. Next we will move them as shown in the same figure. The resultant configuration (right)
satisfies.

3. Neznayka thinks that only an equilateral triangle can be cut into three equal triangles. Is he right?
A: Dunno wrong. Take a right-angled triangle ABC , where the angle A = 30 O . (See. Figure) Let D - the middle of the hypotenuse AB . Cut
the triangle perpendicular DE hypotenuse ( E lies on AC ) and the segment EB .We got three right triangles. Triangles ADE and EDB are two
Katetov ( AD = DB , DE - total), and triangles EBD and EBC are on a leg and hypotenuse ( DB = CB , BE - total).

4. Peter pulls out of the bag black and red cards and puts them into two piles. Putting the card to another card of the same color is
prohibited. Tenth and eleventh card posted Petya - red, and the twenty-fifth - black. What color is the twenty-sixth Laid card?
Answer: red. Note that position when the top two cards are of the same color and position when the top two cards are of different colors,
alternate. Since the 10th and 11th card - red, then after it was laid 11th card from the top were two cards of the same color (red). Hence, after
25 was put th card (and any card with an odd number), were top two cards of the same color. Since the 25th black card, then the top two cards
- black. Therefore, the next, the 26th card can only be red (on a black card, you can only put red).

10-11
1. The base of the pyramid of Cheops - square, and its lateral faces - equal isosceles triangles. Pinocchio climbed up and measured the angle
of the top of the Grand Prix. Get 100 O . Could it be?
Answer: No, it should not be. Note that the triangles ABE and ABO - isosceles with equal bases. In this case, AE > AO and AB > AO (since
the rib smaller pyramid of its projection on the base). Therefore / AEB / AOB = 90O (diagonal of the square intersect at right angles).
2. See. Problem 2 for grades 8-9.
3. The five-digit number is called indecomposable if it can not be decomposed into a product of two three-digit numbers. What is the greatest
number of indecomposable five-digit numbers can be a row?
Answer: 99. Note that all numbers divisible by 100 are decomposable. Therefore, more than 99 consecutive irreducible numbers can not
be. On the other hand, between the numbers 100 * 100 and 100 * 101, all numbers are split, and there are exactly 99.
4. Three equal triangle cut by oppositely medians (see. fig.). Can one get six triangles folded one triangle?

Answer: You can. . See Figure.

5. All the coefficients of the polynomial P ( x ) - integers. It is known that P (1) = 1 and that the P ( n ) = 0 for some positive
integer n . Find n .
Answer: n = 2. Using the fact that P ( x ) - P ( y ) divided by the x - y . Hence, P ( n ) - P (1) = - 1 is divisible by n -1. Hence, n =
-1 + 1. Where n = 0 or 2. Since we are looking for a natural root, then the solution is unique: n = 2.

Physics
Numbers in parentheses after the specified task classes in which this problem is recommended. Sufficient to solve the problem only in its
class (and not necessarily all); solve other tasks is also possible.
1. (7-11) Dunno lies on the bottom shelf and looking at the cloudless starry sky through the window on the right side in the course of the
train. Stars slowly "move" on the window in the direction from the first to the last car."How quickly we go!" - Dunno thought. What can you
say about the speed and direction of motion of the train?
2. (10-11) Why surface railway wheels resting on the rails do not cylindrical, but slightly conical shape? In what direction should be directed
apex of the cone, inside or outside the railway track?
3. (7-11) In the process of laser printing paper is heated to a temperature of about 250 O C. Just a printed sheet of paper, you can safely pick up
- it seems a little warm, but if you spend over the surface of the sheet with your finger (or stretch it between your fingers) - you may burn your
fingers. How Come?
4. (7-11) standing on a wooden table transparent plastic bottle of soda, about half filled with sugar and screwed the lid. Dunno took a bottle
bottom, shake and put back. Thus prilpli sugar crystals to the inner surface of the bottle. Why, if the surface of the bottle to bring finger (not
even touching) saharinki from the inside surface of the finger immediately uprygivayut, jumping to adjacent areas or falling down?

5. (10-11) crane lifts the pipe. What portion of the cable (A or B - see Fig.) Stretched more and why?

6. (8-11) In modern cars are leading all 4 wheels. Special mechanical (or electronic) system optimally distributes engine power between the
wheels and prevents wheel slip when cornering on a road surface (for this wheels have to rotate at different speeds). However, in some cases
it is necessary to force the same speed of rotation of the wheels, the vehicle control systems such mode is also provided. When and why is it
used?
7. (11) Some irresponsible citizens include electric meter AC so that it starts spinning in the opposite direction. How do they do it (current
variable, just change the wires sometimes does not seem to make sense)?
8. (7-11) contraction and relaxation of muscle fibers occurs as a result of changes in the properties of their environment. To demonstrate these
phenomena biologists invented a mechanical device. It is based on a special thread (of collagen protein fibers that are similar to the muscle),
which, if it is immersed in a special solution (e.g. LiBr), reduced, then if - in the water recovers its original properties, and so can be done
many times. Diagram of the device shown in Fig. Different parts of the strands at a time immersed in water and the solution, the wheels
(diameter of large castors same) device continuously rotate. And in which way (specify the drawing in the right direction: A or B) and
why? (For the operation of the device is also required regular thread-pulley spanned by two round nozzles of different diameter, attached to a
large Castors).

9. (11) The electric charge of the neutron is 0, t. e. the Lorentz force does not act on it. However, the trajectory of the neutron can be bent in
an inhomogeneous magnetic field. Why is this happening?
Problem solving competition in physics
1. Answer. About speed nothing can be said, the direction - left turn.
T. k. The stars are very far away (you can tell at infinity), then the rectilinear motion of the train Dunno may not notice any relative movement
of the stars. Another thing is if the train turns (turns). Move the stars in the plane of glass, which oversees Dunno directed from the first to the
last car (Dunno himself looking into the right window), then the train turns left.
To Nightly nearest star, Proxima Centauri moved on discernible eye angular distance, the speed of the observer must be a lot more third space
- but then Dunno about the train and the other passengers we would have learned nothing. So changing the orientation of the observer. This
can be a turn trains or, in extreme cases - the daily rotation of the Earth.
2. The railways exist for a long time (the first is the line between the English cities of Stockton and Darling, opened 1825); it is obvious that
since the device railroad accumulated a lot of variety of technical details. Many of them emerged and spread totally awesome reasons and not
always been (and is) a successful and "right", but without them, as they say, does not go far. A similar situation - with a diameter water pipes
(if you do a little more or less - will not be screwed), voltage and frequency of household power outlet (220 V, 50 Hz, done differently - will
not work) and even form elektrorazetok and forks.
In this sense, the answer to the question "Why ...?" obvious - or on rails, is focused on a standard wheel (for historical reasons) it will be
impossible or difficult to ride.

In the development of complex technical systems the most unfortunate things often all bored and eventually become part of history (in
European museums transport really can see the rail wheels (or drawings, photographs, descriptions, projects) the most bizarre forms - many of
them were "rediscovered "tournament participants). Therefore, we can try to answer the question of why such a (possibly not the only and not
the best) solution remains as a standard and what problems it solves (or, at least, does not create); answer the question of why the decision it is
- much more difficult.
The convex shape of the rail head in combination with the conical shape of the wheel (the top of the cone is directed outwards railway track)
can improve the stability of the course car, uvtomaticheski adjust it under the direction of the railway track. Consider how the car will behave
in a straight section (a similar problem about electric locomotives, diesel locomotives, locomotives and other rolling stock with self-propelled
drive wheels is much more difficult - we will not consider it, the Ball, that historically it originated later, after replacing locomotives horses) .
At each time point the conical surface of the wheel in contact with the convex surface of the rail head at only one point (small area), all points
on the wheel lie on a circle (a circle of contact), the radius (distance from the point of contact of the wheels with the rail to the wheel axis) of
the circle depends on its location on the cone surface (closer to the top of the cone, the less). The right and left wheels wheel n \ 'ares
obviously rotate with the same angular velocity (make the same number of turns in the same time). Linear velocity of the wheel (at a given
angle) will be the greater, the greater the length of the circumference of contact.
If the direction of movement of the car coincides with the direction of the track and he goes exactly in the center (symmetric case), it's all
right. If the car has deviated from the center (for whatever reason - yaw, lateral impact force, and so on. N.) - The radius of the circle of
contact between the right wheel n \ 'ares with the rail will be more similar radiuca left wheel, the right wheel will move faster (with b \ 'Olsha
linear velocity) than the left, which will lead to the rotation rate of the wheelset towards the center of the track (in the case of overshoot
"equilibrium position" Wheelset will be deployed in the same way back and as a result of a number of "oscillation" is finally reigned "in
place" \ footnote {In large (unfortunately achievable with modern high speed train) speeds this may not be so - there are growing
"fluctuations", to deal with these fluctuations have to use special measures.}).
A similar (slightly more complicated) situation arises and curved track sections - the car automatically goes into a position (offset from the
center of the track toward the outside) when the radii of the circles of contact with the inner wheel (the closest to the center of rotation) and
the outer rails provide their movement with such linear speed (v wheel riding on the inner rail, the linear velocity is smaller in appearance more), which correspond to equal angular velocities of wheels. It also allows you to turn without sliding surfaces wheels on rails, with this
additional wear of rails and acoustic effects \ footnote {Trams, which the rail wheels and flat, passing turns "squealing"}, as well as virtually
avoid involvement in correcting the course flanges wheels (friction on their rails) basically undesirable for the same reasons.
Note that the reaction force of the rail in the process of correcting the course of the car plays a supporting role, not just changing the
coordinate, and the only direction of movement --- it allows you to distribute the work of the forces of reaction rails for moving the carriage in
the perpendicular direction of the section of rails rails more \ 'ow length and thus avoid unwanted b \ 'Olsha local loads (part of the work for
the car in general is accomplished by the reaction / tension intercar couplings, that is actually distributed to the forces of reaction on all
wheels zhelezhnodorozhnogo composition and strength of traction locomotive).
3. When we take a piece of paper in hand, our hand in contact with only a small portion of the sheet, and therefore the heat received by hand,
small \ 'o. (*) When we hold on the sheet, since the cooling time of a paper from small finger \ 'a (thin sheet, and the finger is much more heat
conductor than air), the heat is transferred from the finger across the strip, on which they held.
Supplement: finger in the ground (about 2/3) consists of water - its great heat capacity \ 'th, and the paper, even if it was wet, pre-dried in an
oven printer - its (paper) heat capacity is much smaller. After the establishment of thermal equilibrium between the portion of paper, squeezed
his fingers and toes areas, which actually distributes the heat from this site paper, get a small temperature. The word "small \" on "(*) must be
understood in this sense.
4. As a result of the friction saharinok bottle wall and each other and the walls (the wall surface portions) saharinki acquire electric
charges. Sugar and plastic (which is made of bottle) do not conduct electricity, so the charges are trapped on the surface of the plastic and
crystals of sugar during the shake-up, may for a long time (tens of minutes) to remain seated (while shaking the bottle, opposite charges arose
on plastics and sugar as a result of friction, and can also move from one surface to another during direct contact of these surfaces, for
example, when hitting saharinok each other, and when the shaking ceased saharinki "lay down" the greater part of their surface and the wall
surface of the bottle, no schem is not in contact and got on the surface of these charges because there are - on the one hand, sugar or plastic,
on the other - the air, and both - izlyator).
Obviously, the strength of the electrostatic interaction of these charges, together with the forces of gravity and friction reatsiya and hold on the
inner surface of the bottle "stuck" to it saharinki (note that the strength of the electrostatic interaction in this case - is not necessarily a force of
attraction, repulsion forces here may well act as a "backups").
When we offer a bottle of thumb, it causes charge redistribution and disrupts the existing balance of forces - part saharinok "are superfluous"
and fall down or find themselves "more convenient" place (perhaps even higher than the original).
Finger is a conductor of electricity (its electrical resistivity is much less than the resistance of sugar and plastic) and therefore interact with
electric charges. Recall that the electric charge is attracted to the uncharged conductor (he "collects" on the conductor charges of opposite sign
himself "close to him" and attracted to him more than repelled by the charges of his mark left on). (This is why experience is obtained, if the
bottle stands on a non-conducting wooden table, not on the metal.) The man conducting experience (Dunno) may well turn out to be charged
(with his finger) - it is enough to walk on dry linoleum or carpet (in one of the suburban holiday homes have this effect is particularly evident

- if a person goes through the corridor with carpet by tall windows with curtains - curtains "are trying to" wrap it, if not then touch the metal
railing on the stairs - beats current).
Offering this task, the jury had hoped the tournament easily determine the sign of the charge of the bottle and saharinok and write about it in
the decision. But it was not so easy. It was found (by the deviation in the electric and magnetic fields) that among the flying bottle of fine
particles have charged both positively and negatively. Watching the scene, besides it is impossible to know which of these particles - saharinki
what - ripoffs by shaking the bottle surface with pieces of plastic, and which (possibly) - Are stuck "snowballs" of saharinok and pieces of
plastic. Apparently, the only way to solve the problem is to catch a particle; come up with a simple way of fishing we could not ...
Apparently, these small particles flights and are the main reason for the charges "drain" with larger saharinok, and those, in turn, fall off from
the walls of the bottle. Raised a finger, we obviously not only directly affects the static equilibrium saharinok, but also to change the trajectory
of these inconspicuous small particles, that after some time also affects the behavior of "large" saharinok.
5. Answer: A. Decision. The remaining non-designated portion of a cable is denoted by the letter B. Obviously he is vertical (because the
force of its tension is balanced vertically directed force of gravity acting on the tube). It is also evident that "the sequel to" tether passes
through the center of mass of the tube (which coincides with its geometrical center, ie. A. Reasonable to assume that the pipe is homogeneous;
denote the center of the letter O ).
To further avoid confusion, the end of the tube, to which is attached a cable A and is denoted by the letter A , the second end, of course, the
letter B , and Y -shaped junction of three cables - this letter Y . ("E, B sat on the tube ..." - children's rhymes).
It turns out that in this situation at any angle tube (except when it is horizontal or vertical) angle OYA is always smaller than the angle OYB - it
will be proved in the last paragraph of the decision of purely geometric considerations.
The horizontal projection of the wire tension forces A and B must be equal in absolute value (no other forces in the horizontal direction on the
tube does not work, so they have to balance each other).
A rope "more vertical" than B (for the angles formed by them with the vertical angle of the equality OYA < OYB , which we mentioned above
and prove below), so that it be the same as in B, the horizontal projection of the tension force, it must be tensioned stronger than B.
Now finally we shall understand with corners like this. Almost obvious, obtuse angle / YOA , understand, less acute angle
cosine / YOB . Triangles AOY and BOY have equal areas (they have a common base OY and equal sides OA = Ob , is inclined to the ground at
equal angles (though in different directions), the has a height of these triangles are equal). On the other hand, these areas can be expressed in
terms of interest to us are the angles S = (1/2) OY * YA * sin / OYA = (1/2) OY * YB * sin / OYB . Since OA = Ob and YA >: YB , up for
equality necessary condition sin / OYA <sin / OYB . If these angles are acute (which corresponds to the figure), it follows
immediately / OYA < / OYB if / OYB blunt and / OYA - acute (ie, the point Y is below the point B ), it is also obvious (less acute angle
obtuse ); case obtuse angle OYA physically unrealizable (formally, in this case the tension force of cables A and B will be negative).
6. On turns the outer wheel is spinning faster, so it served more power. Hence, the system is designed so that the higher the speed spinning
wheel, the greater portion of the power is flowing through it.
After contact wheels for example dirt capacity RV redistributed mainly to the wheel, which rotates at a higher speed, so if one wheel in
contact with dirt, or, for example, ice, or if one of the wheels with a 'hanging "in the air (a machine is based on the three other wheels), it will
put more power and spin will actually only it, but on the other wheel power supplied almost will not, so the car will skid. This can be avoided
if it is compulsory to equalize the speed of all axes.
7. The instantaneous electrical power is given by N inst. = I inst. U inst. . It can be shown that if the current and voltage depend on time
sinusoidally with the same frequency (but not necessarily the same phase; this is what is usually called "network variable current "), the
average (per period) power can be calculated by the formula N = I U cos (*), where I and U - average (t. n. current) values of current and
voltage, respectively, - the phase difference between the current and voltage.
For simplicity, we will not consider the specific design electricity meters, and we assume the counter "black box" in which a certain part
rotates at a speed proportional to the expression in the right-hand side of (*) and the amount of its turnover is calculated by special mechanical
device.
The direction is determined by the sign of the torsion meter power supply current. N = I * U * cos , (1) where - phase shift between I and
U. To change the sign, it is necessary to change the phase between the current and the output voltage to a value more / 2. You can
implement this by connecting an ammeter in parallel counter coil and parallel to the consumer in the apartment - the condenser. (Note that the
inside of the counter can not climb for this and then not to disrupt the seal power company and is not to be penalized.) The impedance of the
capacitor 1 / ( C ) must be less than the impedance of the voltmeter to the counter, and the impedance of the coil L must be less than
the impedance of the ammeter in the counter, then the coil cause a positive phase shift in the measured current is larger than / 4, and the
capacitor will cause a phase shift in the measured voltage is less than - / 4). Then the phase difference between the measured current and
the measured voltage exceeds / 2, therefore cos / 4 in formula (1) is negative, hence a negative power is measured. Counter will spin the
other way.

In this case, it is actually fairly: the flow of electromagnetic energy (Poynting vector) through the counter is really aimed at "the apartment"
(of course, due to reverse flow through the coil).
To combat the above landlords, it would seem, it is possible to produce counters that measure is not an absolute value, a power module,
passing through the meter (technically it is quite possible, though difficult - in the extreme case record I ( T ), U ( T ) and then let the
computer still thinks). But in fact this is wrong - at the present as the power and presence in the apartment of large reactive loads (eg washing
machine) periodically there are moments when the electromagnetic energy "leaking" out of the apartment into the grid. Do not make
consumers pay for it, and in 2 times more!
8. Just note that the decision does not need to be told \ footnote {the jury do not know a single relatively simple, concise and understandable
explanations} students why the device will be working - it is given in the problem.
The velocity value v -thread at each pulley Collagen filaments from the speed at the respective wheels (1 and 2, with the nozzles of the
radii r 1 and r 2 , respectively) having a radius R . Therefore, the rate of collagen fibers in wheel 2 must be greater than the rate at wheel 1. (*)

In a concise and free parts of the collagen strands set the speed u 'and u ", respectively.
Per unit included in the solution comes out of solution and the same mass of collagen (and where it would otherwise accumulate or,
conversely, just ended). Since compressed collagen has their increased linear density than uncompressed, compressed then have to go with a
smaller skoroctyu.
So from (*) collagen thread should be compressed in one wheel and stretched from wheel 2. Thus through the wheel 2 is thrown loose thread
that already had time to go in the water, and therefore moving in the direction of B.
The correct answer to this problem can be easily guessed and partially justified from the "engineering" reasons (which is why the problem
was proposed and including 7-graders). In fact, if the direction of motion A was then pre Washed yarn immersed in water to a solution being
spanned by the wheel 2, and it would begin to "puchitsya"; this design decision - obviously unsuccessful, so in fact it should be the opposite.
Tournament participants noted that the device described is a "perpetual motion machine" (and some of this on the main concluded that the
device will not work because in fact there is no perpetual motion machines). Engine and this is not really the eternal: during his work with the
collagen filament from a solution of LiBr in water at all times of the transferred solute \ footnote {more precisely, the concentration of LiBr in
the vicinity of the thread is an increasing function of the concentration of the solution and a decreasing function of the relative lengthening the
thread; if the device is forced to rotate in the direction opposite to that in which it is itself "wants" to rotate, it can be used to extract LiBr from
the vessel with water and transfer to a vessel with a saturated solution (if the concentration of LiBr) in a container with water is not very
small} when the concentration of LiBr solution in the former and the former water catch up device stops working.
9. In order to bend the magnetic field trajectory of a moving particle, the particle does not necessarily have to be an electrical charge: a
counterexample is quite good flying in the magnetic field of the magnet .... This is formally correct answer.
Now try to explain what is simplistically still occurs in a magnetic field with a neutron. Neutron itself is as though a small magnet. Complex
and not completely clear question of why this is so, we will not disassemble, is organic-known fact that the energy of interaction between the
magnet and the magnetic field is proportional to the scalar product of the vectors of the magnetic moment of the magnet and the intensity of
the external magnetic field (unit vector of the magnetic moment M flat closed loop current equal to the product of the current I on loop
area S , a vector directed perpendicular to the plane of the loop, in a sense, a neutron behaves as a circuit with M = IS = 9.6614 * 10 -31 A *
m 2 ).
The magnetic moment of the neutron geometrically connected with its angular momentum, their orientation can not be changed. If along the
trajectory of the neutron intensity of the magnetic field changes (for simplicity we assume that the magnetic field does not change), this leads
to a change in the energy of the neutron interaction with the magnetic field due to the kinetic energy. This leads to a change in kinetic energy,
therefore - pulse (Ek =) and the de Broglie wavelength of the neutron. E., Some of the magnetic field for the neutron fields are "optically" (in
terms of the wavelength of de Broglie) more dense and some - less dense, resulting in a "refraction" (by analogy with optics), m. E. Change
the direction of motion of the neutron.
Unfortunately, many school physics textbooks are written so that the material contained in them is very easy to understand is not entirely
correct (which is why, in fact, experienced this problem), and need to be clarified. For example, of the allegation that the magnetic field in a
cloud chamber charged particles fly through curves, it does not follow that the uncharged particles always fly in a straight line - is, as we have
just seen, is not true. Another example - the experience of a deviation from the scope of the current in the field of permanent magnet - here, in
our view, a clear need to clarify that the frame should be made of non-magnetic material.

Chemistry
Tournament participants are invited to solve three or four tasks of choice. Preferably, but not necessarily,
choosing tasks for their intended or older class (classes are listed in parentheses after the number of tasks).

1. (8-9) Volume of the mixture of carbon oxide (II) with 250 ml of oxygen (eg. in.). After oxidation, the entire volume of the oxide mixture
was found to be 180 ml (n. In.). The resulting gas mixture is passed into a solution containing 0.25 g of sodium hydroxide.
A) to determine the composition of the initial mixture (by volume).
b) For a compound formed in a solution after absorption of the reaction products? Answer confirm calculation.
2. (8-9) Two cubes of the same size, one of which is made of aluminum, magnesium and other dissolved in dilute sulfuric acid. In the first
case, the amount of hydrogen evolved was two times larger than the second. What is the density of magnesium, aluminum, if the density of
2.7 g / cm 3 ?
3. (8-10) In the laboratory, there are five flasks, aqueous solutions of various substances. At each bulb has a label with the name. The first
flask is written "barium iodide," the second - "Sodium carbonate", the third - "sulfuric acid" in the fourth - "copper chloride", and the fifth "Sodium hydroxide". Unfortunately, the labels are reversed so that none of the solutions is not properly signed. When pumping solution from
the first flask with a solution of a second flask gassing, and by mixing a solution of the first flask with the contents of the flask third white
precipitate formed.
A) How to swap the labels to all flasks were signed right?
b) Write the reactions mentioned in the subject.
c) What other reaction could be drawn between these substances?
4. (9-10) The young chemist Bob found at the dump plate of gray metal. He brought it home and weighed. Her weight was 10.50 g Vasya
prepared copper sulphate solution, taking for this purpose 5.00 g of copper sulfate (CuSO 4 * 5H 2 O). Then he put his plate in the
solution. After a while the blue color of the solution disappeared. Bob got a record that has got red, washed it with water, dried and weighed
again. At this time, the mass of the plate was found to be 9.54 g Vasya made a simple arithmetic operations and determine from a metal plate
made found them. How did he do it? From a metal plate was made Vasin?
5. (9-11) Metal A simple gas reacts with the substance B to form a solid compound in which dissolved in excess hydrochloric acid, forming
salts G and D . Sol D interacts with the alkali solution with the release of gas E .What are the compounds listed, if you know that
salt D contains 25.26% of the metal A .
6. (10-11) What is the minimum amount of monochloroacetic acid to be burned, so that after absorption of gases formed potassium hydroxide
solution of 100 g with a mass fraction of 11.2% did not precipitate by the addition of calcium chloride? What substances are contained in the
solution after the absorption of the products of combustion?
7. (10-11) Give an equation for the reaction, allowing for a chain of data transformations; specify the conditions of their
occurrence. Transcribe substances A , B , B , G , D (offer options).
A) CH 4 -> A -> C 6 H 6 -> C 6 H 5 NO 2 -> C 6 H 5 NH 2 -> B - > C 6 H 5 OH
b) CH 2 = CH 2 -> B -> CH 3 CHO -> CH 3 COOH -> G -> acetone
c) CH 2 = CH 2 -> C 2 H 5 Br -> D - > C 2 H 5 COOH -> C 2 H 5 COOCH 3
8. (10-11) Alkene A appends one mol HBr to yield a compound B . When heated, B with metallic sodium forming hydrocarbons in
the bromination with molecular bromine which leads only to the two monobromo - F and D, with a substance D formed in much larger
quantities. Molar mass bromides D and E of 1.34 times the molar mass of B . Define the structure of all the substances. Give an equation for
the reaction.
Problem solving competition in Chemistry
1. a). Carbon monoxide oxidation occurs by reaction
2CO + O 2 = 2CO 2
Since two moles of CO to form two moles of CO 2 , t. e. the volume remains the same, it is obvious that the reduction of volume equal to the
volume of the mixture of oxygen, which entered into reaction, ie. e. reaction took 250-180 oxygen = 70 ml.
Since one mole of O 2 reacts with two moles of CO, then reacted with 140 ml of CO.
In the condition states that carbon monoxide is completely oxidized, then it was 140 mL.
Oxygen was 250-140 = 110 ml (70 of them spent ml and 40 ml left). Thus, the composition of the initial mixture of 140 ml (56%) CO, and
110 ml (44%) O 2 .
b). Upon absorption of the reaction products with NaOH solution may flow following processes:
CO 2 + NaOH = NaHCO 3 ,
2CO 2 + NaOH = Na 2 CO 3 .
May form sodium carbonate, sodium bicarbonate or both substances that determined the ratio of the reactants. (CO 2 ) = n (CO) = 0.140 /
22.4 = 0.00625 (NaOH) = 0.25 / 40 = 0 , 00625.
The molar ratio of the reaction products exactly corresponds to the formation of sodium hydrogencarbonate, and then turn it into solution.

2. The tournament participants suggested many correct ways to solve this problem. Thus, it should be borne in mind that here is only one
possible calculation.
Let the volume of cubes about 1 cm 3 . Then, m (Al) = 2,7 g, (Al) = 2.7 / 27 = 0.1 mole (wherein 27 - the molar mass of aluminum).
2Al + 3H 2 SO 4 = Al 2 (SO 4 ) 3 + 3H 2
mol Al 2 give 3 mole H 2
0.1 moles
x mole x = 0.15 mole.
When dissolving 0.075 mol of magnesium hydrogen released as the condition for the amount of hydrogen in this case is less than two times,
and hence the amount of substance in half.
Mg + H 2 SO 4 = MgSO 4 + H 2
(Mg) = (H 2 ) = 0.075.
The weight of magnesium cube 24 * 0.075 = 1.8 g (24 - is the molar mass of magnesium). Since the volume was set at 1 cm 3 , the density of
magnesium 1.8 g / cm 3 .
3. a) Vial 1 Vial + = 2 gas.
The gas is formed only by the reaction of sodium carbonate with the sulfuric acid. Hence, the first and second flasks - is sodium carbonate
and sulfuric acid. In this case, sodium carbonate can not be in the second flask, because it says "sodium carbonate", and all the signs are not
true. Therefore, the sodium carbonate is in the first flask and the sulfuric acid - a second.
Vial 1 Vial + 3 = white precipitate.
Na 2 CO 3 + 3 bulb = white precipitate.
The only option for the third flask - barium iodide (CuCl 2 to form a green precipitate carbonate and alkali does not form a precipitate with it).
Inscriptions on the fourth and fifth flask remains just swap, as all solutions are signed correctly.
Thus, we have: 1 - Na 2 CO 3 , 2 - H 2 SO 4 , 3 - BaI 2 , 4 - NaOH, 5 - CuCl 2
b) The reactions mentioned condition:
Na 2 CO 3 + H 2 SO 4 = Na 2 SO 4 + H 2 O + CO 2 ;
Na 2 CO 2 + BaI 2 = BaCO 3 + 2NaI
c) In addition, these substances can enter into the following reactions:
BaI 2 + H 2 SO 4 = BaSO 4 2HI +
2NaOH + H 2 SO 4 = 2H 2 O + Na 2 SO 4,
CuCl 2 + 2NaOH = Cu (OH) 2 + 2NaCl
2CuCl 2 + 2Na 2 CO 3 + H 2 O = (CuOH) 2 CO 3 + 4NaCl + CO 2
2CuCl 2 + 2BaI 2 = 2CuI + I 2 + 2BaCl 2
4. When making metallichechskoy plate in a copper sulfate solution, the metal is oxidized and passes into solution and the copper plate on
osazhdaetcya.
Assume for the moment that a monovalent metal.
Then the Cu 2+ + Cu + = 2Me 2Me +
disappearance of the blue color of the solution leads to the conclusion that copper has reacted completely. Calculate how much copper is
contained in a given amount of CuSO 4 5H 2 O. The molar mass of copper sulfate 250 g / mol, and copper 64 g / mol. If 250 g contain 64 g of
copper, then 5 g - 1.28 g of copper. Thus, 1.28 g of settled plate copper.
If on the plastic was not copper, weight plates would have 9,54-1,28 g = 8.26 means that a solution has passed 10,50-8,26 = 2.24 g of an
unknown metal.
Denote molar mass of metal through M .
Then 64 g of copper was reacted with 2 M g of the metal
copper of 1.28 g - 2.24 g of metal with M = 56. This molar mass is only iron, but it is not monovalent.
If a divalent metal, M = 56 = 2 * 112. Suitable cadmium. For the case of trivalent metal no options. Thus, the plate was made of cadmium.
5. By reaction with gas evolution alkali salt, D - ammonium salt, in this case, the ammonium chloride. The formation of ammonium salts with
a metal salt simultaneously indicates that the compound B - a metal nitride. Indeed, the nitride is formed upon reaction with simple metal
substance gas (nitrogen). Thus, the salt of A is a metal chloride. Let the molar mass of metal x . Then chloride molar mass x 35.5 n , where n the valence of the metal chloride in.
( x / ( x 35.5 n )) = 100 25.26.

From x = 12, n . When n = 1 there is no such a metal. When n = 2 is magnesium. When n = 3 is no metal. For n = 4 corresponds to the mass of
titanium (48), but does not come into titanium described reaction and TiCl 4does not exist in aqueous solution.
Thus: A - magnesium B - nitrogen in - magnesium nitride, F - ammonium chloride, L - magnesium chloride, E - ammonia.
3Mg + N 2 = Mg 3 N 2
Mg 3 N 2 + 8HCl = 3MgCl 2 + 2NH 4 Cl
NH 4 Cl + NaOH = NH 3 + NaCl + H 2 O
6. 2ClCH 2 COOH + 3O 2 = 4CO 2 + 2H 2 O + 2HCl
In the absorption of gases of alkali solution are reaction CO 2 + NaOH = NaHCO 3 and NaOH + HCl = NaCl + H 2 O.
Hydrogen must be formed precisely, since in this case does not fall precipitate upon addition of calcium chloride, but if the medium is a salt
solution, the calcium carbonate is precipitated.
KOH solution contains 11.2 g, mp. E. 0.2 mol of KOH.
It is evident that during the combustion of 1 mol 2 mol acid obtained CO 2 , and 1 mole of HCl, m. e. on the absorption of combustion
products of 1 mole of the starting acid need 3 moles of alkali (to get the acid salt).
1 mole to-you 3 moles KOH x mol to 0.2 mol, you KOH. x = 0.067 mol.
Calculate this value in the program is not a problem, but it can not do, as in the problem asks amount of substance.
7. The reactions may for example be as follows:
a) CH 4 -> A -> C 6 H 6 -> C 6 H 5 NO 2 -> C 6 H 5 NH 2 -> B -> C 6 H 5 OH
CH 4 -> C 2 H 2 + 3H 2 (1500 O C, A - acetylene)
3C 2 H 2 -> C 6 H 6 (600 O C, activated carbon)
C 6 H 6 + HNO 3 -> C 6 H 5 -NO 2 + H 2 O (in the presence of H 2 SO 4 )
C 6 H 5 -NO 2 + 3Zn + 6HCl -> C 6 H 5 -NH 2 + 3ZnCl 2 + 2H 2 O
C 6 H 5 -NH 2 + NaNO 2 + 2HCl -> [C 6 H 5 -N + = N] Cl - + NaCl + 2H 2 O ( B - fenildiazoniyhlorid)
[C 6 H 5 -N + = N] Cl - + H 2 O -> C 6 H 5 OH
b) CH 2 = CH 2 -> B -> CH 3 CHO -> CH 3 COOH -> G -> acetone

CH

= CH

+ (1/2) O

O
/ \
= H

C --- CH

(200

C, the catalyst - Silver) ( In - ethylene oxide)

O
/ \
H 2 C --- CH 2 -> CH 3 CHO (300 O C, Al 2 O 3 , isomerization)
CH 3 CHO
+
2Cu
(OH) 2 ->
CH 3 COOH
+
2CH 3 COOH
+
Ca
(OH) 2 ->
2
(CH 3 COO) 2 Ca
+
2H 3 O
(CH 3 COO) 2 Ca -> CH 3 (CO) CH 3 + CaCO 3 (pyrolysis)
c) CH2 = CH2 -> C2H5Br -> D -> S2H5COOH -> S2H5COOCH3
CH 2 = CH 2 + HBr = CH 3 CH 2 Br
CH 3 CH 2 Br + NaCN = CH 3 CH 2 CN + NaBr (substance D - CH 3 CH 2 CN)
CH 3 CH 2 CN + 2H 2 O (H + ) = CH 3 CH 2 COOH + NH 3
CH 3 CH 2 COOH + CH 3 OH (H 2 SO 4 ) = CH 3 CH 2 COOCH 3 + H 2 O
8.
With n H 2 n + HBr = C n H 2 n +1 Br ( A -> B )
2C n H 2 n +1 Br + 2Na = C 2 n H 2 (2 n +1) + 2NaBr ( B -> B )
C 2 n H 2 (2 n +1) = C 2 n H 4 n +2
C 2 n H 4 n +2 + Br 2 = C 2 n H 4 n +1 Br + HBr ( In - > T + D )
Molar mass G or D is 12 * 2 n + (4 n +1) + 80 = 28 n +81
molar mass of B is 12 n +2 n + 1 + 80 = 14 n +81
(28 n 81) / (14 n +81) = 1.34. Hence, n = 3
Then A CH 2 = CHCH 3 B CH 3 CH (Br) CH 3 In

C CH
\ /

Cu
(D-

O
+
2H
calcium
acetate

O
)

CH-HC
/ \
H 3 C CH 3
It can be seen that the bromination of this substance can actually get two isomeric monobromo. This further confirms the structure
bromide B , t. k. a hydrocarbon is produced from the reaction by distilling it (from sodium).Confirmation of the structure of B is necessary as
a condition of its receipt is not specified, and under certain conditions (in the presence of hydrogen peroxide) connection HBr goes against
Markovnikov's rule, and then turned to the other bromide ( B ).
Substances G and D - (CH 3 ) 2 C (Br) CH (CH 3 ) 2 ( F ) and (CH 3 ) 2 CHCH (CH 3 ) CH 2 Br ( A ). Substance D - Product substitution at the
tertiary carbon atom - is formed in larger amounts.
Competition in Mathematics
Assignments
In brackets, which classes addressed the problem. Your performance in mathematics is considered successful if properly resolved at least two
tasks addressed to you or more senior classes. Only one correct solution of the problem will also be awarded by the jury.
1. (7-9) For some years (from January 1 to December 31 inclusive) the amount equal to the number Tuesdays Thursdays. Does it follow that
the number of media was the same? Consider two cases: a ) in the year was 365 days, b ) in the year was 366 days.
2. (7-9) All natural numbers from 1 to 1000 inclusive are divided into two groups, even and odd. In some groups the sum of all digits used for
recording numbers more and by how much?
3. (8-9) It is known that x = 2 and 5 = 5 b 2 > 0, the number of a and b - integers. What is the smallest possible value of x ?
4. (7-10) given line and a point outside it. How to use a ruler and compass to construct a line parallel to a given line and passing through a
given point, drawing a smaller number of possible lines (circles and lines), so that the last drawn line - is required straight line? How many
lines you have achieved? 5. (8-11) Given a square with side 1. each side is divided into three equal parts. Through the points of division
conducted segments (see. Figure). Find the area of the shaded square.

6. (8-11) divide each four-digit number for the sum of its digits. What is the biggest result you get?
7. (10-11) The polyhedron inscribed in a sphere. Could it be that this polyhedron is nonconvex? (Polyhedron inscribed in a sphere, if all the
ends of its edges lie on a sphere.)
Solutions to the competition in mathematics
1a. In the year of 365 days, that is 52 full weeks plus one day. If it starts from the environment (for example, the 2,003th year), then the media
will be one more than the Tuesday and Thursday.
The answer is: not to be.
1b. 366 days - is 52 weeks and 2 days more. They can not be Tuesday and Thursday as these days are not consecutive. Not one of these days,
not the medium, because otherwise the next day would be Tuesday or Thursday, and this condition is violated equality Tuesdays and
Thursdays. Hence, the media of the year no more and no less.
Answer: should be.
2. The sum of digits of the number 1 is the sum of the digits of 1000; the remaining numbers will divide into pairs: 2-3, 4-5, 6-7, 8-9, ..., 998999. In each pair of units of an odd number greater than 1, than the even, and dozens and hundreds of them equally. Total of 499 pairs.
Answer: The sum of the digits in the odd numbers greater than 499.

3. The number x is divided by 2 and 5. x = 2a 5 , so as divided by 5. As x = 5 b 2 , then x is divisible by 2 two means and divided by 2 and
also. Assuming a = 2 * 5 = 10, we obtain x = 200000 and b = 200.
Answer: 200,000.
4. Dana direct a point and O (designation). Note on the line two arbitrary points A and B . Draw a circle centered at B of radius AO , and the
circle with center O radius AB . They intersect at point X . Quadrilateral AOXB- parallelogram, as its opposite sides are equal. You can now
carry out the required line - OX .

Outlining the same solution in other words, we can say that we have a standard way to construct a triangle BOX two vertices ( B and O ) and
the lengths of two sides of equal length segments AO and AB . It is obvious thatABO = XOB (on three sides). Therefore, the
angle ABO = angle XOB , and this internal nakrestlezhaschie angles to direct a and OX and cutting BO . Of these angles should be equal,
that a and OX parallel.
Another solution. Note to direct an arbitrary point A and draw through the point O circle centered at A . This circle intersects the line at two
points; we denote by M and N . Next measure [ 1 cm. clarification on the page. 11 ] compass segment MO and spend about a point N circle of
radius MO . Seeking line passes through the point O and the point B of intersection of two circles constructed.

MAO = NAB on three sides, therefore, are equal and the height of the triangle drawn from the vertex O and B . The bases of these
triangles ( MA and NA ) lie on the line A , so that the points O and B are straight froma the same distance.
The disadvantage of this solution is that if the point A happens to be the base of the perpendicular drawn from the point O , then the
points O and B are the same and we do not define the desired line.
However, students drive this decision, it was counted fully (to grade +) [ 2 In fact, this same lack of "masked" in the first decision, in the
sentence "Note on the line two arbitrary points A and B . " If the points are arbitrary, they can coincide by chance (and then construction will
not work), and for the construction of a straight two distinct points have to spend additional lines. ].
We now prove that the two lines can not be dispensed. The second line should be sought straight. To hold it, you need to get a second point
which is at the same distance from the line a, as the point O. But after one line all the points of the line, except at the points of intersection
with the line a, are indistinguishable, and to find the second point, located on the the right distance from the line a, built only one line, it is
impossible.
A: 3 lines.
Explanation. The decision we mentioned parallelogram, triangle, cross-section and the corners of BO. However, to construct we needed only
points (vertices of the parallelogram and triangles, cutting the ends of the segment, the ends of the segments forming the corners), they
themselves need to build the segments were not, so we have not carried out and, of course, do not qualify conducted lines.
5. Let the length of the side of the shaded squares (figure on p. 7 ) through x . Note that all the sloping segments have length x , 2 x / 3 and x /
3, it follows from the theorem of Thales. Then, put together a trapezoid with sides x , x , 2 x / 3, 1/3 and triangle with sides x 1/3, x / 3, we

obtain the square equal to the shaded. Similarly, you can fold the square of the two trapezoids with sides x , x / 3, 1/3, 2 x / 3. A total of 10
identical squares of total area of 1. This means that the area of each square is equal to 1/10.
You can solve the problem by using the Pythagorean theorem, we can write it to the smallest triangle in the figure:
x 2 + ( x / 3) 2 = (1/3) 2
The solution of the equation x = 1 / (10 1/2 ).
Answer: The area is equal to 1/10.
6. Answer: 1000/1 = 1000.
We prove that it is impossible to get more than 1000. Suppose the number written in figures A b C D . If A b C D > A 0 0
0, A b C D <( A +1) 0 0 0, and the sum of its digits S > A +1, so the quotient is less than 1000.
Another argument.
(1000 A +100 b +10 C + D ) / ( A + b + C + D ) =
= ((999 A +99 b +9 C ) / ( A + b + C + D )) + 1 > ( (999 A +99 b +9 C ) / ( A + b + C )) + 1 =
= ((990 A +90 b ) / ( A + b + C )) + 10 > ((990 A +90 b ) / ( A + b )) + 10 =
= ((900 A +90 b ) / ( A + b )) + 100 > (900 A / A ) = +100 1000
7. Here is an example of a nonconvex polyhedron inscribed in the sphere. Take a regular octahedron, we describe the scope around it. Now
take two of its adjacent faces ABC and BCD , remove them with the edge of BC, and instead add the edge AD and
faces ABD and ACD . Segment BC no longer belongs to the figure, so it is not convex. But at the top of this polyhedron, the same as that of a
regular octahedron. Consequently, the polyhedron inscribed [ 3 Artistic image thus obtaining a non-convex polyhedra see. on the cover . ]
One could proceed as follows. Take a cube and go through it in the "hole" in the form of a cuboid (through the two parallel faces of the
cube). Then eight points, edges "holes", connect the ribs with the nearest vertex of the cube and "lift" ("delete") them into the sphere
circumscribed around the cube.
Clarification of the task N 4
In the classic work on the geometry discussed what Compass-and-straightedge construction possible in principle, but does not discuss the
number of operations required for a particular construction. Meanwhile, this issue may occur discrepancies. Thus, in the classic book
"Elements" of Euclid is considered impossible to measure the distance and compass to move it to construct a circle with any center. But in
Theorem N 2 of this book proved that the transfer of the measured distance is possible, but not in one step, and with the help of some
constructions performed a few steps.
After this theorem can forget about how to transfer the distance - a single step or a few - unless we are talking about the fundamental
possibility of building, not including the necessary constructions.
In modern books on geometry is assumed that no special constructs for transferring the distance is not required. So, in a textbook Pogorelov
said that if given the center and radius, then the circle is considered to be built.Offering this task to the competition in mathematics
Tournament them. Lomonosov, the jury came precisely from this point of view.
Criteria for testing and awarding
For each task (and separately for items and and b of the N 1) Put one of these assessments:
+! +. + - + / 2 - + -. 0
This is the traditional grading system for the Moscow Mathematical Olympiads: + put for the right decision - for the wrong, +! is a very good
decision. + - The right decision with some mathematical "irregularities", + - - the right decision with significant shortcomings. 0 put, if the
task is not recorded in the work. Meaning of the other intermediate results of the jury usually determined separately for each task.
Letters "for successful performance at the competition in mathematics" and all-around points are awarded as follows.
Grade 7 and younger. Speech is considered successful if resolved at least one point (ie the evaluation should not worse than + -; two issues
of the N 1 are counted as separate tasks). e will not be awarded.
Grade 8. Speech is considered successful if solved at least one problem (for which put at least + -). Two items are considered to be the first
task for one task; it is believed that this task should at least + - if for each point is worth at least + -.
e is awarded for at least + - on any item of the first problem.

Grade 9. Speech is considered successful if solved at least one problem (for which put at least + -), and there is at least + / 2 for another
problem (two first task is counted as a single task, for it has at least + / 2 if at least one point there is at least + -).
e is awarded for at least + - on any task.
Grade 10. Speech is considered successful if at least solved by + - at least two tasks, starting with the problem of N 4.
e is awarded for at least one problem, starting with the problem of N 4.
Grade 11. Speech is considered successful if at least solved by + - at least two tasks, starting with the task 5.
e is awarded for at least one problem, starting with the problem of N 4.
Competition in Physics
Assignments
Numbers in parentheses after the specified task classes in which this problem is recommended. Enough to meet the challenges of their class,
and not necessarily all. Solve other tasks is also possible.
1. (6-9) "Magic bridge." Before you two pictures of a wooden bridge (across the river Belaya, Beloretsk city, summer 2002). On the left can
be seen the photograph railing on both sides of the bridge (as it actually is), and on the right - only one. Where and why "share" the
railing? (Railing nobody saw off, photomontage was not used).

2. (6-9) to determine the approximate average speed of the ball during a game of football.
3. (7-11) three-headed Dragon found in the open field (no trees and no do nothing to catch on) a piece of sturdy rope. Dragon can each of their
heads sank his teeth into any place of the rope and pull on this site in any direction with the force of 1 Newton or less. What strength to
withstand the tension rope to Dragon could not break it? Clumsy Dragon can not catch the rope for themselves and get to her feet, wings or
tail.
4. (8-9) to a tripod suspension bob on the wire. Brought up this design on a frost. As a result, the length of the wire cooling decreased bob
rose to a certain height. Where did the energy to increase the potential energy of the sinker?
5. (8-11) Find the resistance between points A and B chains of resistors infinite (the resistance of each resistor R ).

6. (8-11) Sometimes sunbeam almost exactly follows the shape of the mirror, which he "allowed", sometimes only approximately, and
sometimes in the form of sunshine on the mirror does not look like. What does it depend?
7. (9-11) Swimming competition held in the river flow velocity v 0 = 5 km / h. The finish line stretched across the river at an angle of 45 o to
the direction of flow. Athletes placed upstream at a distance of L = 100 m away from the finish line and then may start to float in either

direction. After some time, an athlete who can swim at a speed v , doplyvt to go, if will sail in the right direction?

8. (10-11) The end of the logs tied a rope length h to an anchor and thrown into the design of this body of water. Construct a graph (or at least
a sketch of the graph) according to the tension force of the rope on the water level in the reservoir. Density log is less than the density of
water 0 , the length of the log L , mass M . Rope can be considered as non-extensible, thin and weightless.
9. (10-11) "device to generate electricity from the water." From the water reservoir located two tubes through which the metal of the ring
(positioned around the droplet places) drips. The droplets fall into metal containers, each container is connected to the ring wire, through
which water drips into another vessel. Other electrical connection device is not (receptacles insulated from each other and the water tank). In
the process of water dripping metal vessels become dissimilar electrical charges. Explain how and why such a device works.

Answers and solutions to the competition in physics


1. The right photo was taken by the camera mounted on the horizontal surface of the left railing of the bridge. For a variety of reasons (of
course, individual for each person) this surface (occupying almost the entire foreground picture) may be mistaken for a walking surface of the
bridge.
Among the reasons (the answer to the question "why?") Can be divided into the following:
1) The photo was taken in an unnatural for human perspective (located eyes in the same way as a camera, it is difficult - mingled chin and
nose - if you look at the end of the railing)
2 ) is made of the entire bridge of the same material (wood) is located for a long time in the same conditions (at the water surface, as is clear
from the photograph and problem conditions), so that all surfaces have the same optical characteristics (color, reflectance, light scattering) and
do not differ in the photo on these grounds;
3) the boundary of the shadow on the surface of the right foot railing practically coincides with the right edge of the left railing, which is why
it seems that the shadow falls not on "their place", and on the surface of the left railing, which is why these surfaces are confused.
4) low quality playback pictures of replication tasks risograph conditions (note that when viewing high-quality color originals of these photos
in the problem described visual effect is also observed).
A careful comparison of the photos is easy enough to find that the picture on the left walking surface consists of about 10 parallel boards, and
on the right - only one; And the size of this "board" (width and length) is clearly contrary to the biological concepts of height and diameter of
trees growing on the land.
2. suit any reasonable considerations. For example (from the work of the student, paraphrase): "During the game, the ball is almost never
stand still. Behind him run all the time, but rarely catch up. Run Speed man about 3-4 m / s, then the average speed of a ball about 5 m / c ".
Solutions, the authors of which a little cheated and looked at average speed vector, also recognized the right. This rate apparently is
approximately equal to 0 and no more than
(Distance from the center to the corner of a football field) / (the game)
3. Answer: 1 Newton.

If the rope can withstand the tension of less than 1 H, Dragon can break the rope, stretching her two heads.
Now we show that Dragon can not pull any portion of the rope with a force greater than 1 N. Assume that the Dragon is somehow able to
do. More precisely, the head of the serpent Gorynycha we consider the point from which may be spending a few ropes and modulus of the
vector sum of the tension ropes no more than 1 Newton.
Select the part of the rope stretched with a force greater than 1 Newton (shown in bold segment), and any point on it. Draw point through the
plane perpendicular to the selected portion of the rope (the intersection of this plane with the plane of the figure shown by the dotted line). For
simplicity, we assume that this plane does not pass through any head (if that is not the case - choose another point and perpendicular to the
plane, and so on. Etc .; head only 3, so the fourth time we definitely lucky).
On some sides of the plane would be no more than one head Gorynycha (because he has only 3 heads). And to balance the tension force of the
selected area of the rope on this side of the plane will be impossible. A contradiction.

The above solution (it may seem complicated) is suitable for the case when the Serpent Dragon is allowed to put a rope, tie knots, do the
"web", pass the rope between the teeth (at the suggestion of the tournament), using them as a block, and so on. N. that is parsed is the general
case.
4. Most of the work on the movement of weight on their increased height was made with wire. At the same time, the internal (thermal) energy
resulting from the cooling of the wire decreased. It is logical to assume (as in fact is), that part of this energy went not to heat the atmosphere,
and to perform work on the movement of the sinker.
More subtle (but significant) effect. The energy of the elastic deformation of the wire (in the simplest case is equal to ( k ( x ) 2 ) / 2) as a
result of cooling can be reduced (as a result of a possible decrease in the elongation of the wire x , and (or) the resulting possible reduction
of stiffness k ). In such a case one may assume that the increase in the potential energy of the sinker was spent partly elastic energy of the
wire.
One of the participants of the tournament quite rightly pointed out that the internal energy of the wire is first converted into kinetic
(movement of the wire, but the top end), and then the potential energy of the sinker (as a result of inelastic interaction of wire with a small
weight).
Of course, a list of energy-exchange processes in the system - not exhaustive. For example, we can take into account the decrease in the
potential energy of the atmosphere (the direction of the Archimedes force coincides with the direction of movement of the sinker). This effect
is likely to be negligible. But formally provided nothing written about the mass and density of the sinker - each crucial task can imagine the
weight on his own.
5. Answer: 2 1/2 R .
The easiest way to solve such problems with the endless chain - described as something (for example, R x ) resistance of the whole
chain. Then "unhook" from the string part so that the remaining infinite "tail" coincides with the original scheme. Then replace the tail
resistor R x and calculated by the usual rules of the circuit resistance of a finite number of resistors (which is also equal to R x ). We obtain the
equation R x = F ( R , R x ), root (one of the roots) which is the solution of the problem.
Here is one possible implementation of the above ideas to solve our problem.
1. Let the resistance through the entire chain of R x .
2. Leave the left 8 resistors and the remaining "tail" (completely coincides with the original chain) is replaced by a resistor R x .

3. Slightly pererisuem scheme for convenience denote the letters C and D compounds in the central part of the scheme.

On the wire between points C and D no current flows, so that the wire from the circuit can be removed. (Let towards CD flowing current I .
changed to terminals A and B to the opposite polarity - on the one hand the current must change to - I , on the other - is the same as the
resulting scheme was the same as before the change of polarity. Consequently, I = 0. Another explanation. The resulting circuit is
symmetrical, and the point C , and point D are on the same "axis of symmetry" and therefore have the same potential. If such points to
connect a wire conductor on the current will flow not ).

4. Replace all the successive chain - R - R - at -2 R -. The scheme is quite simple.

5. Finally simplifies

and reduce it to a single resistor


R

2 R * (((2 R * R x ) / (2 R + R x ) +4 R )

2 R * ((8 R

+6 R * R x ) / (2 R + R x ))

2 R + (((2 R * R x ) / (2 R + R x ) +4 R )

2 R + ((8 R

+6 R * R x ) / (2 R + R x ))

(Recall the simple fact that we used the above several times: the circuit resistance of two parallel connected resistors R 1 and R 2 is equal
to R 1 * R 2 / ( R 1 + R 2 ).
6. Solve the resulting equation. Multiply the numerator and denominator of the right-hand side of the equation for (2 R + R x )
2 R * (8 R
R

+6 R * R x )

=
2 R * (2 R + R x ) +8 R 2 +6 R * R

Divide the numerator and denominator of the right-hand side of the equation by 2 R
8R
R

+6 R * R

=
2 R + R x +4 R +3 R
8 R +6 R * R
2

4 R +4 R * R
2

=
6 R +4 R

=
3 R +2 R

R x * (3 R +2 R x ) = 4 R +3 R * R x
3 R * R x +2 R x 2 = 4 R 2 +3 R * R x
2 R x2 = 4 R 2
R x2 = 2 R 2
R x = + 2 1/2 R

R x 2 = 1/2 R

If R > 0 (which is often implied by the use of the term "resistor"), then the solution -2 1/2 R has no physical meaning. However, there are
circuits and devices, which formally executed Ohm's law I = U / R , where the value ofR is not a positive number. The selection of the "extra"
roots in this case is more complicated.
Infinite chains are interesting not only in themselves - they are models of many phenomena in electrical engineering (for example: two
conductors - the chain of resistors with low resistance, insulation between conductors - lateral resistors with "big" resistance), biophysics (eg,
tubular nerve cell, in which the surfaces of the membrane resistance significantly less than the resistance through the membrane), and others.
We have painted a very detailed solution to the problem primarily for those readers who are faced with an endless chain of resistors for the
first time - to allow for good in everything sorted out. From the tournament such a detailed and long decision, of course, is not required.
6. We assume that the mirror is the most ordinary, particularly flat. Also, for simplicity, we assume a flat surface (screen), which falls
"bunny".
The first obvious reason for the possible distortion of the bunny - orientation of the plane of the screen (the surface on which the image is
observed "Bunny"). Tilting the screen from different angles, you can "stretch", "bunny" in different directions.
For simplicity of explanation, will temporarily assume the sun's rays and draw a parallel scheme of education "sunbeam".

This diagram shows a situation where a plane parallel to the plane of the mirror screen (both of which are perpendicular to the plane of the
page). From simple geometric considerations, it is clear that the shape of light spot in this case coincides with the shape of the mirror.
Is it possible to still somehow arrange the screen to form the bunny coincided with the shape of the mirror? We construct the figure circle
whose radius is equal to the length of the segment depicting the "bunny", and the center is the left end of this segment. It is clear that there is

one (and only one, then there are only two for one position of the Sun and Mirror) such orientation of the plane of the screen.

This orientation is shown dashed line - a conditional line of intersection of the plane of the screen (in this orientation) with the plane of the
page (these planes are perpendicular).
It is also understood that (at predetermined positions of the mirror plane of the drawing and the sun) when the plane of the screen will not be
perpendicular to the plane of the drawing, the rabbit stretched on the screen (in the direction perpendicular to the line of intersection of the
plane of the screen and the plane of the drawing).
Above we have specifically used the word "orientation" instead of, for example, the word "location", emphasizing that the screen can be
moved, but you can not tilt (change the orientation in space).
The second reason - the Sun in terms of this problem can not be considered a point source of light, which is obvious if (carefully!) To see him
- the Sun seen from Earth in the form of a disk, and not the point. The sun's rays, therefore, can not be considered in parallel. As a result,
border bunny, that in the first case we consider the lines actually turn out blurry. The width of the fuzzy boundaries may even be larger than
the light spot (in this case, "sunbeam shape on the mirror does not look like").
At each point of the mirror rays hit with the entire visible surface of the Sun in the form of a cone (the vertex angle of this cone is called the
apparent angular size of the Sun and is approximately 0.5 O ) and recorded in the form of a cone with the same angle. On the screen this cone
will give an image in the form of a circle (or ellipse, if the screen is not oriented perpendicular to the axis of the cone). It is easy to see that
every detail on the shape of the mirror image "Bunny" will be "smeared" to the size (diameter) of a circle. Accordingly, all the details of the
mirror shape (including its boundary), the size is much bo'l`shimi diameter of the circle, survive, similar-size - will be blurred, and much
smaller - become invisible [ 5 These parts often can be restored using computer data on the form and distribution of illumination
sunbeams. "By eye" is probably impossible to do, although the people involved in optics for a long time, there is a certain experience in
assessing the original shape of the object to image blur. ]. (Scheme angular size of the Sun, for clarity, are greatly exaggerated)

Obviously, if in the central part of the bunny relate two circles, cones formed from opposite points of the boundary mirror (this corresponds
exactly to the borderline cases, when the contours of the bunny is already heavily eroded, but still follow the contours of the mirror), the angle
between the directions of these points under the supervision of from the bunny just equal to the angle of the cones (simple construction vertical angles), that is, in turn, the angular size of the Sun as seen from Earth. The same angle can be rightly regarded as the angular size of
the mirror when viewed from the "bunny".
Conclusion: if the angular size of the mirror substantially larger angular size of the Sun that form the border bunny fairly accurately follows
the contours of the mirror if these dimensions are about the same - mirrors the contours are reproduced clearly if, on the contrary, the angular
size of the mirror is much smaller angular size of the Sun that form loops "bunny" does not depend on the shape of the mirror [ 6 But may
accidentally coincide with the shape of the mirror, for example, if the mirror - round and Bunny is a blur of rounded shape. ]. (Meaning the
angular size of the mirror and the Sun as seen from the point of the screen where the image of "bunny".)

Note. If we do emission of Solar "Bunny" is not on Earth, but on another planet, the decision should be replaced on the name of the Earth
planet (the angular size of the Sun as seen from the planet will be different, different from the earth) [ 7 And the Sun will always be on location
- in the problem statement says that it is sunny bunny. ].
7. A: L / ( v + v 1 ) = L / ( v + ( v 0 / 2 1/2 )).
Let one of the athletes can not swim (only holds water, v = 0). Then he doplyvt to finish the current, swim a distance of 2 1/2 L for time t 1 =
2 1/2 L / v 0 . His speed of convergence to the finish line is v 1 = L / T = L / (21/2 L / v 0 ) = v 0 / 2 1/2 .
Another athlete who started from the same place, who can swim at a speed v , may in time t swim away from the first "athlete" at a
distance vt in any direction, including towards the finish line.
If at some point in time t after the start of the distance from the first athlete to finish equal to VT , it just means that the second athlete finished
at this point. Composition and solve the equation.
VT = L - v 1 T
(v+v1)T=L
T = L / ( v + v 1 ) = L / ( v + (v 0 / 2 1/2 ))
8. The water level in the reservoir will be denoted by the letter H , the tensile force of the rope - F .
Two sections of the graph are constructed very simply. If the depth is less than the length of the rope ( H < H ), then the log will float freely
on the surface of the water, the rope is not tensioned ( F = 0).
If the depth is greater than the sum of the lengths of rope and logs ( H > H + L ), then the log will obviously be completely under water. In this
case, a force acts on the log Archimedes F Arch = 0 gV log = ( 0 / ) Mg , which is balanced by the gravity Mg and the tension of the rope
force F , i.e.
F + Mg = ( 0 / ) Mg
and
F = F Arch - Mg = ( 0 / ) - Mg = (( 0 / ) -1) Mg
Let us examine now the intermediate case H < H < H + L . Possible position of the beam in this case is shown in Figure (vertical thread is
obvious - have the force of gravity and the Archimedes force acting on the beam, there is no horizontal components, therefore, these forces
can not compensate for the horizontal component of the tension force inclined thread; the stability conditions of equilibrium in the vertical

logs and an inclined position will mainly be understood from the following solutions, for simplicity, we formally define them will not).

Let x - length of the submerged part of the log, - the angle of the log (the angle between the log and the horizontal). Archimedes force acts
only on the submerged part of the log and it is x / L times "more" (less than [ 8 ]There is a view that force must be multiplied by a factor
of x / L and x / L <1. ) Archimedes force acting on all the log, i.e. equal ( x / L ) * ( 0 / ) Mg .
To log does not rotate in a vertical plane about an end to which is tied a rope must be equality of compensating each other moments of gravity
and Archimedes on this end (the other forces acting on the beam and having a non-zero points relative to this point, no).
Mg ( L / 2) cos = ( x / L ) * ( 0 / ) Mg cos
Will reduce the resulting equation (the equation of moments) is obviously not equal to 0, the expression Mg / 2, you get
L cos = ( x / L ) * ( 0 / ) cos
or
L 2 cos = x 2 ( 0 / ) cos
This condition can be satisfied only if or cos = 0 (i.e., = 90 O - timber is in an upright position and can be located under water any portion
thereof), or
x = ( / 0 ) 1/2 L (angle in this case may be any of <90 O , the length of the immersed part of the log, as it turns out, is independent of the
angle ).
In the case of <90 o Archimedes force acting on the beam (submerged portion of the log) is equal to
( x / L ) * ( 0 / ) Mg = ((( / 0 ) 1/2 L ) / L ) ( 0 / ) Mg = ( 0 / ) 1/2 Mg ,
and the tension force of the rope (in this case), respectively, and the difference between the Archimedes force of gravity
F = ( 0 / ) 1/2 Mg - Mg = (( 0 / ) 1/2 -1) Mg

Now it is clear that after the water level exceeds the value H + ( / 0 ) 1/2 L , the log will take a vertical position (position contrary to any
other equation moments) and with a further increase of the water level Archimedes force, and hence strength tension of the rope will increase
linearly as long as the beam is completely submerged; then power will no longer change. The formula for this linear relationship can be easily
obtained, but as in the problem is only required to plot, we simply connect the two corresponding points on the graph by a straight line.
The final schedule is obtained as follows:

9. Just note that the device is assembled on the subject is given in the scheme will not necessarily work (unfortunately collecting).
In addition, it is impossible to come up with an explanation that fits all devices work: it would have to consider all possible design details of
each specific device and the external environment in which it is located.
Therefore, proceed as follows: first, we present a simple, but very superficial and inaccurate explanation, and then - the most interesting and
not too complicated and clarifying.
A simple explanation. Suppose the right (in the figure) of the bank and, accordingly, the left metal ring (connected to the jar conductor) have a
positive electric charge, while the left bank and right ring - negative electric charge. (In fact, these charges arose, we are not going to
understand).
Water - the conductor located therein electric charges (positive and negative) that can be moved within the volume occupied by the water (but
this amount can not leave since it is limited by the insulating walls of the vessel and the air, which is also a poor conductor of electricity).
Under the influence of electrostatic forces, positive charges are attracted to the left of the ring (and, accordingly, are collected in the water
located near the right end of the tube), and the negative charges on the same reason arranged near the left end of the tube. The phenomenon of
redistribution of charges in a conductor as a result of interactions with other charged bodies is called polarization.
As a result of a drop of water dripping from the right tube, is charged positively, and dripping from the left tube - negatively charged.
Dripping from the right tube positively charged droplets pass through a negatively charged ring and then fall into the positively charged bank
(the attractive force drops to the oppositely charged ring and repulsion of like charges banks overcome the force of gravity).
Similarly, negatively charged droplets dripping from the left tube to fall to the left (negatively charged) jar.
Thus, in this process there is an accumulation of charges in metal cans (and associated conductors rings). As the gain of the charge
polarization occurs in the tank water, and consequently, an increase in the charge of the newly formed droplets (compared to the charges
previously formed). That is, the process is samonarastayuschim (or, at least, self-supporting).
Clarification and explanation.
1. Explaining the reasons for the redistribution of the charges contained in the tank water, correct to speak of electrostatic potentials generated
at the appropriate places (or rather, the potential difference between the left end of the tube and the end of the right tube). When banks (as
well as rings and connecting their conductors) already accumulated enough large charges, these charges and are major contributors to the
corresponding potentials.
2. Up to this point the main contribution to the potentials create other charges, the occurrence of which in each case due to accidental causes
(which physicists call fluctuations). The potential difference may be due to ground potential, the charge on the experimenter's clothing and
spectators, work from a nearby electronic equipment, flying through the air charged dust particles, and so on. N.
3. At first glance, it may seem strange that in the process of "symmetric" device on one of his "balanced" often appears positive charge and
the other - negative. (Who "decide" where to put the sign of the charge?)

As explained above, this "solution" depends on many random factors. Moreover, almost absolutely symmetrical design make possible. And if
it turned out - to create a completely symmetrical external conditions still can not.
4. As mentioned and explained above, arose a small charge redistribution can spontaneously grow. But we do not claim that it will last forever
and always. Before the device to "earn" the charge distribution may change to the opposite, and more than once.
5. In any physical experiment is interesting not only theoretical foundation, but also the results. That is interesting to check whether at banks
and connected them with the metal parts of the device are accumulated electric charges. We recommend for this purpose a few simple ways.
a) Hang on a thin piece of wire to a metal part (a charge which we want to test) two light piece of metal foil ("petals"), for example, made of
candy wrappers. When you have a big enough charge, these petals will repel each other. Petals should hang freely on the wire, for example,
they can be pierced with holes and wear on the wire. Also suitable piece of Christmas "rain", which can be pierced in the middle (the petals
are two dangling ends).
b) Connect the cans with wire, then cut the wire, leaving a gap between the cut pieces no more than 1 mm. In the dark, you can see how
through this gap occasionally skips a spark.
c) Carefully watch the drops. Their trajectory as a result of the forces of attraction to the rings and repulsive forces from cans can noticeably
bent.
d) Connect the cans of neon (gas discharge lamp) - it will flare up periodically. Note that it is necessary discharge lamp, which "lights up"
only when a certain voltage, and before that an insulator. Ordinary incandescent bulb is not necessary - it is a guide and not dissimilar charges
will accumulate. A similar problem can occur with electrical devices (such as regular school voltmeter).
In any case, the experiment requires patience - sometimes noticeable effect does not appear for a long time - 5-10 minutes or even
longer. Why is this happening? One plausible hypotheses - can not charge the device settle flying around dust particles of opposite sign
(thereby reducing the charge). Accordingly, it is necessary to wait until the device "catch" flying around the charged dust.
Described in the problem of adaptation is sometimes called "Kelvin water dropper", on behalf of the alleged inventor. Several complicating
the design, based on the proposed scheme in the problem can be assembled device generating voltage of several thousand volts; such devices
previously used in physical experiments as simple (but not perfect), the high voltage generator. A more detailed description we do not give - it
is quite difficult to "school" books, and also the consequences of experiments with high voltage can be harmful to health.
Competition in Chemistry
Assignments
After each task numbers in parentheses indicate which classes she recommended. To solve problems not of their class are allowed, but the
tasks for the younger class than yours, will be assessed fewer points. It is proposed to solve the problem of 2-4. (By the terms of the tasks
were applied and the solubility of the periodic table.)
1. (7-8) The manual that came with the set of "young chemist", described the following experiment. In an evaporating dish pour 2-3 ml of
alcohol or acetone, set fire to it, and then pour the flames with water from a prepared tube. Immediately turn off the flame. Then repeat the
same experience with 2-3 ml of benzene or hexane. At this time, the addition of water the flame does not go out, and the liquid continues to
burn. How can we explain this result?
2. (7-9) Mom asked Masha prepare the marinade. Mary was diluted beaker (200 ml) in 1 liter of vinegar water. Instead of vinegar she
accidentally spread in water 200 ml of vinegar. When Masha encountered an error, some of it is diluted with water and the resulting solution
was the marinade in the exact amount and that the concentration, as requested by my mother. How the resulting solution and the water was
taken as? Assume that vinegar containing 7% acetic acid, vinegar essence - 70%, and the density of said substances is equal to 1 g / ml.
3. Fill in the blanks in the equations of chemical reactions.
7-9.
1) = CaO + H 2 O
2) + = AgCl + NaNO 3
3) + 3O 2 = 2 CO 2 + 3H 2 O
4) + C = 2CO
5) BaO 2 + + H 2 O = BaCO 3 +
9-11.
6) 2 + 2 = I 2 + 2FeCl 2 + 2KCl
7) + 8 = 3MgCl 2 + 2NH 4 Cl
8) + SO 2 = NO + 9) 2CrCl 3 + 3 + 6H 2 O = 2 + 6NaCl + 3H 2 S 10) 4 + 2Ca (OH) 2 = Ca (NO 3 ) 2 + Ca (NO 2 ) 2 + 2H 2 O 11) + 5KI +
3H 2 SO 4 = 3 + 3I 2 + 3H 2 O

4. (9-10) After school Misha went to Pete and gave him a blank sheet of paper. This is - a letter written in invisible ink, - he said. Hold it over
a cup of ammonia, and you will see what is written there. Peter went home, took a first-aid kit ammonia, pour it into a cup and held it over her
sheet of paper. On paper, the text appears really crimson. And it was the decision of the home math problems! Petya ran for a notebook, to
rewrite the solution there. But while he was running, the inscription disappeared. Before it was again a clean sheet.
(1) What are the "ink" was written in the letter?
(2) What Peter had to do to sign reappeared?
(3) What do I need to do to sign appeared no longer disappear?
5. (9-10) Metal mass was treated with 13.0 g of dilute nitric acid excess. Metal solution without gas evolution. To this solution was added an
excess of potassium hydroxide, and the gas was allocated 1.12 liters (STP).Determine what kind of metal was taken.
6. (9-11) It is known that the carbon atoms are able to connect with each other, forming a chain -CCC- this is causing extreme diversity of
existing organic compounds. What other types of substances you know where the atoms (not necessarily identical) to form long chains? What
types of chemical bonds they sold? Give examples.
7. (10-11) to one mole iodalkana acted alcoholic solution of potassium hydroxide to give a mixture of two isomeric alkenes in the ratio of 7:
1. Mass major reaction product (prepared in large quantities) of 49 was defined to structural formula of the starting compound, assuming that
the reaction proceeded quantitatively. Answer justify.
8. (10-11) according to one of the existing methods of quantitative determination of phenol in water to a sample acidified with hydrochloric
acid, is added solutions of potassium bromide and bromate, and then potassium iodide solution. Catching iodine is titrated with sodium
thiosulfate. What chemical reactions inherent in the scheme determination of phenol? Write their equations. Whether the method is applicable
to a substituted phenols?What conditions must be met to this method gave accurate results?
Problem solving competition in Chemistry
Challenges and solutions for the competition in chemistry prepared Sofia Vladimirovna Lushchekina and Zinaida Svitanko.
1. The alcohol is dissolved in acetone and water, wherein the concentration of combustibles is reduced and the flame extinguishes. The fact
that alcohol and acetone soluble, it was obvious not all students, although quite common in everyday life all sorts of alcoholic solutions, and
acetone may be familiar to those of you whose mothers or older sisters are based on liquid acetone to remove nail polish.
Hexane and benzene insoluble in water and as their density less than the density of water, the water sinks to the bottom, and the organic
substances remain on the surface and continue to burn freely. Some sixth graders wrote about rainbow film of gasoline on puddles, older
students should be aware that the hexane - a nonpolar substance and therefore hydrophobic.
Common misconceptions:
alcohol and acetone denser than water, so they sink to the bottom, and the water floating on top of and
blocks access of oxygen;
alcohol and acetone are reacted with water, and the products of this reaction are off;
alcohol and acetone containing less combustibles (have lower flammability) than gasoline and hexane.
2. First, recall that 1 liter = 1000 ml.
Masha was to get 200 + 1000 = 1200 ml of solution containing 200 0,07 (7%) = 14 g CH 3 COOH (chemical formula of vinegar), but
instead she received the same volume of 200 0,7 ( 70%) = 140 g vinegar. It is understood that the tenth part of this solution (120 ml)
contains 14 g of vinegar required. It remains to obtain the necessary volume to add 1200-120 = 1080 ml of water.
Many students offered all the resulting solution was diluted ten times. One can only wonder what would have made Masha and her mother
with a 12 liter marinade.
A few words about the accuracy of our measurements: if you add 120 ml of this solution is not 1080 ml, and a liter of water, then nothing will
happen: the marinade will be a little more robust [ 9 concentration will increase by about 4% compared with what happens if measure volume
of water strictly on the calculation. ], which is not terrible at home. Accuracy of instruments that we use in everyday life, low, and otmerivaya
liter using the measuring cup, we can easily be mistaken for 50 or even 100 ml, the more that is sure to be part of the solution on the floor, in
the form of fly spray.
In laboratory practice, all is different. Chemists often have to solve the problem for the breeding of various solutions. Virtually all laboratory
experiments (especially biochemical) used small amounts of substances, it requires a very high accuracy. For example, analytical balances
allow to weigh to the nearest tenth of a milligram. Many laboratory pipette (samplers) allow you to type microliters (and especially expensive
and few nanoliters) of liquids. In such circumstances, the work in the first place in importance goes way of dealing with dimensional and
utensils. For example, in the room where the weigh-in, try to maintain a certain temperature, and how to lower the pipette with a few
microliters of the precious liquid in a test tube, so that our material is not smeared on the walls and flew away in the form of spray - a science.

3. To solve this task, first had to remember the law of constant composition. E. The number of atoms of each element in the left and right
sides must be the same. Since all the coefficients of the reactions are given, this task is not so complicated.
With a simple calculation you can immediately find out about what atoms are and in what quantities contained in the missing molecules. Then
you need to understand what kind of reaction (redox, exchange, ...) and what substances (relating to any class) must be in place omissions.
And finally, you must correctly write the answer.
1) Ca (OH) 2 = CaO + H 2 O
Many students noticed that the starting material contains one formula unit of two of Ca and O and H, and writing and CaO 2 H 2 or
O 2 CaH 2 ; is forgiven students who have just started to study chemistry or not yet studied it at all, but the older must wonder whether there is
substance, which they write.
2) AgNO 3 + NaCl = AgCl precipitate + NaNO 3
This is a very common reaction to the quality of chloride ion and silver ion. Difficulties were among those who do not know that these
substances are composed of ions Ag + , Na + , Cl - , NO 3 - .
3) C 2 H 6 O + 3O 2 = 2CO 2 + 3H 2 O
In this task, it was necessary to simply count the number of carbon and hydrogen atoms are given in the right-hand and left-hand side:
left

right

the difference

6O

2C, 7O, 6H

2C, 6H, O

Empirical formula C 2 H 6 O corresponds to two substances: ether CH 3 OCH 3 and alcohol C 2 H 5 OH.
4) CO 2 + C = 2CO
This item is similar to the previous one, because once you will notice that an unknown substance contains one molecule of carbon and two
oxygen molecules. This reduction reaction of carbon dioxide to carbon monoxide, it usually happens in the furnaces where not all the fuel
burned, and the cover is already closed (blocked the access of oxygen), and there was only CO 2 . The resulting carbon monoxide is very
dangerous, it is imperceptible to the human depresses respiratory function. Many people who are unable to properly use the stove, died from it
(carbon monoxide poisoning).
The decisions of the common variants of O 2 + 2C = 2CO, students are summed ordinary negligence.
5) This quest led bolshi'e difficulties. Not all noticed that BaO 2 - contains peroxide and oxygen in the oxidation of -1. Correct answer:
BaO 2 + CO 2 + H 2 O = BaCO 3 + H 2 O 2 .
Many offer BaO 2 + CO + H 2 O = BaCO 3 + H 2 O.
Direct calculation shows that the starting material is missing can be CO, which must be reacted with BaO 2 .
This solution contained two problems: CO - non-acid, and chemically inactive with the oxide solution is passed through BaO 2 will not
interact with it. However, many students can not yet know or do not remember, the more the reaction and an electronic balance formally
correct.
C

+2

-1

-2 e - -> C
+ e -> O
-

+4
-2

|*1

- Restorer

|*2

- Oxidant

The second problem, and it could be noted in any consists in the fact that a missed product - water, and is probably not involved in the
process; why did it then generally written in the reaction?
Note that if any substance is necessary in the reaction, but it is not consumed (this applies primarily to catalysts and solvents), it does not
indicate the same time in the right and left side, and is placed above the arrow, as the reaction conditions. By the way, in parentheses next to
the substance usually write concentrated or diluted substance, ground or suspension, and indicate the state of aggregation as a subscript. For
Example:

In the chemical literature can be found quite a variety of symbols and rigid rule, there is only one thing: it is necessary to write the equations
and reaction schemes so that the record could easily understand any competent chemist.
Let's go back to our reaction. When adding CO 2 to a solution of BaO 2 , the equilibrium [ 10 This equilibrium takes place in the interaction of
barium peroxide and cold water and warm water (50-60 O C), this process is easily goes toward the formation of peroxides. ] BaO 2 + H 2 O
<=> H 2 O 2 + BaO, rather
BaO 2 + 2H 2 O <=> H 2 O 2 + Ba (OH) 2
is shifted to the product side, so as to interact with BaO CO 2 and removed from the reaction zone in the form of an insoluble carbonate.
6) 2FeCl 3 + 2KI = I 2 + 2FeCl 2 + 2KCl
To solve this problem, it was necessary first to think what type of this reaction. Since the reaction products are included in the iodine
oxidation 0, can immediately assume that this redox reaction, because the reaction in which I 0 does not change the oxidation state (except for
formation of a blue complex with starch iodine) are quite rare and considered in this case is not worth it.
In the next step will try to find an item that could oxidize or restore iodine (criterion will serve electronegativity). Consider all the options:
K . In theory, potassium metal reducing agent suitable for the role I + . But such a reaction is practically impossible to obtain I 2 , because it
will be held simultaneously with the formation of I - .
Cl . Chlorine may be suitable for the role of the oxidant (if included in the right side of the zero or positive oxidation state), but since the
chlorine (especially chlorates) - very strong oxidant in foods can not be formed Fe 2+ , because iron will be oxidized in the first and to Fe 3+ ,
and iodine almost certainly will move in a positive oxidation state.
Fe . Iron can be both oxidizing agent and a reducing agent, as part of the right side of the intermediate oxidation state. However, in this
reaction, metallic iron can not restore to the molecular iodine from the iodine, because I + insufficiently strong oxidant. But the option to
restore the iron three iodine -1 fits very well as Fe 3+ soft enough oxidizer to obtain I 2 .
In order to correctly write the starting compounds, write electronic balance, counting atoms available to us, because we are given as a guide,
all the coefficients of the reaction:
Fe

+3

+ e - -> Fe

- e -> I

|*1

+2

- Oxidant
*2

-1

|*1

From a set of cations and anions (2Fe


iodine.)

+3

, 2I , 2K , 6Cl ) can "create" only 2Fe

- Restorer
+3

Cl 3 and 2KI. (On 2Fe +3 I 3 simply "not enough" of

7) Mg 3 N 2 + 8HCl = 3MgCl 2 + 2NH 4 Cl


This is not an ordinary exchange reaction. The starting material - dinitrid trimagnesium. This yellow-green crystalline solid. Besides hydrogen
N 3- forms compounds with alkali and alkaline earth metals. These compounds are highly reactive, are relatively stable only nitrides of lithium
and magnesium because of their ionic radii are not very different from the ionic radius of nitrogen. Lithium nitride obtained under normal
conditions by reacting lithium metal and nitrogen. For magnesium nitride necessary heating. Note that, in liquid ammonia, these compounds
behave as oxides in water (in this case, similar to oxygen, nitrogen). But still, they are easily decomposed by the action of various agents
(acids, water and the like. D.), Ammonia (ammonium salt) and the corresponding metal salt.
8) NO 2 + SO 2 = NO + SO 3

In this task, it should be noted that the second unknown product must contain sulfur and oxygen. We are aware of the following oxygen-sulfur
compounds [ 11 We consider only connections to one sulfur atom, but in general, sulfur may form a rather long chains of its atoms, thanks to a
variety of sulfur compounds is very large (see. Task 6 ). ]: H 2 SO 3 , H 2 SO 4 and SO 3 .
Sulfurous acid is not suitable, since in this case the reaction should occur transfer water molecules from a compound of type NO * H 2 O to
SO 2 , but non-acid oxide NO, ie. e. can not attach water to form acids.
In the case of H 2 SO 4 and SO 3 takes place redox reaction. Sulfur loses two electrons, and since we have only one nitrogen atom, then it takes
them:
S

+4

-2 e - -> S

+4

+2 e -> N
-

+6
+2

|*1

- Restorer

|*1

- Oxidant

Hence, the initial product - NO 2 . Option hydroxide (formation of H 2 SO 4 ) is impossible, since NO 2 - also non-acid oxide.
9) CrCl 2 + 3Na 2 S + 6H 2 O = 2Cr (OH) 3 + 6NaCl + 3H 2 S
It's a simple exchange reaction. We write the equation with gaps in its ionic form: 2Cr 3+ + 6Cl - +3
3S -

+ 6Na + + 6Cl - + 6H + +

+ 6H 2 O = 2

As in all other areas of this problem, look closely at what is missing (it is necessary to recall that H 2 O <=> H + + OH - ):
left
6Na

and 3S

right
2Cr

3+

and 6OH

Many wrote that chromium enters the Cr 2 O 3 . From the solution of chromium hydroxide is precipitated in crystalline form Cr
(OH) 3 * n H 2 O, and then with a long distance may gradually lose the water of crystallization (this is called "aging" precipitate chromium
hydroxide thus loses reactivity). A strong heating is transformed Cr (OH) 3 -> T O Cr 2 O 3 + H 2 O. Since we did not find the "excess" of water
in the products, it can be said that the chromium oxide in the reaction is not formed.
10) 4NO 2 + 2Ca (OH) 2 = Ca (NO 3 ) 2 + Ca (NO 2 ) 2 + 2H 2 O
At this point, we have only one missing ingredient, so it is clear that it contains all the atoms that are in the right-hand side, but are not in the
left, with the law of constant composition. It is therefore advisable to calculate how many atoms of each species in each part, and to
equalize. So, let the missing substance contains x N atoms, y O atoms, z - H and u - Ca. Then:
4xN

0N

2N

2N

0N

|x=1

4yO

* 2O 2

2 * 3O

* 2O 2

2O

|y=2

4zH

2 * 2H

0H

0H

2 * 2H

|z=0

4 u Ca

2Ca

1Ca

1Ca

0Ca

|u=0

ie, missed connection - NO 2 .


11) KIO 3 + 5KI + 3H 2 SO 4 = 3K 2 SO 4 + 3I 2 + 3H 2 O
This reaction soproportsionirovaniya. Reactions of this type are very characteristic of elements having more than one stable oxidation
state. Facing in the reaction mixture, the atoms of the same element having a high oxidation degree, tend to "split equally" their electrons and
can form depending on the stoichiometric ratios and conditions, one or more intermediate compounds, for example:
H 2 S + H 2 SO 4 (conc.) -> S (pellet) + SO 2 (g) + 2H 2 O,
H 2 S + 3H 2 SO 4 (conc.) -> (boiling) 4SO 2 (g) + 4H 2 O.
In addition, the possible backlash: disproportionation. For example, nitrogen dioxide disproportionation under the action of alkalis:
2N +4 O 2 + 2NaOH (dil.) = NaN +3 O 2 + NaN +5 O 3 + H 2 O.
4. Several students suggested the most correct solution to this problem. They felt that Pete would be more useful not khimichit home in the
kitchen, and to solve domestic problems in mathematics independently. Tournament organizers fully endorse this position!
Nevertheless, it is necessary to deal with the chemical aspect of this criminal cheating:

1) The letter was written phenolphthalein. It's pretty common indicator that in acidic and neutral media is colorless, and at pH 8,2-9,8
becomes crimson-red (transition interval indicator [ 12 pH = -lg [H + ], usually expressed as the concentration of hydrogen ions - index the
acidity of the solution. Alternatively, if it another way, pH = x means that the concentration of the ions H + in the solution is 10 - x mol / liter.
In neutral conditions pH = 7, under acidic conditions pH <7, in alkaline, respectively pH> 7 ]), this is because in an alkaline medium as a
result of redistribution of electron density in the molecule formed quinoid structure (chromophore), which gives the compound of crimson
color.

In strongly alkaline medium (pH ~ 13) phenolphthalein loses its color and becomes colorless again.
For those who have not studied or have just started to study organic chemistry, explain: in the chemical formulas of the "angle" between the
bonds, which gave no letters (as well as "fork", "crossroads" and just end) conventionally indicates that This site is a carbon atom; if the
valence of the carbon atom at the expense of all the designated chemical bonds is equal to n <4, it is assumed that this carbon atom more
connected (4- n ) hydrogen atoms. Cosi only carbon and hydrogen, the associated structural formulas of organic compounds [ 13 This decrease
is acceptable, but not necessarily. Designation of C and H atoms are not considered an error, but in most cases reduces the "readability" of the
formula and increases required to play this formula area of the paper. We offer you a full draw (in the study of chemistry is still useful to
perform at least once), some of the structures shown here phenolphthalein and verify this yourself. ], other atoms (oxygen, nitrogen, and so
on. d.) Indicates required. Please note that when we write the formula of inorganic compounds, we rarely specify its structure, but clearly
write down its composition in organic matter is much more important to the structure, and the exact composition can be determined by
valence.
When Peter held the letter above the cup with ammonia, soaked sheet para'mi ammonia. When this occurs that ammonia reacts with water
adsorbed on the fibers of the paper:
NH 3 + H 2 O -> NH 4 + + OH - .
Wednesday in the thickness of the paper becomes alkaline, and the indicator changes color.
The decisions, especially those who wrote the younger students were quite common "food" versions: milk and lemon juice. Probably due to
the fact that in the literature, these substances are often used for cryptography. Just do not understand why the inscription was shown by the
action of ammonia vapors.
2) From this point virtually no problems arose, many rightly offered to hold a piece of the cup with ammonia again, but the full answer at this
point include an explanation of why the inscription disappeared while Peter ran for a notebook. The fact that the ammonia is evaporated and
phenolphthalein rapidly returned to the former form.
3) Phenolphthalein changes its color under the influence of any base. Ammonia is bad that is volatile. But almost all the other reasons not
have this disadvantage. Therefore, the inscription appeared to no longer be removed can be treated with an alkali solution of the ordinary,
such as NaOH.
But NaOH and many other hydroxides have their drawbacks - they are too "caustic" - require careful handling: Causes burns in contact with
skin, mucous membranes; Eye contact - this is a serious injury.
In addition to the experimenter NaOH "erodes" and plain paper (it "spreads", and after drying crumbles into dust). Therefore hydroxides for
our problem is likely not appropriate. Besides, they are not used in everyday life (on the same reasons described above). Peter probably will
not find such materials at home.
The alkaline environment may create other, including a "household" substances, such as drinking solution (NaHCO 3 ) or washing
(Na 2 CO 3 ) in the soda water. This occurs as a result of hydrolysis:
Na 2 CO 3 (dil.) = 2Na + + CO 3 2- , CO 3 2- + H 2 O + OH NaHCO 3 (dil.) = Na + + HCO 3 - , HCO 3 - + H 2 O + OH The reaction of thus formed in solution the ions OH - molecules formed phenolphthalein dyed the form of these molecules.

The soda solution should be applied carefully so as not to wash away the text (for example, a brush, or attach a letter to the moistened with a
sheet of paper or a piece of cloth). Dip entirely in solution of soda letter itself - not a good idea.
When checking solutions counted as correct any answers, where it was proposed to act on a piece of non-volatile base. Most of the
participants of the tournament, guess until then, wrote that piece of alkali necessary to moisten, and despite the fact that in practical terms, this
option is not very successful (see. Above), it estimates the total score.
Very reasonable, but, unfortunately, it is not the chemical had an offer of several participants to circle the inscription pen or pencil, place in a
bag and even laminate.
Those in the first paragraph have been proposed as ink milk or lemon juice, are offered to expose a piece of heat-treated (iron the iron and
hold over the lamp, flame stove ...).
A common variant was dipped in an ammonia. The problem of the rapid evaporation of ammonia can solve this, but the inscription will wash
off, paper razmoknet, and rewrite the problem in mathematics Peter will not be able.
5. Answer: Zn (zinc).
The interaction of nitric acid with metals depending on the acid concentration and the properties of the metal may be released NO 2 , NO,
N 2 O and NH 3 . In the latter case, the gas would not be released as ammonia will react with nitric acid (which, although strongly diluted, but
there is sufficient) to form an NH 4 NO 3 . We write the reaction of the metal [ 14 We assume a monovalent metal. Since reaction rates defines
the number of electrons moving from one atom to another, in the case of a divalent metal, we can multiply the molecular weight, we obtain
two to trivalent - three. You can check it yourself, write the equation of the reaction with the corresponding coefficients and recalculate the
molar mass. ] M with dilute nitric acid:
8M + 10HNO 3 = 8MNO 3 + NH 4 NO 3 + 3H 2 O.
Gas, which is released under the action of alkali - ammonia:
NH 4 NO 3 + KOH = NH 3 (gas) + KNO 3 + H 2 O.
Of these two reaction equations it is seen that one mole of gas have 8 moles metal. Ammonia was allocated 1.12 / 22.4 = 0.05 mol metal
hence was 0.4 mol and the molar mass M if univalent metal, is 13.0 / 0.4 = 32.5 g / mol. But with such metal atomic mass not found in nature,
has a similar mass of sulfur (32), but for obvious reasons, can not be considered the correct answer. Divalent metal must have a mass of 32.5 *
2 = 65 g / mol. This has an atomic mass of zinc, and this is the correct answer. Suitable trivalent metal (molar mass = 3 * 32.5 97.5 g / mol)
are not.
The same response could be obtained without considering cases of valence and using equivalents, though in essence a decision would remain
unchanged.
6. This problem does not involve a complete solution. We give only some of the most typical examples.
Some simple matter to form long chains of covalently bonded atoms. Typically, many such substances have allotropic [ 15 Allotropy existence of the same chemical element in the form of two or more simple substances of different structure and properties, ie. n. allotropic (or
allotropic) modifications. ] and modifications of the compounds are also prone to the formation of polymers.
Sera. Typically Elemental sulfur consists of eight atoms which may be variously packed in the crystal lattice. In addition, there is an unstable
plastic sulfur - chaotically twisted zigzag chain length. In sulfur compounds often form long polymer chains in which the units are connected
via oxygen atoms or sulfur itself. The composition and structure of these compounds depends on the preparation conditions.
Bor. He has more than 10 allotropic modifications. Boron atoms are combined in groups B 12 having the shape of an icosahedron icosahedron.

Boron hydrides (boranes) form a rather complex structure connection with multicenter bonds.
Particularly interesting structures form oxygen compounds of boron. It can be flat zigzag chains that are connected with each other tetrahedra,
"cell", and so on. D.
Phosphorus. Phosphorus has a large number of allotropic modifications, of which the most important - the white and red phosphorus. In the
vapor phase it is present as a molecule chetyrhatomnoy P 4 , is formed during the condensation of white phosphorus P 4 , which during
prolonged storage, or heating the polymer becomes - red phosphorus. Another allotrope (or allotropic) modification - black phosphorus which has a structure similar to graphite, consists of corrugated layers. All modifications phosphorus associated with three of its
neighbors. Phosphorus may also form a chain (not very long) and oxygen compounds.
Silicon. Simple substance has a diamond modification. Silicon compounds with oxygen to form a large class of compounds oksisilikaty. They are based on the structure shown in Fig.

Each oxygen bonded to two silicon atoms and each silicon - with four oxygens. The mutual spatial arrangement of these atoms can be very
different, moreover, a structure oksisilikatov includes various ions. And together they form a very long and branched chain network and
spatial structures.
Biopolymers. Many biologically active substances have the form of long chains. Also C-C bonds due to meet nitrogen, for example, belka'h,
is a long sequence of amino acids, they are joined together by a peptide bond.

The figure depicts a dipeptide - two amino acids, R 1 and R 2 - radicals, which determine biological (biochemical) properties of amino acids.
Although the body is found only 20 species (20 different radicals) amino acids [ 16 in the tissues of some organisms are also other (unusual)
amino acids - see. p. 73 (N 7 question in biology). ], they form very long chains, the protein can comprise up to 10 thousand amino acids
(although the reservation that they all form a single long chain, usually a few long chains (two to ten), connected to each other by electrostatic
interactions or covalent linkages via disulfide -SS-).
Another example of biological macromolecules - molecules that carry genetic information - DNA (deoxyribonucleic acid) and RNA
(ribonucleic acid), the relationship between adjacent links of very long chains carry phosphate (acidic phosphoric acid residue) and sugar
(deoxyribose and ribose, respectively).
Scope and purpose of this book does not allow any detail to describe the structure and function of proteins and nucleic acids, as well as a list
all biologically active molecules, is a long chain. Your familiarity with them will begin in the course of general biology. Just to say that these
long chains of atoms make up the chemical basis of all life.
Hydrogen bond. In all the above examples obrazovyvaniyu long chains served covalent (both polar and non-polar) connection. Another
example of communication - hydrogen. Hydrogen bond - is the electrostatic interaction of hydrogen carrying positive charge, with the atom
carrying a large negative charge. For example, water molecules do not form a chain, and the entire lattice of the molecules. Of course, these
relations are very fragile, but, nevertheless, they have a major effect on the properties of substances. This can be seen by comparing the
boiling point of water and hydrogen sulfide and other hydrides of group VI elements - the water boiling temperature abnormally high.
Note that HF between molecules very strong hydrogen bonds as hydrogen fluoride electronegative, but HF boiling point much lower than that
of water. This is due to the fact that hydrogen fluoride molecules "stick" to form dimers, but no branched chain linked molecules like H 2 O.
For reference, a boiling point of these compounds under atmospheric pressure.

H
H
H
H

100.00

-60.35

Se

-41.4

Te

-2

HF

19.52

HCl

-85.08

HBr

-66.8

HI

-35.4

C
C

7. cleavage reaction to form hydrogen halide elimination diene call. There are several mechanisms of this reaction, we sketch only one of
them.

And a little more about the notation used in the chemical literature: usually B: or B - denotes the base (from the English basis - base), in this
case, OH - , and the product BH, respectively, water. Acid conventionally denoted by A + (on acid - acid). In square brackets marked with = /
= denote intermediate, which in its pure form does not exist, but it reflects the mechanism of the reaction. - and + denote, respectively, an
excess and a lack of electron density [ 17 Since the electron - negatively charged particles excess electron density gives the atom a partial
negative charge, and the lack - a partial positive. ] arrows - direction of its redistribution.
Since we have two alkene isomers, it can be assumed that the source iodalkan - secondary [
one that has exactly two carbon-carbon bond. ].

18

iodine atom linked to a secondary carbon, i.e.

Calculate the weight of the product: we know that 7/8 mole product weighs 49 g, that is, the molecular weight of this product is 56 g /
mol. Total gross formula of alkenes C n H 2 n . We construct a simple equation: 12 n +2n = 56, n = 4, m. e. obtained alkene C 4 H 8 . Hence, the
starting compound had bruttoformulu C 4 H 9 I. Simple search can show that the secondary isomer with the formula only one: CH 3 -CHICH 2 -CH 3 (if you have got two, then check to see whether they are symmetric).
An interesting question, which of alkenes was the main product and get in their increased numbers. In organic chemistry, there is a rule
Zaitseva, in which hydrogen is more likely to split off from the atom C, which was due to minimal cleaving atomic number N. This is
explained by the fact that in this case a more stable intermediate.
More details about the chemical reactions that match (or, conversely, do not comply with) the rule Zaitsev know those of you who will study
organic chemistry at the university.
8. Phenol - a toxic substance that may be contained in the wastewater industry. To ensure that waste water is well cleaned, you need to be able
to identify them in small concentrations of harmful components, including phenol. The method described in the problem, based on the
reaction of phenol with bromine, which runs as follows:

We know that benzene bromination comes only in the presence of a catalyst (e.g., FeBr 3 ) and wherein only one hydrogen atom on the
benzene ring is substituted by a bromine atom. In the case of phenol reaction proceeds very easily and immediately gives
tribromproizvodnoe. This is due to the activating effect of the hydroxyl groups on the benzene ring. An unshared pair of electrons belonging
to the oxygen atom in the ring is pulled, whereby the electron density in the ring is increased. Since the bromination reaction proceeds by
electrophilic substitution mechanism, increasing the electron density for the favorable reaction. Thus it can be seen that the hydroxyl group
does not activate any of the provisions in the benzene ring, namely positions 2, 4 and 6. The substance 2,4,6-tribromophenol - solid
precipitates.
Bromo necessary for bromination of phenol in the reaction mixture formed by reacting potassium bromide and bromate in an acidic medium:
5KBr + KBrO 3 + 6HCl = 6KCl + 3Br 2 + 3H 2 O

The excess bromine remaining after coupling all of phenol is reacted with potassium iodide
Br 2 + 2KI = I 2 + 2KBr
And finally catching iodine is titrated with sodium thiosulfate. Thus the reaction proceeds according to the equation:
2Na 2 S 2 O 3 + I 2 = Na 2 S 4 O 6 + 2NaI
Compound Na 2 S 4 O 6 , which is thus formed, called sodium tetrathionate. The reaction of iodine with sodium thiosulfate often used in the
redox titration, since it proceeds rapidly and quantitatively. Furthermore, the end point is easily detected as iodo intensively with the starch
forms a blue-violet colored complex. Before the titration the solution was added a few drops of starch solution, thus immediately appear blueviolet color as the solution contains iodine. As the addition of Na 2 S 2 O 3 , iodine gradually consumed. At that moment, when the added
amount of thiosulfate exactly equivalent to the amount of iodine, the reaction takes place completely (iodine is left), and the color
disappears. This is the endpoint. By the number of solution of Na 2 S 2 O 3 consumed by this time, counting the number of iodine, which is
obtained in the reaction with bromine, and on it - the amount of bromine that is left after reaction with phenol. Since we can determine the
starting amount of bromine in the solution which is obtained from bromide and bromate, but now can be found from the difference between
the amount of bromine which has gone the reaction with phenol, and finally, the amount of phenol. Of course, there is no need to spend each
time this whole sequence of calculations. In developing the methodology usually immediately deduce the formula by which the desired value
can be determined directly from the amount spent reagent, in this case, the concentration of phenol in water will soon be determined by
substituting the formula amounts of sodium thiosulfate.
The method is applicable to any of phenols, which contain a CH bond activated by reaction with a halogen. In the phenol, as we have seen - is
due at 2, 4 and 6 carbon atoms. Naturally, in the case of substituted phenols is necessary to know the amount of reactive CH bonds in order to
properly calculate the mass determined by phenol. If all of the positions occupied by other substituents, such as in 2,4,6-trichlorophenol, the
method is not applicable.
That the method gave accurate results, you must follow these rules: the amount of bromide in the solution should be at least 5 times the
amount of bromate (otherwise be allocated Cl 2 and Br 2 ), hydrochloric acid should not be very much, as sodium thiosulfate is unstable in
acidic medium.
Mathematics
In brackets, which classes addressed the problem. Your performance in mathematics is considered successful if properly resolved at least two
tasks addressed to you or more senior classes. Only one correct solution of the problem will also be awarded by the jury.
1. (6-7) Find the greatest four-digit number, all digits are different and that is divisible by 2, 5, 9 and 11.
2. (6-9) Ivan believes that the fraction "cut" zachrkivaya identical numbers in the numerator and the denominator. Serge noticed that
sometimes gets Vanya right of equality, such as 49/98 = 4/8.
Find all the proper fractions with numerator and denominator consisting of two non-zero digits that you can "cut".
3. (6-11) The king allocated for the maintenance of order Pisarsky 1000 rubles per year (all clerks get equal pay). The king was advised to
reduce the number of clerks at 50%, and the remaining clerks increase salaries by 50%. As far as changing the costs of the king on Pisarsky
order?
4. (8-11) noted four vertices of a square. Note four more points so that all middle perpendicular to the segments meet at the marked points
lying on two marked points.
5. (8-11) is located on the plane as much as possible points so that any three points do not lie on a straight line and are the vertices of an
isosceles triangle.
6. (10-11) It is known that the roots of the equation x 2 + px + q = 0 - integers, and p and q - primes. Find p and q .
7. (10-11) Is there a tetrahedron, all of whose faces - isosceles triangles, with no two of them are not equal?

Math games
Competition for mathematical games held for students not older than 8 class. To solve advise you to choose the game that you interesting and
understandable.
Answers and comments on the tasks of the contest on mathematical games

1. "The sum of the digits"


Given the multi-valued number, for example 999. The figures the amount of numbers (in this case 27), and the game begins. The first player
to call any number from 27 to 999 (27 to name possible, and 999 - no). This number is considered the sum of digits, then the second player
calls any number less than the title and no less the sum of its digits, and so on. Anyone who can not walk, loses. Who will win with the right
game? How does the game from the original number?
2. "Last candy"
in the box are candy, which
a) 57
b) 50
c) 1000
d) N > 1,
played by two, take turns. In one move, everyone can afford to take any number of sweets, observing two rules: the rules of courtesy - you
can not take sweets more than just picked up the enemy, and the rule of honesty - the first course you can not take immediately all the
candy. Victory is considered having taken the last piece of candy. Who will win with the right game?
3. "Paint stripes"
Dana strip 1 * n and sealed tubes of paint. Two players take turns paint one unpainted cage bars in any color, printing out, if they wish, with a
new tube of paint (tubes much). Requires that adjacent cells have different colors. When the whole strip will turn, is determined by the loser he will be the one who last unsealed tube. Who wins when
a) n = 15?
b) n = 20?
in) n = 10?
For what more original (relatively large \ 'them) long strips you can specify the method of a win-win game for some of the players?
4. "Paint wide strip"
The rules are the same as in the previous game, but now given a strip of 4 * n . Neighboring cells are considered a common side. Who will
win with the right game? Come up with (if you can) strategy game some of the players on a strip width (not 1 and 4).
5. "A piece of chocolate"
There are chocolate, divided into square slices grooves. Play two. First move to break a chocolate bar along the grooves into two parts. Next
opponent breaks one of the resulting parts, then one part of the first breaks again and so on. The one who broke off a piece of one slice
winner. Who will win with the right game? Consider the case of this size chocolates:
a) 3 * 7 cloves;
b) 5 * 12 slices;
c) 7 * 7 cloves;
For what else initial (relatively bolshi'h) the size of chocolates, you can specify the method of a win-win game for one player?
6. "Tic-tac-toe in a new way,"
On the field, 5 * 5 play two. Everyone puts their own power in any cell or cross toe. Those who managed to put three of any character in a
row (horizontally, vertically or diagonally), the winner. If anyone is not possible, and the free field over - draw. Which player is guaranteed to
win and how it should be for this play?

Physics
Numbers in parentheses after the specified task classes in which this problem is recommended. Students grade 7 and under sufficient to
solve one "their" problem students grade 8 and older - two of "their" problem.Solve other tasks is also possible.
1. (6-7) The builder picks up the load with the help of ropes and two identical blocks (see illustration). Which unit (1 or 2) at the same time
spinning faster? (Block - a wheel with a groove for the rope.)

2. (6-8) The walls of industrial chimneys are usually specially made so that they are poor conductors of heat. Why (still all the heat with the
smoke "will fly into the pipe")?

3. (6-11) On the table are two large plastic bottles with carbonated water: the most common one (bought in the store and more it does
nothing), and the other was kept in the refrigerator, and some of the water it turned into ice. When they opened the first bottle, nothing
unusual had happened, and when you open the second bottle of everyone who was there, were drenched powerful jet of soda water. Explain
why ice is so "bad" acts on the bottle.
4. (7-10) along the length of the street there are columns with lanterns. The first and last columns - switches. Every night watchman includes
lights, every morning off, with both can make any switch (for example, those to which the closer). Draw as switches should be arranged and
how they want to connect.
5. (9-10) Why on the street in cold foggy weather is easier to catch a cold than at the same temperature of air when the fog is not present?
6. (10-11) In many old appliances of various functions used this detail: a thin glass tube (inner diameter 4.1 mm) is almost completely filled
with mercury (a small section of the tube is filled with mercury instead of an aqueous solution of silver iodide AgI 2 ). Along the tube - scale
with graduations. During operation of the device through a tube passed a constant electric current. That this will occur in the duct? What do
you think, why this item is needed?
7. (8-11) The German physicist Wilhelm Feddersen in 1857 built a device for the study of electrical discharges (sparks) in the air. In one
experiment Feddersen example, it appeared that the illumination of the spark was 0.000025 seconds, and this spark consisted from several
consecutive bursts of approximately 0.000005 seconds. How Feddersenu able to measure such short periods of time (no exact hours in the
middle of the 19th century, of course, was not)? Suggest any way to determine the duration of the glow spark, using only techniques
available, in your opinion, the experimenters of the time.
8. (9-11) In a small mountain stream built gidreolektrostantsiyu. After that ecologists surveyed the river 1 km from the power plant
downstream and have not noticed any changes (flow velocity and water were the same).Where does the energy to generate electricity, and
where this energy had happened earlier, before the construction of the power plant?
9. (9-11) The body, which has no initial velocity begins to move in a straight path with variable acceleration A , which in each point of the
trajectory proportional to the distance x from a given point to the place of beginning of the movement (ie, A ( x ) = Kx ). Find dependence of
the velocity of the body v the distance traveled x .
10. (9-11) is fitted with a three-pin, consisting only of resistors (and wires). Conclusions are numbered 1, 2 and 3. The electrical resistance
measured between terminals 1 and 2 is equal to R 12 . Similarly, the resistance between terminals 1 and 3 is equal to R 13 , the resistance
between terminals 2 and 3 is equal to R 23 . Could it be that R 13 > R 12 + R 23 ?
11. (9-11) The girl wants to enjoy the magnified image of his face with a plane mirror and a magnifying glass (dvoyakovupukloy lens). How
and where to place the lens? Explain why the proposed alternative you do.
12. (10-11) The metal rod suspended horizontally by the two ends, one end - on a spring (or elastic), the other - on a string. The mass of the
rod is evenly distributed over its length. If the threads are cut, it would seem, the load on the gum should be increased and the end of the rod
to which it is bound, should immediately start to fall down. But in fact, he, on the contrary, "jumps". Specify a mistake in reasoning and
explain what is really going on.

Chemistry
Participants are encouraged to solve problems 2--3. After each task numbers in parentheses indicate which classes she recommended. To
solve problems not of their class are allowed, but the tasks for the younger class than yours, will be assessed fewer points.
1. (7-8) The following figure shows the four tubes A , B , B , G . In the tube was poured original 3 ml of water, and then one of them was
added 7 ml of acetone, and the second - 7 ml of hexane, the third - 7 ml of chloroform, and in the fourth - 7 ml of sunflower oil and two drops
of detergent. The tubes were mixed, and then they were placed in a rack and left for a while. Determine the contents of each of the

tubes. What you need to know?

2. (7-8) A mixture of hydrogen and chlorine in a closed vessel at a constant temperature of scattered light was irradiated. After some time, the
content of chlorine has decreased by 20% compared to baseline, and the volume fractions of components in the mixture at this time become as
follows: 60% chlorine, 10% hydrogen and 30% chloride. What was the quantitative composition of the original mixture of gases?
3. (8-10) The figure shows the nucleoside adenosine molecule. (In the "corners" or "forks", where not indicated chemical element symbol is
meant an atom or a group C CH or CH 2 , so that the total valency of the carbon is equal to four). Calculate the molecular weight of the
substance and one molecule in grams? What fraction of the total mass of the molecule is the mass of all it contains nitrogen atoms?

4. (8-10) gave chemist solution for analysis, in which it is found following ions: Na + (92 mg / l), K + (156 mg / l), Cl - (213 mg / l) and
NO 3 - . Calculate the concentration of nitrate ions (NO 3 - ) in the solution. From which salts can be prepared in a solution? Consider all
options.
5. (9-10) of the metal plate from an unknown mass of 7.8 g was placed in a container with dilute sulfuric acid. Another such plate from the
same metal into a vessel with concentrated sulfuric acid. Gas volume of 3.12 liters, select one of these vessels were collected, mixed with an
equal volume of a heavy yellow-green gas and irradiated with ultraviolet light. The resulting gas mixture is passed through a solution of silver
nitrate, thus began to fall white curdy precipitate. Identify the parent metal. Write equations of all reactions. Explain why in one case the gas
is released (in what?), And the other - no.
6. (9-11) The young researcher has 10 ul of reagent A with a concentration of 15 mmol / l. He needs to add a reagent in ten test tubes
containing 1 ml of reaction mixture so that each reagent tube A was at a concentration of 10 mmol / l. Help him to do this if the researcher has
a pipette, which you can dial any fluid volume from 10 to 200 l, 10 times (t. E. 10, 20, 30 l and t. D.). Note: L = microliter = 10 -6 L mmol =
millimoles = 10 -3 mol mol = micromolar = 10 -6 mol.
7. (9-11) Into a flask with nitric acid was placed a piece of copper metal, after which the flask was stoppered with a gas outlet tube. The result
was isolated brown gas that is collected in a glass jar. After cooling to 0 receptacle O C there faintly found almost colorless liquid. Such a
piece of copper is placed in another flask, which also contains nitric acid. Here again stood brown gas which likewise collected in a glass
jar. This time, after a strong cooling of the vessel there was a bright blue liquid. Explain the observed phenomena. What caused the difference
in the results of the experiments (the nitric acid in the first flask was different from nitric acid in the second flask)? Write the reactions.
8. (10-11) In the 19th century German chemist Liebig proposed to produce glass mirrors with silver plating. His method comprises the
following steps. First, an aqueous solution of silver nitrate, an aqueous solution of potassium hydroxide. The resulting precipitate was filtered
off and stirred with aqueous ammonia. Then, the resulting clear solution was immersed a glass sheet, one surface of which was thoroughly
degreased and formaldehyde added. As a result, cleaned glass surface was coated with a layer of metallic silver. Write the reactions that occur
in the process of manufacturing mirrors.
9. (10-11) What is the structure of the hydrocarbon composition can have a C 5 H 8 ? Draw possible isomers.

Competition in Physics. Assignments, answers and solutions.


Tournament participants about the proposed job provides the following information. In parentheses after the number of tasks listed classes to
which this task is recommended. Students grade 7 and under sufficient to solve one "their" problem students grade 8 and older - two of
"their" problem. Solve other tasks is also possible.

1. (6-8) A passenger wants to travel on the underground metro line, so that during his visit to take place in tunnels as little time. Where he
should sit down: at the beginning of the train, the middle or the tail?
(Station tunnels themselves are not considered, although they are under the ground. Different cars is carried out in a tunnel at different times
due to the fact that the train station gradually increases its speed and as smoothly brakes.
Solution. We denote points on the time axis appears on some stations beginning, middle and end of the train + H + and + K, and moments
later the county with the station - respectively -H, -C, and -R. Will look something like this:
..... + H + C + K ............... ........................ ...........- H ...............- With .....- K
The interval between + H + C and shorter interval between C and + + K because by the time the train at the station mid-he managed to slow
down, so the exit from the tunnel back half of the train will be spent more time than to leave the front half. Similarly, the distance between the
points selected -H, -C, and -R. While parked at the train station (from + K to H) is chosen arbitrarily and the answer is not affected.
If the train accelerates and slows down "symmetrical", the most profitable place in terms of our problem - the middle of the composition (the
distance between letters With more than honey letters H or letters K, which is exactly the middle of the train spends the most time at the
stations, and therefore least of all - in tunnels.). Slight differences in the velocity versus time during braking and acceleration can slightly shift
the most advantageous location.
Of course, in unusual situations that are in the problem are not assumed, the answer may be very different from that obtained in the
solution. For example, if a station wagon has left only the first, and then the train broke down, stopped and went on only an hour, the most
profitable place will be precisely in the first car.
Note. This problem arose from the practice. Namely, it is known that (some) stations (Moscow Metro) mobile phones operate, and in no
tunnels. Specified in the decision process can "zoom in" his time at each station (in the area of reliable reception signal strength) for a few
seconds. This time, just enough for a conversation on a mobile phone.

2. (6-9) In the distance stands a rustic lodge. Of pipe smoke coming and the wind is right. Considering the house through a magnifying glass
(standing in the same place), we will see an inverted image (trumpet will stick down). And in what direction will go "upside-down" smoke
(after the "real" smoke or vice versa)?
Answer. "Inverted" smoke goes in the opposite direction "present."
Comment. The term "inverted" in the context of "lens produces an inverted image" - not very good. He, of course, rather unfortunate is tricky
(but only after explaining the essence of the case, which in many textbooks available).
In fact, the lens, of course, nothing turns, and converts the central symmetry (which is equivalent to the plane of rotation of 180 o around the
center of symmetry), what is easy to see, mentally constructing the path of the rays in space.
But to say "Central symmetrized image" is less convenient than the "inverted image." Apparently, so often and say "inverted". In addition, the
construction of the plane, "coup" segment produces the same result as the central symmetry. Apparently, therefore, the term "inverted image"
is often used in this situation. But it must be understood precisely as a 180- O , and not only as a change in top-bottom orientation.

3. (6-9) A person who fails to step on a rake lying on the ground, can get hit handle these rakes. Offer (and explain) the method address this
shortcoming (the method should be such that the rake does not appear in other, more significant shortcomings).
Note. Of course, in times of technological progress can be made rake in the electronics, the handle of which, for example, will be equipped
with a video camera. Then a special computer program could recognize situations when the handle moves per person, and reject their own pen
aside.
But such "rake" certainly be too expensive (which is a major drawback that contradict the conditions of the problem). In addition, the
development of mechanical parts, control systems and software for such "rake" - a necessary part of the solution (if you choose this way of
solving), and the case is very difficult.

(Note that such "smart" tools independently monitor can do with these manipulations do exist and are used where mistakes are not allowed or
disadvantage compared to the cost of such tools - in research, medicine, space technology, nuclear industry, the assembly conveyors, etc.).
But rake to such situations do not apply. Therefore, we consider an easier way.
a) You can make the handle heavier - so that the weight of a person stepping on the teeth, was not enough to handle was set in motion. But
then rake will not be very easy to use. Moreover, if the rake still fail occur with sufficient force (e.g., a run), then ... Reasonable knob contrary,
as can be easily done (such materials are, for example, ski poles are manufactured for athletes). In any case, you can put on the end of the pen
tip to mitigate the potential impact.
b) The teeth can not bend at right angles, and their increased slightly or somewhat less than 90 O . In the first case the base of the teeth will
serve as a "stop" (see. The end of paragraph e). In the latter case the handle may jump during movement but abut teeth rake the land before
the handle reaches the vertical position and be able to hit a person. (Note, bent at a right angle is not the teeth do not rake less comfortable raking material may or "engraved" in the teeth, or "slip" out from under them.)
c) It is possible to make the teeth such that they can occur as it was difficult and inconvenient. For example, to make them long - about 50 cm
in length (row such unusual rake is quite possible). In addition, these teeth will be more noticeable. But if for them yet someone zapntsya,
then ...
g) It is possible to attach the comb to the arc (the reverse side of the tooth in a plane perpendicular to the handle). These rakes will be simply
impossible to put on the ground so that you can step on the teeth. But for the very arc can be tripped over, and ...
d) can be removed from the central comb teeth that are most dangerous to stepping. And when stepping on the side teeth pen likely fly
sideways and will not get to come. Also easy to see that for the occurrence of undesirable effects discussed with a rake on the side teeth must
be pressed with biggest weight.
e) It is obvious that the handle of the rake "jump" as a result of stepping on the teeth more easily than more torque creates a human foot on a
possible turning point. Therefore, the time necessary to reduce as much as possible (this radius of curvature of the teeth in a plane
perpendicular to the ground surface should be as small as possible, but at the same time the safety margin for yet must not be acute). Torque
can be generally negative, i.e. the pressing handle to the ground surface. This can be achieved by means of special stops protruding from the
base of the teeth perpendicular to these teeth (ie, the direction of the handle). In order not to interfere with these protrusions must be shorter
teeth is 3-4 times.
g) You can even make a "balanced" rake - with two rows of teeth sticking out in opposite directions. Then, if the rake lying on the ground,
some teeth will function as stoppers, and besides podkovyrivat ground, which also make it difficult bouncing of the handle.
h) the comb can be made straight, and the arc (the rake handle is attached to the center of the arc from the outside, the teeth are made of
different lengths so that their ends are located in the plane, most suitable for work).In this case, when stepping on the edge of the arc center
teeth will serve as stops, while stepping on one of the edges of the rake will rest in the opposite edge of the comb. Of course, the garbage out
of such rakes will "scatter" and grass or hay (which still cling to the teeth) can row without problems.
i) If the rakes are for hay or grass, the teeth can be made of an elastic rubber. Elasticity should be such that they are not bent on hay, but bent
under the weight of a person (note also that in the hay load is distributed across all the teeth, and leg can occur only in two or three
neighboring).
k) combining the previous methods, can make the rake tines which central - rubber and the side - metal.
Some participants suggested to make on the handle rake articulation with emphasis, the handle allows you to go in one direction only. In their
eyes when stepping on the teeth of the handle should bend. Unfortunately, such a rake will be unfit for work because during the handle will
bend in the same direction.

4. (7-10) Up to a rapidly rotating grinding wheel touched a metal object. Fly away from the contact "spark" that while flying through the air in
the form of points of light, and then "explode" (fly apart), even though the external conditions in which there is a spark, do not change. A
physical explanation for the "explosion" of sparks.
Note. Of course, the right to be considered any description that is suitable for the problem statement does not contradict the generally
accepted physical concepts.
In the photograph the observed phenomenon is as follows: (As you can see, not all "traces" end "explosion." Most likely, this means that they
occurred after the end of photography. For this reason, some tracks do not start at the grindstone, but just in the air - their initial section and

just was not filmed).

The most likely story each spark - like that.


a) a small piece of metal detached from the metal object. Shock and separation - is inelastic deformation, resulting in the heat is. But on a
metal object is heated relatively small (compared to the size of the object) area;thermal conductivity of the metal is good, so the heat from the
heated portion is relatively rapidly distributed over the volume of the metal.
b) Another thing - a little piece. There's nowhere to be distributed heat. Therefore, the surface is still hot. If its temperature exceeds ignition a piece of lights (usually burning begins as early as the time of separation, but the main part of the burning metal because of the heat
immediately terminated and a piece of lights on).
c) If the size of the pieces are comparable with the characteristic dimensions of the field of combustion in the combustion process involves the
whole surface of the metal. From that moment is inside the metal piece to the surface of the combustion temperature of the metal. From this it
will only be heated (and can not be cooled by atmospheric air, ie. K. Has no direct contact with the atmosphere).
g) Then, first metal heated to a melting temperature, and then melted (melt on burning also requires energy) and turned into a liquid to be
heated further.
d) At the time when the temperature of the molten metal will be such that the vapor pressure of the metal at a temperature equal to the
atmospheric pressure, a drop "boils", i.e. flies apart, and that looks like an explosion.(More precisely, boiling is not the metal itself, and at
least a solution of this metal (or metal alloy when it was) combustion products and possibly other substances, such as metal compounds and
nitrogen formed at high temperature).
Clearly, the above effect will be observed not with all metals (and alloys), and only having the best set of parameters (temperature of ignition
and combustion, the specific heat in the solid and liquid state, melting point, vapor pressure corresponding to the atmospheric pressure, etc. ).
Another possible "scenario" of events - cracking sparks due to heating, which almost certainly will be uneven. May crack as the metal itself,
and formed on the surface of brittle oxide crust.
Some participants rightly noted that the situation described in the problem may also occur when falling to Earth metallic meteorites.

5. (8-11) In one industrial enterprise solve the problem - on the territory there is a road that makes a sharp turn. View of the road around the
bend is enclosed equipment. At this point, there were often small accident - drivers too late to notice the bend oncoming car. Decided to
produce and install cornering a large mirror that the driver had seen what was happening around the corner. What mirror is best suited for this
purpose - flat, convex or concave?
Answer. categorically answer the question can be given. In most such cases, probably best to use a convex mirror.
Comment. The flat mirror (compared to any "crooked") provides less opportunity review (limited condition "angle of incidence equals the
angle of reflection"). Distorting mirror is as if from the "flat", located at different angles and allow you to view all sorts of directions. In the
concave mirror in comparison with convex difficult and uncomfortable "look"; moreover, it can focus light into the eyes of the driver of the
Sun, washers and the like, which is undesirable. Distortion of the image of the car in the "distorting" the mirror in this case is not very
important - important to get information about the car around the corner at all available. Distortion in such cases is even desirable - if the
image looks unnatural, the driver will be harder to accidentally take a mirror of the real picture of the terrain.

6. (9-10) As a result of the combustion of hydrogen in air is obtained water. In fact, this process is much more complicated. For example, by
hydrogen peroxide is also formed (chemical formula H 2 O 2 . A solution of this substance in water is relatively stable at room temperature
(used for example in medical applications). However, in the flame of the high temperature, almost all the hydrogen peroxide formed here is
decomposed. proposed a method of burning hydrogen, can significantly increase the amount of produced hydrogen peroxide.
Answer. One of the simplest and most effective ways to ignite --- "Torch" hydrogen and "lick" them a piece of ice. Surface temperature
tayaschego ice - always 0 O C, more precisely, does not differ much from this value.Ice during melting very effectively absorbs heat. As a
result, one can create a low temperature region directly into a combustion zone where the hydrogen peroxide is formed, as required.
When ice melts, produces only water, that is obtained peroxide will no longer be contaminated by anything.
Note that to come up with a more efficient method of cooling is quite difficult - accommodation in the combustion zone of any chilled to a
temperature of less than 0 O C objects will lead to the fact that they still namrznet ice.

7. (9-10) Dunno walked along the beach is not very fast and not very deep river and noticed that growing algae at the bottom of a grassy slope
in the direction of flow and have because of this almost the same shape as the big trees in a hurricane . With trees all clear - thought Dunno trunk resilient, so the wind can not squeeze a tree to the ground. And why not pressing for algae to the bottom (they herbaceous stems - not
elastic)? Help Dunno understand.
Comment. Trees keeps upright trunk. If you think, why the algae do not lie on the bottom, it remains the only sensible option - because of the
Archimedes force (ie, the average density of plant tissues of algae, which saw Dunno, less than the density of water). Archimedes force acting
only on top, small, so easy for this top of tips. A tensile force base of the stem should compensate for the difference Archimedes forces and
gravity acting on all located in the water part of the plant. This difference forces directed vertically, so if the stream will be able to tilt too
much stem, its tensile force is too large (such that the vertical projection had the desired value), and the plant will come off. Because of these
plants on the bottom and there, and there are only those that saw Dunno (We assume that the flow is not very quick and lift (or, conversely,
pressing) the power flow is insignificant. It is also possible to note that the flow velocity near the bottom usually less than at the surface although this is not directly related to the observed form of algae).

8. (9-11) There is a wire mesh with square cells (for example, a fence, a wire is the same everywhere). How to use batteries, ammeter and
connecting wires to determine the resistance of the wire segments of equal length of the square. The fence should not be broken, but you can
connect the wires to an arbitrary location. Battery voltage U . The resistance of the battery (internal) and the wires are not taken into account.
Solution (one). For the convenience of all parties will be called squares resistors. To introduce the notation desired resistance R . Select one of
the resistors. It is directly connected with the other six (three on each of the connection pins). Opposite conclusions of 6 resistors are
connected by wires. Get a simple circuit isolated from 7 resistors. Connected to the terminal originally selected resistor series connected
battery and ammeter. It is obvious that any resulting current can flow only on these seven resistors connected and contact wire (and can not
"leak" off-site scheme, limited this wire). Similarly, no external currents also can not get inside this area (the wire will close them).
Thus, the resistance of the isolated circuit resistor 7 is determined experimentally as a U / I , where I - the ammeter. On the other hand, it is
equal to
( R * (2 * ( R / 3))) / ( R + (2 * ( R / 3))) = ((2/3) R 2 ) / ((5/3) R ) 2 = R / 5
2R/=5U/I
R = (5/2) U / I

9. (9-11) In a vertical dam made two holes located on the same vertical line at a distance h 1 and h 2 from the surface of the water before the
dam, respectively ( H 1 < H 2 ). Assume that the stream of water flows through the holes horizontally at a speed of (2 GH ) 1/2 . Find at what
distance from the surface of the water jet intersect with each other.
Decision. Consider two particles of water: taking off from an opening located at a distance h 1 from the surface of the water and off from the
opening located at a distance h 2 from the surface of the water.
Let these particles are met (at the intersection of the jets), while they are in free flight before the meeting is denoted by t 1 and t 2 ,
respectively.
Horizontal components of the particle velocity with time does not change, and the vertical - uniformly increasing (acceleration G ).
The particles meet, they should be at the same time and at the same distance from the dam, and the same distance from the surface of the
water. Composition and solve the corresponding system of equations.

t 1 (2 G H 1 ) 1/2 = T 2 (2 G H 2 ) 1/2 ;
t 1 h 1 1/2 = T 2 h 2 1/2 ;
t 12h 1 = T 22h 2 ;

H 1 + ( G T 1 2 / 2) = H 2 + ( G T 2 2 / 2)

H 1 + (( G T 1 2 ) / 2) = H 2 + ( G T 2 2 ) / 2)
H 2 - H 1 = ( G T 1 2 / 2) - ( G T 2 2 / 2)

t 2 2 = T 1 2 ( H 1 / H 2 );

H 2 - H 1 = ( G / 2) ( T 1 2 - T 2 2 )

t 2 2 = T 1 2 ( H 1 / H 2 );

H 2 - H 1 = ( G / 2) ( T 1 2 - T 1 2 ( H 1 / H 2 ))

h 2 - H 1 = ( G / 2) (1- ( H 1 / H 2 )) T 1 2
h 2 - H 1 = ( G / 2) * (( H 2 - H 1 ) / H 2 ) T 1 2
t 1 2 = ( H 2 - H 1 ) (2 / G ) * ( H 2 / ( H 2 - H 1 )) = 2, h 2 / G
We found the time spent in the flight of the first particle. At the time of departure from the hole is placed at a distance h 1 from the water
surface, and after a time t 1 after this would be below the water surface at a distance of H = H 1 + (1/2) G T 1 2 = H 1 + (1/2 ) G (2 h 2 / G )
=H1+H2
That is, it is at a distance from the water surface and the point of intersection of the jets situated.
Note. The formula of the velocity of the fluid from the hole v = (2 GH ) 1/2 corresponds to an idealized situation. Actual numerical coefficient
that depends on how the fluid flows are arranged in the vessel near the opening that is largely determined by the geometry of the openings, in
particular openings for these same factors are likely to be similar. Please note that in response to our problem numerical coefficients, as well
as the value of the gravitational acceleration g is not included, that is, the value of H does not depend on them.

10. (9-11) One way to obtain fresh water - salt water bursting through a special membrane permeable molecules of water, but retained in the
water salt ions (for this purpose pressure salt water should be more fresh pressure by a certain amount, denoted by P ).
Construct with the help of a membrane "perpetual motion". It is known that the density of freshwater ( n ) is less than the density of salt
( c ).
Take the pipe length H , placing it vertically, the lower end close membrane, immerse the tube in salt water (eg, ocean) and the inside of the
pipe nalem fresh water (to the level of the ocean surface).
If n GH < with GH - P , then pipe the fresh water level is maintained above the level of salt, and this difference in levels can be used to
generate energy.
Explain why the proposed device is actually not a perpetual motion machine, and where does the energy for such a device. (Note: According
to estimates available for the world's oceans necessary immersion depth h is approximately 10 km.)
Solution. The fact that a uniform aqueous solution of the salt in a gravity field is not stable - spontaneously salt will "ramp" down (i.e., after
the establishment of equilibrium salt concentration will increase with depth).Described in the problem device uses the potential energy of the
salt dissolved in the ocean. To maintain the operation of the device must be continuous mixing of fresh water discharged to the surrounding
salt, and it must constantly expend energy.
In terrestrial seas and oceans mixing is realized in particular, due to currents (which consumes energy from solar radiation), lunar and solar
tides (at the expense of the kinetic energy of the Earth, Moon and Sun), evaporation (in tropical latitudes, the salt concentration at the surface
may be higher than depth t. k. the water is intensively evaporates from the surface, and the salt remains in the surface layers) and other
factors.
The provided solution in any case is not a complete description of the situation: we did not consider the dependence of the required pressure
difference P of the salt concentration, the compressibility of salt and fresh water, thermal effects on the membrane, and more.

In brackets, which classes recommended task; solve the problems of the older classes are also permitted.
1. (6-7) Mother gave money Vase 30 pencils. It turned out that in the pencil factory store an advertising campaign in return for proof of
purchase of a set of 20 pencils return 25% of the value set, and in exchange for proof of purchase of a set of 5 pencils 10%. What is the
greatest number of pencils can buy Bob?

2. (6-8) Pete mother also gave money for pencils. Terms of promotion are the same (see. Problem 1). Peter tried to buy as many pencils and as
a result he was able to buy 12 pencils more than my mother asked. How many pencils mother gave the money?
3. (6-9) Fill in the box 9 * 9 a few cells so that the center of the square were not visible on his part (ie any ray emanating from the center,
touched some shaded cell at least on the corner). You can not paint the cells adjacent to the side or corner, and the central cell.

4. (6-9) In the tops Nonagon arranges the numbers 1, 2, 3, 4, 5, 6, 7, 8, 9, and then at each diagonal product write numbers on its ends. Is it
possible to place the numbers at the vertices that all the numbers on the diagonals have been different?
5. (8-11) Among the numbers A , b , C , there are two identical. And the remaining number - more. Make this an arithmetic expression of the
letters A , b , C , signs +, -, *,:, and braces to happen as a result of computing that number. (Parentheses, signs and letters can be used any
number of times.)
6. (9-11) The board drawn pentagon inscribed in a circle. Masha measure its angles and said that they are equal to 80 O , 90 O , 100 O , 130 O ,
140 O (in that order). Whether the rights Masha? Answer justify.
7. (10-11) The sum of the three angles is equal to positive 90 O . Can the sum of the cosines of the two of them to be equal to the cosine of the
third?

Math games
Dear participants! Here are the rules of four games and tasks on them. We strongly recommend that you take a few games at once, and read
the terms and choose the one that seems more attractive. Play different versions of the game, feel it and try to perform the task. More or less
serious analysis of even just one game certainly will allow you to consider one of the winners. If you have chosen to solve the game you find
any laws, principles, were able to make out is not specified in the task fairly common cases, write about it too.
1. "Unequal handful"
The rules. There are a bunch of multiple stones. Over the course can be divided into one of the existing piles into two non-empty. This
requires that at no point did not have two identical piles. Anyone who can not go losing.
Job.
1) In a game of 16 stones start to make progress 5 + 11. Show how now the second player can win.
2) In a game of 16 stones start to make progress 5 + 11. Give an example of (unsuccessful) move the second player, after which he lost.
3) Who will win in the game of 11 stones? Justify your answer.
4) Who will win the game in 22 stone? Justify your answer.
2. "Checkers in a new way"
Rules. The black and white checkers is played on a checkered board. Originally them equally, and they are at the top and the bottom row of
the board. Checker for the course can go into any adjacent empty cell on the side of the sword or beat an opponent standing in any of the four
corner adjacent to the cells (which hit checker is put in place battered, beaten --- removed from the board). Not necessarily to beat, but the
goal of the game to beat --- all opponent's checkers.
Job.

Position 1

Position 2

Position 3

Position 4

Position 5

Position 6

1) In position 1 white and begin to have an advantage in one piece. Find a win for White. In the same position, specify possible (bad) turn
white, which will lead them to defeat.
2) Describe the possible party from the initial position 2. Which player will achieve victory?
3) at positions 3 and 4 begin to white. Prove that they are losing position 3, but win in 4. What do you think when dvuhshashechnyh positions
will win the white, black and when?
4) In position 5 start to black. Show that in any of their course they can not avoid defeat.
5) Find a win for White from position 6.
3. "The Road"
Rules. The country has two major cities, A and B. Many cities are connected country roads (map attached). Two construction firms in turn
asphalt on the same road. That company, which for the first time after the move there is the opportunity to drive on asphalt from A to B, is
declared the winner and the other loses.

Map 1

Map 2

Map 3

Map 4

Map 5

Assignments
1) What kind of company, one that begins or her opponent to win on map N 1?
2) The same question for the map of N 2.
3) The same question for the map N 3 (where n "jumpers")?
4) Who will win the card N 4
5) Who will win the card N 5?
6) Who will win the card that generalizes the latter two cases, and looking like a rectangle m * n cells?
4. "Beads of Staples"
The rules. There are a chain of red and white paper clips, the total number of which is odd. A player for one course can detach yourself from
any edge of a chain of several clips, but so that she was "beautiful", that is symmetrical, does not change when turning. Chain from one clip is
considered beautiful. If the whole chain of beautiful, can one move to take her entirely. Who at the end of the game will be more staples, the
winner.
Job.
1) Prove that no matter what the chain of more than two clips, the player can unhook it from at least two clips per turn.
2) Which player will win a game with chain KBKBBBKKBBBKBKB? (The letters K and B are marked red and white paper clips.)
3) Which player will win a game with chain KKBKKBBKBKKKBKBBKKBKB?

Physics
Numbers in parentheses after the specified task classes in which this problem is recommended. Students grade 7 and under sufficient to
solve one "their" problem students grade 8 and older - two of "their" problem.Solve other tasks is also possible.
1. (6-8) The highway goes convoy of buses. Column length of 1 km, the speed of 60 km / h. On the way there was a road sign "30 km /
h." Each bus, driving past the sign, slow down and goes on with permitted speed of 30 km / h. Find the column length after all buses traveling
therein will pass by this road sign.
2. (7-9) The young builder suspended load (see. figure). What a mistake he made? All components (except the shipping) are considered to be
weightless, there is no friction.

3. (7-9) The aircraft IL-96-300 brand has 4 engine. Traction force of each motor is sufficient for lifting cargo weighing 16,000 kg (ie only 4 *
16000 = 64,000 kg). However, this allowed for aircraft takeoff weight is 15 times more: 240000 kg. How did the plane takes off?
4. (8-11) In the spring, during the melting of ice on ponds sometimes it can be seen that the ice laced with long round vertical channels
(diameter of about 1 mm) filled with air. A similar phenomenon occurs when ice fills the interior of the drainpipe. But there channels arranged
horizontally from the center of the pipe to the walls (if she drainpipe while positioned vertically). Suggest an explanation of the mechanism of
formation of such channels.
Note. Photo of ice in the drainpipe published here: truba.jpg , 650 KB (The Challenge issued to pupils in the tournament, this picture was
not).
5. (8-11) The athlete rides a two-wheeled bicycle on the bike path round (round around the stadium). Smooth surface of the track, the wheels
do not slip on it. The radii of the front and rear wheels of the bike are the same.Which wheel rotates faster at the same time?
6. (9-11) Three identical positive point electric charge located at the vertices of an equilateral triangle. Draw (draw) a picture of the lines of
force of the electrostatic field generated by a system of charges in a plane passing through the charges. Try to note the characteristics of the
painting field lines (as in the figure, and in the text of the decision).
7. (8-11) of a conventional optical system (microscope, telescope, periscope, overhead projector, film cameras, telescopes, and so on. n.) is
usually possible to obtain a direct or an image, or inverted (ie. e. Rotated 180 O). Is it possible from the lens and flat reflective surfaces
(mirrors) to collect optical system allows us to observe the image rotated to some other (not a multiple of 180 O ) angle?
8. (9-11) In an aquarium with fish for ventilation from the surface of the water is fed a stream of air. Air bubbles while sometimes not begin
immediately float and sink before that to a certain depth. Moreover, the kinetic energy of the bubble ( MV 2 / 2, where v - the initial velocity of
the bubble, m - mass of air in the bubble) is not enough to do work against the force of Archimedes in the immersion of the bubble on the

apparent depth. But bubbles are still immersed, and this is clearly observed experimentally. In what could be wrong here?

9. (10-11) Dunno, Cog and Shpuntik working on the draft of the new spacecraft, faced with the following technical problem.
To exit into orbit spacecraft need to overclock to at least the first cosmic velocity - about 7.9 km / s.
At the same time, the average velocity of the thermal motion of the molecules of air at ambient temperature is only 0.5 km / s.
Designers have come to the conclusion that after the start and acceleration of the spacecraft to the required speed of 7.9 km / s inside the ship
air at the start of a fixed (in the reference frame associated with the Earth), with respect to the ship's hull will acquire the same speed of 7.9
km / s). After a series of elastic collisions with each other and the walls of the ship's direction of motion of air molecules become random, and
the speed will remain the same.
Temperature is proportional to the kinetic energy of the thermal motion of the gas molecules, ie. E. The square of the speed. Thus, as a result
of the launch of the spacecraft temperature increase itself in roughly ((7.9 km / s) / (0.5 miles / s)) 2 = 15.8 2 = 249.64 times that may lead to
problems in the operation of the equipment and cause discomfort to the crew.
Advise cogs and Shpuntik Dunno how to solve this problem.

Chemistry
Participants are encouraged to solve problems 2-3. After each task numbers in parentheses indicate which classes she recommended. To solve
problems not of their class are allowed, but the tasks for the younger class than yours, will be assessed fewer points.
1. (8-9) To prepare the copper sulfate solution with a mass fraction of 10% of the young chemist Peter weighed 10 g of copper sulphate
CuSO 4 * 5H 2 O and dissolve it in 90 ml of water. Then Peter measured density of the resulting solution. Peering into the directory, he was
surprised to find that the density of a 10% solution should be higher.
1. What kind of mistake that Peter?
2. What is the mass fraction of copper sulfate in the resulting solution?
3. How much water he had to take 10 grams of copper sulfate to obtain a solution with a mass fraction of 10%?
2. (8-9) The left side of the table contains the formulas and names of organic compounds. Record missed by analogy compounds of formula
or the names that appear in the right part of the table.

CH
C

CH
/
-N
\
CH

13

-SH (1)

Cl diethyldichlorosilane

\ /
Si
/ \
C 2 H 5
CH 3
C
\ /
C

Trimethylamine

(A) methylethyl
ether
Cl
4

methylbutyl

(B) dimethyl

||
O
C

CCl
C

-NH

Cl CCl 3
(2)
\ /
Si
/ \
Cl Cl
(B) ethyldichlorosilane
C 4 H 9
\
NH (3)
/
C 4 H 9

Ethylamine

-SH Trihlormetilmerkaptan
13

-O-CH

Geksilmetilovy
Ether

3. (8-10) There are two equally sized cubes made of gold and aluminum. In some of them contained a greater number of atoms and how many
times? (Density of gold is 19.3 g / cm 3 and the density of aluminum - 2.7 g / cm 3 ).
4. (8-10) Determine the mass fraction of perchloric acid HClO 4 aqueous solution if it is known that the number of hydrogen atoms in the
solution is equal to the number of oxygen atoms.
5. (9-10) As recognized solutions of aluminum sulfate, ammonium chloride, hydrochloric acid, sodium hydroxide and barium hydroxide,
being in the flasks without labels? At your disposal there is phenolphthalein. Suggest analysis plan. Write the reactions.
6. (9-11) forms a large amount of sulfur acids. For example, H 2 SO 4 , H 2 SO 3 , H 2 S 2 O 7 , H 2 S 2 O 8 , H 2 S 2 O 3 , H 2 S 2 O 4 , H 2 S 2 O 5 ,
H 2 SO 5 .
1. The structural formulas for these acids. Specify the degree of sulfur oxidation in each of them.
2. Which of the acids may have a pronounced oxidative or reductive properties?
7. (9-11) Determine the mixture fluoride and zinc chloride (in weight%) if it contains 55.9% zinc.
8. (10-11) The following formula repeating unit of some polymers.
-CH

-CH

-CH

(A)
2 -CH|
C 6 H
(D)

-CH

-CH|
CH 3

(B)
-N- (CH
| ||
HO

(E)

-C-

-CH

-CH = CH-CH

-CH

(B)
-CH|
Cl

-N-CH 2 -C| ||
HO
(D)

(F)

9. 10-11 Proton magnetic resonance (PMR) allow to distinguish between hydrogen atoms occupying nonequivalent positions in the
molecule. For example, NMR spectra of n -pentane comprises three signals methyl CH 3group signal CH 2 -groups adjacent to the methyl
groups and the signal of the central CH 2 -group. What is the structure may be a hydrocarbon with a mass fraction of carbon 84.375%, in the
PMR spectrum of which there are two signal?
Mathematics
In brackets, which classes recommended task; solve the problems of the older classes are also permitted.
1. (6-9) There are three triangles: acute-angled, right-angled and obtuse. Sasha took a one triangle, and Boris - the other two. It turned out that
Boris can apply (without overlapping) one of their triangles to another and get a triangle equal to Sasha's. Which of these triangles took
Sasha?
2. (6-9) at the station "Lukomorie" sell cards at the 1, 5 and 20 trips. All cards cost a number of gold coins. Five cards per trip more than one
5 trips, and 4 cards for 5 trips more expensive one card for 20 trips. It turned out that the cheapest way to travel for 33 heroes - is to buy a card
for 35 trips, spending a 33 gold coins. How much is a card on 5 trips?
3. (7-11) The board was written several natural numbers, with the difference of any two adjacent numbers is equal to the same number. Kohl
replaced in this record different figures by different letters, and numerals - the same letters. Restore the original number, if written on the

board:
T, EA, EC, LA, SS
4. (9-11) Solve the problem N 3 for the label:
A, BC, DEF, CGH, CBE, EKG
5. (10-11) Little Petya sawed all legs of square stools and four sawed-off pieces lost. It was found that the length of all the pieces are different
and that the stool then standing on the floor, let the pan, but still touching the floor with all four ends of the legs. Grandpa decided to repair
the stool, but found only three pieces with lengths of 8, 9 and 10 cm. How long can be the fourth piece?
6. (10-11) In the Loop Railroad n stations. Sometimes the assistant station communicate with each other by radio. At each time a session are
only two people. Per day between each two stations occurred exactly one radioseans. For each station (if we consider only its sessions), it
turned out that she communicated with other stations on the line in order of their position on the railway (clockwise or counter-clockwise at
different stations, these areas may be different), since one of the neighboring and ending other. What can be n ? (Analysis of the cases n = 4
and n = 5 is recorded as a partial solution to the problem).

Math games
We recommend to choose the most interesting game and answer the questions. The question "Who will win?" must not only meet, but to
explain in detail how it should play the winner to defeat any opponent, as if he went. If you find some more patterns in the proposed games,
will understand quite common unspecified cases, write about them, too. Do not try to solve everything. Good analysis of one of the game will
allow you to consider one of the winners.
1. From the corner of the room. There is a rectangle m * n . Players path leads from the corner to the opposite corner: one draws a line to a
neighboring node on a diagonal from the received second points - on the adjacent side, and so on. You can not cross your path at any
point. Who first come to the opposite corner, he won. Who will win with the right game?
2. Bench. It is known that people avoid strangers to sit side by side on the bench, if you can not do. There is a bench on which is placed N and
a lot of people are not familiar with each other people. Two players take turns put on the bench for one person, with a number of people can
not be planted. The player who can not make a move.
a) What is the minimum and a maximum number of people can be planted at a given length N bench?
Who wins if:
b) N = 6;
in) N = 7;
g) N = 8;
d) an arbitrary odd N ?
e) Who wins when N = 12?
g) Consider the same problem on the ring bench. Prove that the question of the benefit is reduced to the problem of the usual bench.
h) Consider another similar problem. On the bench accommodating N man seated players take turns on two people who agree to sit with other
couples, but certainly want to sit next to each other. Show that in this problem the question of benefit is reduced to the problem of the usual
bench.
3. Web. Players take turns to draw direct the plane. These lines divide the plane into pieces. One after one's progress on the field is formed in
the shape of the pentagon, the winner.
a) Who wins if you can not draw a direct parallel to the existing and can not be held directly through the intersection already drafted?
b) Who wins if you can not spend more than three pairs of parallel lines and more than three straight lines passing through the same point.
c) Who wins if you can not draw a direct parallel to the already existing.
d) Prove that if the game without limitation the second player the second course held straight line parallel to the first, then the first player has
a draw strategy.
d) Who wins the game without restrictions?

Tasks for oral competition. Along with writing contest on mathematical games can be carried out and an oral (maybe not in all places of the
tournament and for all classes). Oral competition can take place both in all three tasks and sessions - one task per session. In this case the task
is conveniently cut into separate tasks.
Participants oral competition describes what mathematical game, and it is called a mathematical solution. If you choose one of the announced
games - raise your hand and tell the decision (or act according to the rules, which declared the leader of competition).
Bench. It is known that people avoid strangers to sit side by side on the bench, if you can not sit down beside him. There is a bench on which
is placed $ N $ and a lot of people are not familiar with each other people. Two players take turns put on the bench for one person, with a
number of people can not be planted. The player who can not make a move.
1. Consider, first, the small value of N ( N announces master).
2. The game is the same, but the bench ring.
3. Seated on a bench in love couple who only have to sit next to them and the rest does not matter.
Wide Web. Players take turns to draw direct the plane. These lines divide the plane into pieces. One after one's progress on the field is formed
in the shape of the pentagon, the winner.
1. Prohibited conduct parallel lines and more than two lines through one point.
2. Prohibited only parallel lines.
3. Allowed to hold no more than three parallel lines in one direction and no more than three lines through one point.
From corner to corner. There is a rectangular m * n . Players path leads from the corner to the opposite corner: one draws a line to a
neighboring node on a diagonal from the received second points - on the adjacent side, and so on. You can not cross your path at any
point. Who first come to the opposite corner, he won.
The values of m and n announces presenter. In this game, there are situations where a player have nowhere to go - these provisions shall be
deemed a draw.

Physics
Numbers in parentheses after the specified task classes in which this problem is recommended. Students grade 7 and under sufficient to
solve one "their" problem students grade 8 and older - two of "their" problem.Solve other tasks is also possible.
1. (6-8) A man is standing on the deck of the ship and sees on the surface of the water a bright spot - a reflection of the sun. How to change
the distance between the spot and the ship, if a person will approach closer to the board? Horizontal deck of a ship.
2. (7-9) How does the device of a bicycle wheel, allow for air flow inside the wheel while pumping his pump, but not the exhaust air
back? Can you describe one of the well-known designs or come up with your own. Be sure to explain how it works.
3. (7-9) The snails usually move follows. Animal clings to something special mucosal surface. Moving is due to the agreed reduction of
individual sections of the surface. If a snail crawling on a transparent object (eg, glass), the reductions observed in the form of "waves"
(darker areas of the mucosal surface), "running" along the direction of movement of the animal. And in which direction the "run" these
"waves" - in the same direction, where the snail crawls, or the opposite?
(If you're interested in biology, and you know the correct answer, it is not enough just to write - you still need an explanation).
4. (8-11) times to supply electrical devices connected in parallel using several identical electrochemical cells (batteries). Explain why it may
be necessary (at first glance it seems that there is no sense in this, as several parallel connected identical batteries provide the same voltage as
one battery).
Note. The reasons for the possible use of multiple batteries connected in parallel - and these reasons may be different in different devices.
5. (8-11) swimmer may swim to the maximum speed v = 2 m / s. He needs to swim across the river width H = 200 m. The flow rate in the
river depends on the distance from the shore so that the flow velocity near the coast is zero. In the middle of the river it is maximal and equal
to u = 1 m / s. A graph of the speed of the river on the distance from one of the banks is a half circle. Swimmer in the minimum time crossed

from one bank to another. How far "blown" its course along the coast?

6. (8-11) The filament bulb is usually rolled into a spiral and then get a spiral - once again into a spiral. After the first collapse "inside"
remains about half of the surface of the filament, and after the second "folding" - half the remaining surface. That is, as a result of more than
75% of the thread enters inside. Not for nothing is lost if the light emitted by this "inner" surface?
7. (9-11) Two identical metal (conductive) ball are spaced apart from each other (do not touch each other). Balls are placed on the electric
charges Q 1 and Q 2 (0 < Q 1 < Q 2 ). It was found that the strength of the electrostatic interaction between the balls is equal to zero. How can
that be (why beads having charges of the same sign, not repel each other)? What is the charge q must be added to the charge Q 1 , the force of
electrostatic interaction between the balls was again equal to zero?
8. (9-11) In a container with rigid non-conducting walls is warm helium gas at a temperature of 200 K. The vessel is moving at a speed of 1
km / s. Suddenly vessel collides with a rigid solid wall, and almost instantly stops without changing its shape. What will be the gas
temperature in the vessel after equilibration? The molar mass of helium M = 4 g / mol, the molar heat capacity of an ideal monatomic gas at
constant volume C V = (3/2) R , the universal gas constant R = 8.31 J / (mol * K).
9. (10-11) will be considered through a magnifying glass (magnifying glass) installed perpendicular to the direction of observation, located in
the distance landscape. At the same time remember which objects (or rather, their image) "hit the magnifying glass" (ie, the observer sees the
image of a magnifying glass shaped objects). Now rotate the magnifying glass so that its optical axis made an angle of 45 o with the direction
of observation, and still is horizontal. Observer and magnifier (her optical center) at the same time remain in their places. More or less items
(ie, their increased or smaller fragment of the landscape) observer sees in the loop compared to the original situation?

Chemistry
Participants are encouraged to solve problems 2-3. After each task numbers in parentheses indicate which classes she recommended. To solve
problems not of their class are allowed, but the tasks for the younger class than yours, will be assessed fewer points.
1. (7-8) Young Naturalist Bob decided to call the quartz sand for terrariums. He took the jar capacity of 1 liter, went to bat and filled the jar
with sand. Bob knew that the density of quartz is 2.4 g / cm 3 . Therefore, it is easy to calculate what a mass of quartz sand enters the
bank. However, when Bob brought sand home and weighed it, the mass was 1 kg 100 g less than he expected. Bob has not determined the
cause of this discrepancy, because I do not know that for solids, instead of the density, you should use a different value - bulk density.
1) State that (from your perspective) should be called a bulk density?
2) Why is the density and bulk density substances are not the same?
3) Calculate the bulk density of quartz sand found Vasya.
2. (8-9), X forms several nonmetal oxides and one compound with hydrogen, where the element X -1 is oxidation. Higher oxide contains
61.20% oxygen, and hydrogen bonding - 2.74% hydrogen. Identify the element X, write the formula of its higher oxide and reaction equation
of this oxide with water.
3. (8-10) samples of lithium, sodium and potassium equal mass completely dissolved in hydrochloric acid (in three different vessels).
1) Determine the ratio of the volume (or mass) of hydrogen gas, which is highlighted in these three reactions.
2) Write the equation reactions.
3) After completion of the reaction found in one of the resulting solutions - acidic environment, in the second - neutral, and the third alkaline. How could this happen? Which of these metals is contained in each of these solutions (given that stock solutions of hydrochloric
acid had the same mass and contained the same amount of HCl? A explain.
4. (9-10) In the three banks without labels are three solutions of individual compounds - one, two yellow and colorless (or
solutions 1 , 2 and 3 ). Cans are taken from separate portions of the solution for chemical reactions.When adding a hydrochloric acid
solution 1 becomes orange 2 remains no visible changes in the solution and 3 Gas evolution was observed. Under the action of the silver

nitrate solution 1 red precipitate falls from solution 2- white curdy precipitate insoluble in nitric acid, and 3 gives a white precipitate which is
soluble in nitric acid. By mixing the solutions 1 and 3 with each other (with the addition of a few drops of sulfuric acid) color mixture
becomes purple. What may be substances in solutions 1 , 2 and 3 ? Write the reactions.
5. (9-10) for the osmium and ruthenium are well known oxides containing metals able to octavalent (OsO 4 and RuO 4 ). Relatively recently,
this was obtained an oxide of a third element for this subgroup of the periodic system - iron (FeO 4 ). What should be the acid-base properties
of these oxides (acidic, basic or nesoleobrazuyuschie oxides) and their redox properties? Answer explain. Provide the necessary reaction
equations.
6. (9-11) to extinguish fires apply conventional means (water, sand, etc.)., and various kinds of fire extinguishers, for example:
1) Carbon dioxide (carbonic cylinder of compressed gas);
2) powder (the fine powder mixture of soda and silica);
3) Fireworks (contains organic material and an oxidant).
What principle is the basis for the action of these fire-fighting equipment (ie, due to what is extinguished flame)? What you have to remember
to select the correct fire extinguishing agent (ie, cases in which some of these funds are not suitable)?
7. (9-11) A mixture of acetylene and ethane in volume of 2.0 liters was added 4 liters of hydrogen gas and the resulting mixture is passed over
a platinum catalyst. After the reaction, the total volume of the gas mixture was 3.6 liters. Determine the volume fractions of acetylene and
ethane in the feed mixture. All gas volumes are measured in the same conditions.
8. (10-11) after prolonged use of nickel-plated pans that are regularly put on a gas stove, pots surface in some places covered with
rust. Chemical analysis of the rust showed that it does not contain nickel. How can we explain this result?
9. (10-11) A density of hydrocarbon vapor in the air is 4.14. According to analysis, the hydrocarbon contains 90% carbon. Suggest A
hydrocarbon structure, if it is known that he does not discolour bromine water and bromination in the presence of a catalyst forms
a) one monobromo
b) two different monobromo.
What catalyst is used in the bromination?
Mathematics
In brackets, which classes recommended task; solve the problems of the older classes are also permitted.
1. (6-8) On the table lay a 100 apples, oranges and pears 99. It is so appropriate for children. The first took the apple, the second - a pear, the
third - an orange, an apple again next following it - pear behind him - orange.Next guys assorted fruit in the same order as long as the table is
not empty. How much could be pears? Explain your answer.
2. (6-8) Petya pocket a few coins. If Peter randomly pulled out 3 coins, among them there appear coin "1 ruble." If Peter randomly selects a 4
coins from his pocket, among them there appear coin "2 rubles." Peter pulled out 5 coins. What are these coins.
3. (6-9) Joe knows that to convert pounds to kilograms of the necessary weight in pounds divided by 2 and the resulting number is reduced by
10%. Hence Joe concluded that for the translation from kilograms to pounds need to weight in kilograms multiplied by 2 and that number
increased by 10%. How many percent of the correct value of the mass in pounds, he makes a mistake?
4. (8-11) plays two. At the beginning of the game there is one stick. The first player to break this stick into two parts. And so the players take
turns breaking into two parts any stick available to date. If, after breaking his wand, the player can make from all available sticks one or more
individual triangles (each - just three sticks), he won. Who are the players (first or second) can secure a victory, regardless of the actions of
another player?
5. (9-11) Fill into cells of a square 3 * 3 numbers so that when as the coefficients A , b , C ( A is not equal to 0) of the quadratic
equation ax 2 + BX + C = 0 take any number of rows (from left to right), column, or diagonal (top to bottom) of a square, then the resulting
equation will be at least one root.
6. (9-11) The following figure depicts the figure ABCD . Side AB , CD and AD of this figure - segments (and AB || CD and AD | CD ); BC arc of a circle, with any tangent to the arc cut off from the figure of a trapezoid or a rectangle. Explain how to draw a tangent to the arc BC , to

cut-off figure was the largest area.

Physics
Numbers in parentheses after the specified task classes in which this problem is recommended. Students grade 7 and under sufficient to
solve one "their" problem students grade 8 and older - two of "their" problem. Solve other tasks is also possible.
1. (6-8) along a straight road lay telephone wire. Two telephone operators decided to drag it. One end of a wire picked up and pulls it with a
speed of 3 km / h. A second telephone operator took the other end of the wire and went to catch up with the first telephone operator at a speed
of 4 km / h. How fast is moved along the road location fold wires?
2. (7-9) The wide lowland river is covered with ice, starting from the coast. The water surface is usually covered with ripples, and at the edge
of the ice splashing waves. Why is the ice surface as a result of an almost perfectly flat?
3. (8-9) In Moscow on Krasnopresnenskaya embankment there is a building of high-rise buildings - the tower "Federation". The website
dedicated to the construction, we can read: "Imagine: you walk in the door glass building, which is a 506-meter spire, sit down in one of four
transparent elevators and a speed of 4 meters per second climb to the observation deck, equipped with telescopes, where you can even see St.
Petersburg! " (http://www.federationtower.ru/about/excursion/ ).
Do not mess up the authors of this text anything? For reference: the radius of the Earth is approximately 6370 km, the distance Moscow - St.
Petersburg about 640 km.
4. (8-10) Model Railroad size is 100 times smaller than the real railroad in full size. All details of the model correspond to reality and are
made of the same materials as the real thing. How many times the kinetic energy of the moving train more than the kinetic energy of the train
model (model closely follows all movements committed by this train, a train passes a real part of the way on this rail in the same time as the
corresponding portion of the model on the model railroad ). Explanation for those who do not know the formula to calculate the kinetic
energy of a moving object: E kin = MV 2 / 2, where m - mass of the object, v - speed.
5. (8-11) at night in clear calm weather in the lake reflected the stars; that is, there are imaginary optical images of the stars "in depth" of the
lake. At what about "depth" (at what distance from the surface of the water) are images of these stars? Define this "depth" from the point of
view of a man standing on the shore of the lake at sa'moy water for stars that are located in the sky just above the location of the person.
6. (9-10) According to research by geologists from the inside of the Earth to its surface goes the average heat flux power of 0.05 W /
m 2 . During glacial periods the annual average temperature over the Arctic Ocean for many thousands of years, was equal to -20 O C. The
thermal conductivity of ice 0.9 W / (m * O C). Rate a maximum thickness of ice on the assumption that rainfall over the ocean do not fall, and
ice from the surface evaporates.
7. (9-11) around a closed vessel with small pore walls is normal atmospheric air. Establish equilibrium and the air within the vessel is of the
same composition, temperature and pressure, and around. If air is now located around the vessel, add methane (chemical formula CH 4 )
within the vessel for some time the pressure will rise. (It is assumed that the pressure and ambient temperature does not change). Explain why
there is a temporary increase in pressure. (This method of detection of a sharp increase in the concentration of methane earlier, before the
advent of modern gas analyzers used in coal mines. Vessel was connected to a porous mercury manometer, with increasing pressure mercury
closes an electrical circuit that is connected to the mechanism, the feed alarm.)
8. (9-11) Winnie the Pooh goes to visit Piglet through the forest on a straight road at a speed of 5 km / h. Just through the woods (no roads)
Winnie the Pooh can move at a speed of 1 km / h. At the road in the woods oak stands with bees. Winnie the Pooh bypasses oak
forest. However, he is not close to an oak closer than 1 km and reaches home Piglet in minimal time. How much quicker access to Winnie the
Pooh to his friend, if not afraid of bees? For numerical answer you may need approximate values arccos 0,2 = 1,369 and arcsin 0,2 = 0,201.
9. (10-11) In a vacuum in an inertial frame of reference, two identical particles are electrically charged at time t at the points, which
correspond to the radius vector r and - R . Particles in this point have velocities v 1 = R / T , and v 2 = - R / T . These particles are accelerated
in the same time equal to a 1 = R / T 2 and a 2 = - R / T 2 . At what minimum distance the particle is in motion?
Electrostatic interaction between the particles considered; electrodynamic effects due to the motion of the acceleration, neglected.

Chemistry
Participants are 8 classes (and younger) is proposed to solve the problem of 1-2 participants in grades 9-11 - 2-3 tasks. After each task
numbers in parentheses indicate which classes she recommended. To solve problems not of their class are allowed, but the tasks for the
younger class than yours, will be assessed fewer points.
1. (8-9) The flask was filled with hydrogen chloride gas under normal conditions and then connected by a tube with a large jar of water. Due
to the high solubility of hydrogen chloride, water completely filled the flask.Determine the mass fraction of hydrochloric acid in the resulting
solution, considering that the whole chloride remained in the flask.

2. (8-9) At your disposal are water, air, sulfur and calcium. Using these substances and their transformation products, as well as any laboratory
equipment, receive the maximum number of new substances. Write the reactions.
3. (8-10) The owners have forgotten in the country banks with distilled water, weak tea and sweet cucumber pickle and a bottle of cleaning
fluid for automotive glass. Concentrations of sugar in tea, and sodium chloride in the brine to 1 grams per liter approximately equal. Winter
came. In what order will freeze liquid? Answer justify.
4. (8-10) To clean often metallic mercury from impurities present therein, zinc, tin and lead, it is shaken with a saturated solution
of MERCURIC sulfate. Explain the method of cleaning. Write the equation of the corresponding reactions. Can I clean this method silver
metal impurities from the same metals?
5. (9-10) It is known that concentrated sulfuric acid is strongly absorbs water vapor, it is therefore often used for drying gases. Which of the
following gases may be, and which can not be dried with the help of sulfuric acid: hydrogen sulfide, sulfur oxide (IV), ethane, ethylene,
carbon monoxide (IV), carbon monoxide (II), ammonia, argon? Answer justify, write the corresponding equations of reactions. Suggest other
dehumidifiers for gases, which can not be dried with concentrated sulfuric acid.
6. (9-10) By dissolving the metal oxide in sulfuric acid solution with a mass fraction of 20% obtained metal salt solution with a mass fraction
of 22.64%. Identify metal, if it is known that metal oxide and sulfuric acid react fully formed and the average salt.
7. (10-11) The gas produced by burning a hydrocarbon 0.001 mol limit missed 200 g of a solution of calcium hydroxide with 0.148% mass
fraction. In this case, the mass of the precipitate obtained 0.2 g Which hydrocarbon burned?
8. (10-11) In the electrolysis of molten salt 3.4 g of a gas is released at the anode mass of 1.84 g at 100 O C and atmospheric pressure gas
volume is 1,093 liters, and when the temperature drops to 40 O C without change pressure volume decreases by 1.8 times. Gas is completely
absorbed by alkali solution to form two salts. By heating the obtained salt mixture (after separation from the solution), there is only one salt,
and the melt which has been subjected to electrolysis. Decide on what substances in question. Explain the unusual properties of the produced
gas. Write the equation mentioned reactions.
9. (10-11) The combustion of gas A having a density of hydrogen 14, a large amount of heat and formed two products - a colorless
liquid B odorless with a boiling point of 101.4 O C and a density of vapor and hydrogen gas 10 in . The interaction of gas A hydrogen gas may
be obtained F (density Hydrogen 15) which is completely absorbed when passing in bromine water.
1. Identify the substances A , B , B and D and write the equations of all the reactions mentioned.
2. Which catalyst useful for converting A to D ?
Mathematics
In brackets, which classes recommended task (and not necessarily solve all the problems absolutely his class); solve the problems of the older
classes are also permitted.

1. (6-7) Vanya was a certain amount of biscuits; how much he ate something, and then to visit him came Tanya, and the remaining biscuits
they shared equally. It turned out that Vanya ate five times more cookies than Tanya. What proportion of the total biscuit Vanya ate at the time
of arrival of Tanya?
2. (6-7) In the square 4 * 4 cells left half painted black, and the rest - in white. A single operation is permitted in the opposite color to repaint
all the cells within each rectangle. As for the three operations of the original coloring to get chess?

3. (8-9) Peter and Bob play the stock market. Some days, successful, and in those days the capital Petit increased by $ 1,000, and capital
Vasey - 10%. And other days failed - and then Petit capital is reduced by $ 2,000, and capital Wasi is reduced by 20%. After some time,
capital Petit was the same as was in the beginning. And what happened to the capital Wasi: it decreased, increased or remained the same?
4. (8-11) Given two potatoes of arbitrary shape and size. Prove that the surface of each of them can be routed to the wire so that they would
receive two curved ringlet (not necessarily flat), identical in shape and size.
5. (6-8) The left scales put two balls of radii 3 and 5, and on the right - a ball of radius 8. Which of the bowls outweigh? (All balls are made
entirely of the same material.) 6. (9-11) On the left the scales put two round coins, and on the right - another, so that the scales were
balanced. And what of the bowls outweigh, if each of the coins to replace the ball of the same radius? (All balls and coins are made entirely
from the same material, all of the coins are the same thickness).
7. (6-11) in a left to right is 31 purse, each for 100 coins. From one side of the coin purse shifted: 1 coin purses each of the right of it. In one
question, you can find the total number of coins in any set of purses. What is the minimum number of questions can be guaranteed to compute
the "lightweight" wallet?
8. (10-11) Bob responds Vieta theorem: "The sum of the three coefficients of the quadratic trinomial is one of its roots, and work - a different
way." Examiner: "infidels." Bob: "How wrong? I checked for a randomly selected trinomial, and everything turned out. " What could it be
trinomial if its coefficients - integers?
Physics
Numbers in parentheses after the specified task classes in which this problem is recommended. Students grade 7 and under sufficient to
solve one "their" problem students grades 8-10 - two of "their" problem students in grade 11 - three of "their" problem. You can solve the
problem of the senior classes.
1. (6-9) Why tea bag, if it pour boiling water - usually pops up, as if dipped in boiling water - usually sink? What's the difference?
2. (6-9) Scheduled train must pass railway section at a speed of 60 km / h. The train is late by 5 minutes. Calculate how many kilometers the
engineer need to drive at a speed of 70 km / h to eliminate the delay.
3. (7-11) mouse, cat and Fido are able to run on a plane at a constant rate, with cat runs faster than the mouse and the Beetle - fast cat. The cat
always runs in the direction of the mouse, and the Beetle - in the direction of a cat.
Initially, mouse, cat and Fido sit in a straight line, (Cat - between bugs and mice). It is known that if the mouse will run away from cats along
this straight line, never deviating, the cat catches Mouse before the cat catches up with the Beetle.
Could it be that the mouse, running more artful way, will be able to make sure that the cat will meet with bugs before catch a mouse? Explain
why.
4. (8-11) into the pockets of a mechanical watch the main mechanism to ensure an accurate count equal intervals of time ("escapement") is a
spring-loaded rotary pendulum, which must be rotated around its axis back and forth.
In order to increase the accuracy of the clock used at first sight unexpected technical solution - "escapement" is not fixed to the watch case,
and in the same gears, which is attached to the second hand. Why is it increases the precision?
5. (9-11) In space, away from the other bodies are three identical small balls with masses m and charge q each. Beads bonded in pairs in
three weightless and inextensible threads of equal length L . The system is at rest.Suddenly, one of the strands breaks. What amount of
acceleration to start moving balls immediately after the thread breakage?

6. (10-11) There are two identical uncharged capacitor and battery with emf U . Of them are allowed to collect any electrical wiring, and
assembly and disassembly repeat many times. It is not surprising, with the help of a sequence of such actions can charge one of the capacitor
to a voltage differs arbitrarily little from 2 U . How exactly does this have to do? Why it will lead to the desired result?
7. (10-11) on a horizontal table is a piece of paper with the printed text. Put a magnifying glass on the text (converging lens).
At the table, a man sits and looks at the text through a magnifying glass. Since the lens is far from the edge of the table, gaze direction is a
vertically disposed main optical axis of the lens angle of approximately 45 .

One sees that the image text in the loop a little distorted lines "bend-up" (left). Explain why it is "up" rather than "down" ("wrong" picture on
the right).
8. (10-11) The hydrogen molecule can exist in two stable states, which are called "ortho" (spins of the nuclei of two atoms in a molecule of
H 2 have the same direction) and "pair" (back in opposite directions). Molecule H 2can spontaneously reversible rebuild, the equilibrium ratio
depends on the temperature:
The configuration of
the molecule H 2
"Ortho"
"Pair"

Room temperature,
atmospheric pressure
75%
25%

20,4 K (boiling point of H 2 ),


atmospheric pressure
0.2%
99.8%

Hydrogen gas at room temperature, the liquid turned, cooled to T boiling = 20,4 K, and placed in an insulated container with the free removal of
vaporized hydrogen at atmospheric pressure. What happens to the liquid hydrogen in the vessel: Set equilibrium "o" / "pair" the concentration
of hydrogen or completely evaporated?
The rearrangement of molecules "ortho pair" comes with heat, specific heat of this process is q = 719 kJ / kg, the process is quite
slow. Assume that the specific heat of vaporization of H 2 under these conditions does not depend on the composition of the mixture and is
equal to L = 447 kJ / kg, and the difference of H 2 in the process of evaporation has no effect.
Astronomy
7 of the proposed tasks, we recommend that you choose the most interesting and answer them (students 8th grade and younger recommended
1-2 job, high school students - 2 or 3). The list of questions in each task can be used either as a single response plan or respond to all (or
some) questions individually.
Answers need to provide a reasonable number of examples and explanations of your choice. To list additional examples are not necessarily
(for them to assess the correct answer adds extra points).
1. Why 2009 UNESCO declared the International Year of Astronomy? Anniversaries What important events in the history of science, we are
celebrating this year?
2. Why does the moon - a small satellite of the Earth and the Sun is not huge? Why do some celestial bodies - someone's satellites, and others
- "in free flight"? Can satellites "move" from one host to another? And in general, if there are two bodies, someone around whom "should"
address?
3. What is studying science glaciology? Why glaciological studies around the world have become so relevant in recent times? (By the way,
the German word Glatze means 'bald'.)
4. As one of the possible underlying causes of the crash of the Airbus over the Atlantic (A-330 "Air France", 01.06.2009) considered the socalled "sprites". What is this phenomenon and why sprites can be dangerous to fly planes?

5. Astronomy of the 21st century call-wave. What wave at the disposal of astronomers already have? Yet what is not? No, I think, will never
be?
6. What are the new oceans may have on our planet Earth in the foreseeable future? When approximately? What on the contrary, may
disappear?
7. What astronomical phenomenon will always be for us "unexpected" (unpredicted)?
Chemistry
Participants are 8 classes (and younger) is proposed to solve the problem of 1-2 participants in grades 9-11 - 2-3 tasks. After each task
numbers in parentheses indicate which classes she recommended. You can also solve the problems of high school. Solved problems under
their class do not affect the assessment.
Bidders in chemistry issued (were printed on the back option) the periodic table, the table and the number of stress solubility of metals.
1. (8) The iron filings were mixed with an equal weight of sulfur. The resulting mixture was heated in a crucible without access of air. Got
dark brown mass and cooled it to room temperature. Will this mass is attracted by a magnet? Answer explain.
2. (8-9) In the two cylinders are carbon monoxide (II) and carbon monoxide (IV). Suggest several ways to determine in which a gas cylinder
is. Give, if necessary, the reactions.
3. (8-9) Give the equation for the reaction, allowing to carry out the following transformation. Identify the substance X . Each arrow
represents a single reaction.
Cu X CuCl 2 Cu (NO 3 ) 2 X Cu
4. (8-11) To a solution containing 4 g of copper sulfate, cadmium loaded plate. After complete displacement of copper from a solution of the
mass of the plate decreased by 3%. Determine the initial mass of the plate.
5. (8-10) The action of an excess of alkali to 23 g of solid material was obtained 4.48 liters of ammonia (eg. in.). Calcination of the same
amount of the substance formed of 14.2 g of a solid residue, which is an oxide containing 56.34% oxygen. Determine the formula of the
substance. The equations mentioned reactions.
6. (9-10) To understand the processes of transformation of minerals, it is often convenient (though not always justified) to submit their
formulas in the form of a combination of oxides (eg, CaO CO 2 instead of CaCO 3 , FeO Fe 2 O 3 instead of Fe 3 O 4 ). Expressed in a form
that is more familiar to the chemist, the following transformation.
Going danburite in datolite on the border with limestone:
CaB 2 Si 2 O 8 + Ca (OH) 2 = 2CaB (OH) SiO 4
Conversion of wollastonite, magnetite and quartz hedenbergite and andradite:
4CaSiO 3 + Fe 3 O 4 + SiO 2 = CaFeSi 2 O 6 + Ca 3 Fe 2 Si 3 O 12
7. (9-11) in a sealed vessel of unknown volume introduced 560 g of nitrogen, and 16 g of hydrogen. After heating to 500 C in the presence
of a catalyst in the reaction took 75% hydrogen and the balance established at a pressure of 15 atm. Determine the volume of the vessel.
8. (10-11) The reason for the tragedies in coal mines are most often explosions of methane-air mixtures, in which the volume fraction of
methane is 5-15%. Presents a danger to the mixture density 1.225 g / L? (NU). When calculating the molar mass of air considered equal to
29.0.
Why is this explosive mixture of? Suggest an explanation.
9. (11) Transcribe this scheme transformations: Draw the structural formulas of all these substances in the scheme, write reaction equations
and specify the conditions of their occurrence.
C4H8

CH 3 COOH

C4H8

C 2 H 5 COOH + CO 2

C 4 H 10

C4H8

CO (CH 3 ) 2 + CO 2

Вам также может понравиться